No boot info on ASA 5505

Hi all,
I received a firewall ASA 5505 and I have a problem with it. It always boot on rommon. When I do show boot I get those informations:
ciscoasa# show boot
BOOT variable =
Current BOOT variable =
CONFIG_FILE variable =
Current CONFIG_FILE variable =
I'm really confused on why it doesn't have any values registered for the boot.
Thanks,
Francis

Hi Francis,
Usually the firewall boots with the  the first application image that it finds in internal flash memory.
Try the following:
>Set the boot variable as:
hostname(config)# boot system url
url being:
{flash:/ | disk0:/ | disk1:/}[path/]filename
>Save the configuration:
hostname(config)# wr mem
>Now confirm that the boot image has been set:
show bootvar
Once done, reload the ASA and check if it is still going to the rommon mode.
Regards,
Shrinkhala

Similar Messages

  • ASA 5505 boot problem

    Hello,
    I have a problem with a firewall ASA 5505. I erased the flash via command line.
    Then there's what I did:
    Rebooting ....
    CISCO SYSTEMS
    Embedded BIOS Version 1.0 (12) 13 08/28/08 15:50:37.45
    Low Memory: 632 KB
    High Memory: 251 MB
    PCI Device Table.
    Bus Dev Func VendID DevID Class Irq
      00 01 00 1022 2080 Host Bridge
      00 01 02 1022 2082 Chipset En / Decrypt 11
      00 0C 00 1148 4320 Ethernet 11
      00 0D 00 177D 0003 Network En / Decrypt 10
      00 0F 00 1022 2090 ISA Bridge
      00 0F 02 1022 2092 IDE Controller
      00 0F 03 1022 2093 Audio 10
      00 0F 04 1022 2094 Serial Bus 9
      00 0F 05 1022 2095 Serial Bus 9
    Evaluating BIOS Options ...
    Launch BIOS Extension to setup ROMMON
    Cisco Systems ROMMON Version (1.0 (12) 13) # 0: Thu Aug 28 15:55:27 PDT 2008
    Platform ASA5505
    Use BREAK or ESC to interrupt boot.
    Use SPACE to begin boot immediately.
    Boot interrupted.
    Ethernet0 / 0
    MAC Address: 0024.c49e.7efb
    Link is UP
    Use? for help.
    rommon # 1> set
    ROMMON Variable Settings:
       ADDRESS = 192.168.1.1
       SERVER = 192.168.1.2
       GATEWAY = 0.0.0.0
       PORT = Ethernet0 / 0
       = Untagged VLAN
       IMAGE = asa902-k8.bin
       CONFIG =
       LINKTIMEOUT = 20
       PKTTIMEOUT = 4
       RETRY = 20
    rommon #5> tftpdnld
    ROMMON Variable Settings:
      ADDRESS=192.168.1.1
      SERVER=192.168.1.2
      GATEWAY=0.0.0.0
      PORT=Ethernet0/0
      VLAN=untagged
      IMAGE=asa902-k8.bin
      CONFIG=
      LINKTIMEOUT=20
      PKTTIMEOUT=4
      RETRY=20
    tftp [email protected]
    Received 27611136 bytes
    Launching TFTP Image...
    Cisco Security Appliance admin loader (3.0) #0: Thu Feb 21 13:10:34 PST 2013
    sumval(0x7a4 ) chksum(0x0   )md5(0x41130662 0xeacd779e 0x367c620f 0xa940e651)
    md5(0x1dc0f999 0x599b1643 0x10101012 0xd50a8b8d)
    Checksum verification on install image failed.
    CISCO SYSTEMS
    Embedded BIOS Version 1.0(12)13 08/28/08 15:50:37.45
    Low Memory: 632 KB
    High Memory: 251 MB
    PCI Device Table.
    Bus Dev Func VendID DevID Class              Irq
    00  01  00   1022   2080  Host Bridge
    00  01  02   1022   2082  Chipset En/Decrypt 11
    00  0C  00   1148   4320  Ethernet           11
    00  0D  00   177D   0003  Network En/Decrypt 10
    00  0F  00   1022   2090  ISA Bridge
    00  0F  02   1022   2092  IDE Controller
    00  0F  03   1022   2093  Audio              10
    00  0F  04   1022   2094  Serial Bus         9
    00  0F  05   1022   2095  Serial Bus         9
    Evaluating BIOS Options ...
    Launch BIOS Extension to setup ROMMON
    Cisco Systems ROMMON Version (1.0(12)13) #0: Thu Aug 28 15:55:27 PDT 2008
    Platform ASA5505
    Use BREAK or ESC to interrupt boot.
    Use SPACE to begin boot immediately.
    Launching BootLoader...
    Default configuration file contains 1 entry.
    Searching / for images to boot.
    No images in /
    Error 15: File not found
    unable to boot an image
    Default configuration file contains 1 entry.
    Searching / for images to boot.
    No images in /
    Error 15: File not found
    unable to boot an image
    Failsafe booting engaged.
    Default configuration file contains 1 entry.
    Searching / for images to boot.
    No images in /
    Error 15: File not found
    unable to boot an image
    CISCO SYSTEMS
    Embedded BIOS Version 1.0(12)13 08/28/08 15:50:37.45
    Low Memory: 632 KB
    High Memory: 251 MB
    PCI Device Table.
    Bus Dev Func VendID DevID Class              Irq
    00  01  00   1022   2080  Host Bridge
    00  01  02   1022   2082  Chipset En/Decrypt 11
    00  0C  00   1148   4320  Ethernet           11
    00  0D  00   177D   0003  Network En/Decrypt 10
    00  0F  00   1022   2090  ISA Bridge
    00  0F  02   1022   2092  IDE Controller
    00  0F  03   1022   2093  Audio              10
    00  0F  04   1022   2094  Serial Bus         9
    00  0F  05   1022   2095  Serial Bus         9
    Evaluating BIOS Options ...
    Launch BIOS Extension to setup ROMMON
    Cisco Systems ROMMON Version (1.0(12)13) #0: Thu Aug 28 15:55:27 PDT 2008
    Platform ASA5505
    Use BREAK or ESC to interrupt boot.
    Use SPACE to begin boot immediately.
    Boot interrupted.
    Ethernet0/0
    MAC Address: 0024.c49e.7efb
    Link is UP
    Use ? for help.
    rommon #0>
    How can I do to recover the device?
    Keeps rebooting endlessly!
    Thank you.
    Andrea

    When the checksum fails, either the IOS image you are trying to load is corrupted (or incomplete) or the tftp download corrupted the image.
    I would start by using an older, smaller image. Such as 8.2(5). Tftp that image to get the ASA booted, then upgrade to 9.0(2).

  • Internet Connection Became Slow after Introduction of Cisco ASA 5505 to the Network

    I configured a Cisco ASA 5505 (Version Cisco Adaptive Security Appliance Software Version 7.2(3)
    Device Manager Version 5.2(3)
    in transparent firewall mode and inserted after Cisco 1700 router. However, the internet connection became very slow and users are compaining that they cannot load any pages.
    My setup looks like:
    Internet --> Cisco 1700 --> Cisco ASA 5505 --> LAN
    The license information is:
    Licensed features for this platform:
    Maximum Physical Interfaces : 8
    VLANs                       : 3, DMZ Restricted
    Inside Hosts                : Unlimited
    Failover                    : Disabled
    VPN-DES                     : Enabled
    VPN-3DES-AES                : Enabled
    VPN Peers                   : 10
    WebVPN Peers                : 2
    Dual ISPs                   : Disabled
    VLAN Trunk Ports            : 0
    This platform has a Base license.
    The flash activation key is the SAME as the running key.
    My running-config looks like:
    ASA Version 7.2(3)
    firewall transparent
    hostname ciscoasa
    domain-name default.domain.invalid
    enable password 8Ry2YjIyt7RRXU24 encrypted
    names
    interface Vlan1
    nameif inside
    security-level 100
    no shut
    interface Vlan2
    nameif outside
    security-level 0
    no shut
    interface Ethernet0/0
    switchport access vlan 2
    no shut
    interface Ethernet0/1
    no shut
    interface Ethernet0/2
    no shut
    interface Ethernet0/3
    no shut
    interface Ethernet0/4
    no shut
    interface Ethernet0/5
    no shut
    interface Ethernet0/6
    no shut
    interface Ethernet0/7
    no shut
    passwd 2KFQnbNIdI.2KYOU encrypted
    regex urllist1 ".*\.([Ee][Xx][Ee]|[Cc][Oo][Mm]|[Bb][Aa][Tt]) HTTP/1.[01]"
    regex urllist2 ".*\.([Pp][Ii][Ff]|[Vv][Bb][Ss]|[Ww][Ss][Hh]) HTTP/1.[01]"
    regex urllist3 ".*\.([Dd][Oo][Cc]|[Xx][Ll][Ss]|[Pp][Pp][Tt]) HTTP/1.[01]"
    regex urllist4 ".*\.([Zz][Ii][Pp]|[Tt][Aa][Rr]|[Tt][Gg][Zz]) HTTP/1.[01]"
    regex domainlist1 "\.facebook\.com"
    regex domainlist2 "\.diretube\.com"
    regex domainlist3 "\.youtube\.com"
    regex domainlist4 "\.vimeo\.com"
    regex applicationheader "application/.*"
    regex contenttype "Content-Type"
    ftp mode passive
    dns server-group DefaultDNS
    domain-name default.domain.invalid
    access-list outside_in extended permit ip any any
    access-list inside_mpc extended permit tcp any any eq www
    access-list inside_mpc extended permit tcp any any eq 8080
    pager lines 24
    mtu outside 1500
    mtu inside 1500
    ip address 192.168.1.254 255.255.255.0
    icmp unreachable rate-limit 1 burst-size 1
    no asdm history enable
    arp timeout 14400
    access-group outside_in in interface outside
    route outside 0.0.0.0 0.0.0.0 192.168.1.1 1
    timeout xlate 3:00:00
    timeout conn 1:00:00 half-closed 0:10:00 udp 0:02:00 icmp 0:00:02
    timeout sunrpc 0:10:00 h323 0:05:00 h225 1:00:00 mgcp 0:05:00 mgcp-pat 0:05:00
    timeout sip 0:30:00 sip_media 0:02:00 sip-invite 0:03:00 sip-disconnect 0:02:00
    timeout uauth 0:05:00 absolute
    no snmp-server location
    no snmp-server contact
    snmp-server enable traps snmp authentication linkup linkdown coldstart
    telnet timeout 5
    ssh timeout 5
    console timeout 0
    class-map type regex match-any DomainBlockList
    match regex domainlist1
    match regex domainlist2
    match regex domainlist3
    match regex domainlist4
    class-map type inspect http match-all BlockDomainsClass
    match request header host regex class DomainBlockList
    class-map type regex match-any URLBlockList
    match regex urllist1
    match regex urllist2
    match regex urllist3
    match regex urllist4
    class-map inspection_default
    match default-inspection-traffic
    class-map type inspect http match-all AppHeaderClass
    match response header regex contenttype regex applicationheader
    class-map httptraffic
    match access-list inside_mpc
    class-map type inspect http match-all BlockURLsClass
    match request uri regex class URLBlockList
    policy-map type inspect dns preset_dns_map
    parameters
      message-length maximum 512
    policy-map type inspect http http_inspection_policy
    parameters
      protocol-violation action drop-connection
    class AppHeaderClass
      drop-connection log
    match request method connect
      drop-connection log
    class BlockDomainsClass
      reset log
    class BlockURLsClass
      reset log
    policy-map global_policy
    class inspection_default
      inspect dns preset_dns_map
      inspect ftp
      inspect h323 h225
      inspect h323 ras
      inspect netbios
      inspect rsh
      inspect rtsp
      inspect skinny
      inspect esmtp
      inspect sqlnet
      inspect sunrpc
      inspect tftp
      inspect sip
      inspect xdmcp
    policy-map inside-policy
    class httptraffic
      inspect http http_inspection_policy
    service-policy global_policy global
    service-policy inside-policy interface inside
    prompt hostname context
    Cryptochecksum:8ab1a53df6ae3c202aee236d6080edfd
    : end
    Could the slow internet connection be due to license limitations? Or is there something wrong with my configuration?
    Please see the configuration and help.
    Thanks

    I have re-configured the ASA 5505 yesterday and so far it's working fine. I am not sure if the problem will re-appear later on. Anyways here is my sh tech-support
    ciscoasa# sh tech-support
    Cisco Adaptive Security Appliance Software Version 7.2(3)
    Device Manager Version 5.2(3)
    Compiled on Wed 15-Aug-07 16:08 by builders
    System image file is "disk0:/asa723-k8.bin"
    Config file at boot was "startup-config"
    ciscoasa up 14 hours 16 mins
    Hardware:   ASA5505, 256 MB RAM, CPU Geode 500 MHz
    Internal ATA Compact Flash, 128MB
    BIOS Flash M50FW080 @ 0xffe00000, 1024KB
    Encryption hardware device : Cisco ASA-5505 on-board accelerator (revision 0x0)
                                 Boot microcode   : CNlite-MC-Boot-Cisco-1.2
                                 SSL/IKE microcode: CNlite-MC-IPSEC-Admin-3.03
                                 IPSec microcode  : CNlite-MC-IPSECm-MAIN-2.04
    0: Int: Internal-Data0/0    : address is 001f.9ee8.ffa2, irq 11
    1: Ext: Ethernet0/0         : address is 001f.9ee8.ff9a, irq 255
    2: Ext: Ethernet0/1         : address is 001f.9ee8.ff9b, irq 255
    3: Ext: Ethernet0/2         : address is 001f.9ee8.ff9c, irq 255
    4: Ext: Ethernet0/3         : address is 001f.9ee8.ff9d, irq 255
    5: Ext: Ethernet0/4         : address is 001f.9ee8.ff9e, irq 255
    6: Ext: Ethernet0/5         : address is 001f.9ee8.ff9f, irq 255
    <--- More --->
    7: Ext: Ethernet0/6         : address is 001f.9ee8.ffa0, irq 255
    8: Ext: Ethernet0/7         : address is 001f.9ee8.ffa1, irq 255
    9: Int: Internal-Data0/1    : address is 0000.0003.0002, irq 255
    10: Int: Not used            : irq 255
    11: Int: Not used            : irq 255
    Licensed features for this platform:
    Maximum Physical Interfaces : 8        
    VLANs                       : 3, DMZ Restricted
    Inside Hosts                : Unlimited
    Failover                    : Disabled
    VPN-DES                     : Enabled  
    VPN-3DES-AES                : Enabled  
    VPN Peers                   : 10       
    WebVPN Peers                : 2        
    Dual ISPs                   : Disabled 
    VLAN Trunk Ports            : 0        
    This platform has a Base license.
    Serial Number: JMX1211Z2N4
    Running Activation Key: 0xaf0ed046 0xbcf18ebf 0x80b38508 0xba785cc0 0x05250493
    Configuration register is 0x1
    Configuration has not been modified since last system restart.
    <--- More --->
    ------------------ show clock ------------------
    18:32:58.254 UTC Tue Nov 26 2013
    ------------------ show memory ------------------
    Free memory:       199837144 bytes (74%)
    Used memory:        68598312 bytes (26%)
    Total memory:      268435456 bytes (100%)
    ------------------ show conn count ------------------
    1041 in use, 2469 most used
    ------------------ show xlate count ------------------
    0 in use, 0 most used
    ------------------ show blocks ------------------
      SIZE    MAX    LOW    CNT
         0    100     68    100
    <--- More --->
         4    300    299    299
        80    100     92    100
       256    100     94    100
      1550   6174   6166   6174
      2048   1124    551    612
    ------------------ show blocks queue history detail ------------------
    History buffer memory usage: 2136 bytes (default)
    ------------------ show interface ------------------
    Interface Internal-Data0/0 "", is up, line protocol is up
      Hardware is y88acs06, BW 1000 Mbps
    (Full-duplex), (1000 Mbps)
    MAC address 001f.9ee8.ffa2, MTU not set
    IP address unassigned
    18491855 packets input, 11769262614 bytes, 0 no buffer
    Received 213772 broadcasts, 0 runts, 0 giants
    0 input errors, 0 CRC, 0 frame, 0 overrun, 0 ignored, 0 abort
    0 L2 decode drops, 0 demux drops
    18185861 packets output, 11626494317 bytes, 0 underruns
    0 output errors, 0 collisions, 0 interface resets
    0 late collisions, 0 deferred
    <--- More --->
    0 input reset drops, 0 output reset drops
    input queue (curr/max packets): hardware (0/0) software (0/0)
    output queue (curr/max packets): hardware (0/55) software (0/0)
      Control Point Interface States:
    Interface number is unassigned
    Interface Internal-Data0/1 "", is administratively down, line protocol is up
      Hardware is 88E6095, BW 1000 Mbps
    (Full-duplex), (1000 Mbps)
    MAC address 0000.0003.0002, MTU not set
    IP address unassigned
    18184216 packets input, 11625360131 bytes, 0 no buffer
    Received 206655 broadcasts, 0 runts, 0 giants
    0 input errors, 0 CRC, 0 frame, 0 overrun, 0 ignored, 0 abort
    0 switch ingress policy drops
    18490057 packets output, 11768078777 bytes, 0 underruns
    0 output errors, 0 collisions, 0 interface resets
    0 babbles, 0 late collisions, 0 deferred
    0 lost carrier, 0 no carrier
    0 switch egress policy drops
      Control Point Interface States:
    Interface number is unassigned
    Interface Loopback0 "_internal_loopback", is up, line protocol is up
      Hardware is VirtualMAC address 0000.0000.0000, MTU 1500
    IP address 127.1.0.1, subnet mask 255.255.0.0
    <--- More --->
      Traffic Statistics for "_internal_loopback":
    1 packets input, 28 bytes
    1 packets output, 28 bytes
    1 packets dropped
          1 minute input rate 0 pkts/sec,  0 bytes/sec
          1 minute output rate 0 pkts/sec,  0 bytes/sec
          1 minute drop rate, 0 pkts/sec
          5 minute input rate 0 pkts/sec,  0 bytes/sec
          5 minute output rate 0 pkts/sec,  0 bytes/sec
          5 minute drop rate, 0 pkts/sec
      Control Point Interface States:
    Interface number is 28
    Interface config status is active
    Interface state is active
    Interface Vlan1 "inside", is up, line protocol is up
      Hardware is EtherSVI
    MAC address 001f.9ee8.ffa2, MTU 1500
    IP address 192.168.1.254, subnet mask 255.255.255.0
      Traffic Statistics for "inside":
    7742275 packets input, 903584114 bytes
    10645034 packets output, 10347291114 bytes
    184883 packets dropped
          1 minute input rate 320 pkts/sec,  35404 bytes/sec
          1 minute output rate 325 pkts/sec,  313317 bytes/sec
    <--- More --->
          1 minute drop rate, 17 pkts/sec
          5 minute input rate 399 pkts/sec,  59676 bytes/sec
          5 minute output rate 483 pkts/sec,  503200 bytes/sec
          5 minute drop rate, 9 pkts/sec
      Control Point Interface States:
    Interface number is 1
    Interface config status is active
    Interface state is active
    Interface Vlan2 "outside", is up, line protocol is up
      Hardware is EtherSVI
    MAC address 001f.9ee8.ffa3, MTU 1500
    IP address 192.168.1.254, subnet mask 255.255.255.0
      Traffic Statistics for "outside":
    10750090 packets input, 10432619059 bytes
    7541331 packets output, 870613684 bytes
    109911 packets dropped
          1 minute input rate 328 pkts/sec,  313770 bytes/sec
          1 minute output rate 301 pkts/sec,  32459 bytes/sec
          1 minute drop rate, 2 pkts/sec
          5 minute input rate 485 pkts/sec,  503789 bytes/sec
          5 minute output rate 387 pkts/sec,  57681 bytes/sec
          5 minute drop rate, 2 pkts/sec
      Control Point Interface States:
    Interface number is 2
    <--- More --->
    Interface config status is active
    Interface state is active
    Interface Ethernet0/0 "", is up, line protocol is up
      Hardware is 88E6095, BW 100 Mbps
    Auto-Duplex(Full-duplex), Auto-Speed(100 Mbps)
    Available but not configured via nameif
    MAC address 001f.9ee8.ff9a, MTU not set
    IP address unassigned
    10749794 packets input, 10630700889 bytes, 0 no buffer
    Received 2506 broadcasts, 0 runts, 0 giants
    0 input errors, 0 CRC, 0 frame, 0 overrun, 0 ignored, 0 abort
    0 L2 decode drops
    3 switch ingress policy drops
    7541070 packets output, 1028190148 bytes, 0 underruns
    0 output errors, 0 collisions, 0 interface resets
    0 babbles, 0 late collisions, 0 deferred
    0 lost carrier, 0 no carrier
    0 rate limit drops
    0 switch egress policy drops
      Control Point Interface States:
    Interface number is unassigned
    Interface Ethernet0/1 "", is up, line protocol is up
      Hardware is 88E6095, BW 100 Mbps
    Auto-Duplex(Full-duplex), Auto-Speed(100 Mbps)
    <--- More --->
    Available but not configured via nameif
    MAC address 001f.9ee8.ff9b, MTU not set
    IP address unassigned
    7741977 packets input, 1064586806 bytes, 0 no buffer
    Received 211282 broadcasts, 0 runts, 0 giants
    0 input errors, 0 CRC, 0 frame, 0 overrun, 0 ignored, 0 abort
    0 L2 decode drops
    0 switch ingress policy drops
    10644663 packets output, 10543362751 bytes, 0 underruns
    0 output errors, 0 collisions, 0 interface resets
    0 babbles, 0 late collisions, 0 deferred
    0 lost carrier, 0 no carrier
    0 rate limit drops
    0 switch egress policy drops
      Control Point Interface States:
    Interface number is unassigned
    Interface Ethernet0/2 "", is down, line protocol is down
      Hardware is 88E6095, BW 100 Mbps
    Auto-Duplex, Auto-Speed
    Available but not configured via nameif
    MAC address 001f.9ee8.ff9c, MTU not set
    IP address unassigned
    0 packets input, 0 bytes, 0 no buffer
    Received 0 broadcasts, 0 runts, 0 giants
    <--- More --->
    0 input errors, 0 CRC, 0 frame, 0 overrun, 0 ignored, 0 abort
    0 L2 decode drops
    0 switch ingress policy drops
    0 packets output, 0 bytes, 0 underruns
    0 output errors, 0 collisions, 0 interface resets
    0 babbles, 0 late collisions, 0 deferred
    0 lost carrier, 0 no carrier
    0 rate limit drops
    0 switch egress policy drops
      Control Point Interface States:
    Interface number is unassigned
    Interface Ethernet0/3 "", is down, line protocol is down
      Hardware is 88E6095, BW 100 Mbps
    Auto-Duplex, Auto-Speed
    Available but not configured via nameif
    MAC address 001f.9ee8.ff9d, MTU not set
    IP address unassigned
    0 packets input, 0 bytes, 0 no buffer
    Received 0 broadcasts, 0 runts, 0 giants
    0 input errors, 0 CRC, 0 frame, 0 overrun, 0 ignored, 0 abort
    0 L2 decode drops
    0 switch ingress policy drops
    0 packets output, 0 bytes, 0 underruns
    0 output errors, 0 collisions, 0 interface resets
    <--- More --->
    0 babbles, 0 late collisions, 0 deferred
    0 lost carrier, 0 no carrier
    0 rate limit drops
    0 switch egress policy drops
      Control Point Interface States:
    Interface number is unassigned
    Interface Ethernet0/4 "", is down, line protocol is down
      Hardware is 88E6095, BW 100 Mbps
    Auto-Duplex, Auto-Speed
    Available but not configured via nameif
    MAC address 001f.9ee8.ff9e, MTU not set
    IP address unassigned
    0 packets input, 0 bytes, 0 no buffer
    Received 0 broadcasts, 0 runts, 0 giants
    0 input errors, 0 CRC, 0 frame, 0 overrun, 0 ignored, 0 abort
    0 L2 decode drops
    0 switch ingress policy drops
    0 packets output, 0 bytes, 0 underruns
    0 output errors, 0 collisions, 0 interface resets
    0 babbles, 0 late collisions, 0 deferred
    0 lost carrier, 0 no carrier
    0 rate limit drops
    0 switch egress policy drops
      Control Point Interface States:
    <--- More --->
    Interface number is unassigned
    Interface Ethernet0/5 "", is down, line protocol is down
      Hardware is 88E6095, BW 100 Mbps
    Auto-Duplex, Auto-Speed
    Available but not configured via nameif
    MAC address 001f.9ee8.ff9f, MTU not set
    IP address unassigned
    0 packets input, 0 bytes, 0 no buffer
    Received 0 broadcasts, 0 runts, 0 giants
    0 input errors, 0 CRC, 0 frame, 0 overrun, 0 ignored, 0 abort
    0 L2 decode drops
    0 switch ingress policy drops
    0 packets output, 0 bytes, 0 underruns
    0 output errors, 0 collisions, 0 interface resets
    0 babbles, 0 late collisions, 0 deferred
    0 lost carrier, 0 no carrier
    0 rate limit drops
    0 switch egress policy drops
      Control Point Interface States:
    Interface number is unassigned
    Interface Ethernet0/6 "", is down, line protocol is down
      Hardware is 88E6095, BW 100 Mbps
    Auto-Duplex, Auto-Speed
    Available but not configured via nameif
    <--- More --->
    MAC address 001f.9ee8.ffa0, MTU not set
    IP address unassigned
    0 packets input, 0 bytes, 0 no buffer
    Received 0 broadcasts, 0 runts, 0 giants
    0 input errors, 0 CRC, 0 frame, 0 overrun, 0 ignored, 0 abort
    0 L2 decode drops
    0 switch ingress policy drops
    0 packets output, 0 bytes, 0 underruns
    0 output errors, 0 collisions, 0 interface resets
    0 babbles, 0 late collisions, 0 deferred
    0 lost carrier, 0 no carrier
    0 rate limit drops
    0 switch egress policy drops
      Control Point Interface States:
    Interface number is unassigned
    Interface Ethernet0/7 "", is down, line protocol is down
      Hardware is 88E6095, BW 100 Mbps
    Auto-Duplex, Auto-Speed
    Available but not configured via nameif
    MAC address 001f.9ee8.ffa1, MTU not set
    IP address unassigned
    0 packets input, 0 bytes, 0 no buffer
    Received 0 broadcasts, 0 runts, 0 giants
    0 input errors, 0 CRC, 0 frame, 0 overrun, 0 ignored, 0 abort
    <--- More --->
    0 L2 decode drops
    0 switch ingress policy drops
    0 packets output, 0 bytes, 0 underruns
    0 output errors, 0 collisions, 0 interface resets
    0 babbles, 0 late collisions, 0 deferred
    0 lost carrier, 0 no carrier
    0 rate limit drops
    0 switch egress policy drops
      Control Point Interface States:
    Interface number is unassigned
    ------------------ show cpu usage ------------------
    CPU utilization for 5 seconds = 12%; 1 minute: 11%; 5 minutes: 11%
    ------------------ show cpu hogging process ------------------
    Process:      Dispatch Unit, NUMHOG: 1, MAXHOG: 133, LASTHOG: 140
    LASTHOG At:   04:45:59 UTC Nov 26 2013
    PC:           8be0f7
    Traceback:    8bed19  8bf553  302b87  3030a5  2fad69  7674bf  75ca16
                  c6251d  c62a4c  c62f6c  75c653  767820  797f64  769c85
    <--- More --->
    ------------------ show process ------------------
        PC       SP       STATE       Runtime    SBASE     Stack Process
    Mwe 00c9bb24 01bb8700 013e3250          0 01733fc8 15616/16384 emweb/cifs
    Lwe 001072ac 0176f9c4 013e32d0          0 0176d9f0 8132/8192 block_diag
    Mrd 00223a67 01783d5c 013e33b0     314854 0177be18 25752/32768 Dispatch Unit
    Msi 00f82847 01b07b84 013e3250        229 01b05bc0 7984/8192 y88acs06 OneSec Thread
    Mwe 0011b1a5 01b09cfc 013e3250          0 01b07d88 7864/8192 Reload Control Thread
    Mwe 00120606 01b1260c 013e5258          0 01b10988 7256/8192 aaa
    Mwe 001486aa 01b19404 013e5ae8          0 01b15450 16020/16384 CMGR Server Process
    Mwe 0014c3c5 01b1b4d4 013e3250          0 01b19570 7968/8192 CMGR Timer Process
    Lwe 002227a1 01b239b4 013ee360          0 01b219f0 7524/8192 dbgtrace
    Mwe 004e1ba5 01b29c34 013e3250        157 01b27d50 6436/8192 eswilp_svi_init
    Mwe 01064b1d 01b4a7f4 013e3250          0 01b48890 7848/8192 Chunk Manager
    Msi 008b61b6 01b52d54 013e3250        230 01b50da0 7856/8192 PIX Garbage Collector
    Lsi 00ecb6ac 01b54e94 013e3250         12 01b52ec0 7552/8192 route_process
    Mwe 008a5ddc 01b5dc04 0133b430          0 01b5bc40 8116/8192 IP Address Assign
    Mwe 00acb779 01b60604 01346e10          0 01b5e640 8116/8192 QoS Support Module
    Mwe 0091eba9 01b6275c 0133c530          0 01b60798 8116/8192 Client Update Task
    Lwe 01083c8e 01b656d4 013e3250     123088 01b63770 7840/8192 Checkheaps
    Mwe 00acfd7d 01b6b824 013e3250        623 01b69ad0 3476/8192 Quack process
    Mwe 00b2a260 01b6dad4 013e3250         22 01b6bbf0 7364/8192 Session Manager
    Mwe 00c55efd 01b78564 031d0478          4 01b74a50 14768/16384 uauth
    <--- More --->
    Mwe 00be3c9e 01b7aaec 0135c010          0 01b78b28 7524/8192 Uauth_Proxy
    Mwe 00c52759 01b80e0c 01361770          0 01b7ee88 7712/8192 SMTP
    Mwe 00c3f7b9 01b82eec 01361710          0 01b80fa8 7412/8192 Logger
    Mwe 00c3fd26 01b8502c 013e3250          0 01b830c8 7492/8192 Thread Logger
    Mwe 00f62272 01b9596c 013ac520          0 01b939c8 7188/8192 vpnlb_thread
    Msi 00b4097c 01c598c4 013e3250        190 01c578f0 8000/8192 emweb/cifs_timer
    Msi 005bd338 017a909c 013e3250      25855 017a7108 7412/8192 arp_timer
    Mwe 005c76bc 01b486e4 013fba50      20643 01b46770 7348/8192 arp_forward_thread
    Mwe 00c5a919 023fa5fc 013619e0          0 023f8648 7968/8192 tcp_fast
    Mwe 00c5a6e5 023fc624 013619e0          0 023fa670 7968/8192 tcp_slow
    Mwe 00c754d1 0240d42c 013628a0          0 0240b478 8100/8192 udp_timer
    Mwe 0019cb17 01b404a4 013e3250          0 01b3e530 7984/8192 CTCP Timer process
    Mwe 00efe8b3 0308c15c 013e3250          0 0308a208 7952/8192 L2TP data daemon
    Mwe 00efef23 0308e194 013e3250          0 0308c230 7968/8192 L2TP mgmt daemon
    Mwe 00eea02b 030c62ac 013a5c10         43 030c2338 16244/16384 ppp_timer_thread
    Msi 00f62d57 030c82f4 013e3250        264 030c6360 7924/8192 vpnlb_timer_thread
    Mwe 001b96e6 01b7cbbc 01b1e9c8          1 01b7ac48 7728/8192 IPsec message handler
    Msi 001c9bac 01b8d4dc 013e3250       2917 01b8b548 7648/8192 CTM message handler
    Mwe 00af93b8 031465b4 013e3250          0 03144640 7984/8192 ICMP event handler
    Mwe 00831003 0314a724 013e3250        387 031467b0 16100/16384 IP Background
    Mwe 0021b267 031a83c4 013123c0         31 03188450 123488/131072 tmatch compile thread
    Mwe 009f2405 03290044 013e3250          0 0328c0c0 16072/16384 Crypto PKI RECV
    Mwe 009f305a 03294144 013e3250          0 032901e0 16040/16384 Crypto CA
    Mwe 0064d4fd 01b3e24c 013e3250          8 01b3c2f8 7508/8192 ESW_MRVL switch interrupt service
    <--- More --->
    Msi 00646f5c 032c134c 013e3250    3059378 032bf448 7184/8192 esw_stats
    Lsi 008cbb80 032dc704 013e3250          3 032da730 7908/8192 uauth_urlb clean
    Lwe 008afee7 034a0914 013e3250        197 0349e9b0 6636/8192 pm_timer_thread
    Mwe 0052f0bf 034a35ac 013e3250          0 034a1648 7968/8192 IKE Timekeeper
    Mwe 00520f6b 034a8adc 0132e2b0          0 034a4e38 15448/16384 IKE Daemon
    Mwe 00bf5c78 034ac7ac 01360680          0 034aa7f8 8100/8192 RADIUS Proxy Event Daemon
    Mwe 00bc32de 034ae79c 034dcbe0          0 034ac918 7208/8192 RADIUS Proxy Listener
    Mwe 00bf5e0f 034b099c 013e3250          0 034aea38 7968/8192 RADIUS Proxy Time Keeper
    Mwe 005aac4c 034b3154 013fb980          0 034b1250 7492/8192 Integrity FW Task
    M*  008550a5 0009fefc 013e33b0       3183 034e3b20 24896/32768 ci/console
    Msi 008eb694 034ed9d4 013e3250       2370 034ebc40 6176/8192 update_cpu_usage
    Msi 008e6415 034f7dac 013e3250       1096 034f5eb8 6124/8192 NIC status poll
    Mwe 005b63e6 03517d1c 013fbd10       1963 03515d78 7636/8192 IP Thread
    Mwe 005becbe 03519e4c 013fbcb0          3 03517e98 7384/8192 ARP Thread
    Mwe 004c2b36 0351befc 013fbae0          0 03519fe8 7864/8192 icmp_thread
    Mwe 00c7722e 0351e06c 013e3250          0 0351c108 7848/8192 udp_thread
    Mwe 00c5d126 0352008c 013fbd00          0 0351e228 7688/8192 tcp_thread
    Mwe 00bc32de 03a6982c 03a5ee18          0 03a679b8 7512/8192 EAPoUDP-sock
    Mwe 00266c15 03a6b614 013e3250          0 03a699e0 7032/8192 EAPoUDP
    Mwe 005a6728 01b27b94 013e3250          0 01b25c30 7968/8192 Integrity Fw Timer Thread
    -     -        -         -      47686621    -         -     scheduler
    -     -        -         -      51253819    -         -     total elapsed
    ------------------ show failover ------------------
    <--- More --->
    ERROR: Command requires failover license
    ------------------ show traffic ------------------
    inside:
    received (in 51429.740 secs):
    7749585 packets905087345 bytes
    67 pkts/sec17013 bytes/sec
    transmitted (in 51429.740 secs):
    10653162 packets10355908020 bytes
    40 pkts/sec201026 bytes/sec
          1 minute input rate 412 pkts/sec,  51803 bytes/sec
          1 minute output rate 475 pkts/sec,  522952 bytes/sec
          1 minute drop rate, 24 pkts/sec
          5 minute input rate 399 pkts/sec,  59676 bytes/sec
          5 minute output rate 483 pkts/sec,  503200 bytes/sec
          5 minute drop rate, 9 pkts/sec
    outside:
    received (in 51430.240 secs):
    10758403 packets10441440193 bytes
    42 pkts/sec203021 bytes/sec
    transmitted (in 51430.240 secs):
    7548339 packets872053854 bytes
    <--- More --->
    63 pkts/sec16037 bytes/sec
          1 minute input rate 479 pkts/sec,  523680 bytes/sec
          1 minute output rate 387 pkts/sec,  46796 bytes/sec
          1 minute drop rate, 3 pkts/sec
          5 minute input rate 485 pkts/sec,  503789 bytes/sec
          5 minute output rate 387 pkts/sec,  57681 bytes/sec
          5 minute drop rate, 2 pkts/sec
    _internal_loopback:
    received (in 51430.740 secs):
    1 packets28 bytes
    0 pkts/sec0 bytes/sec
    transmitted (in 51430.740 secs):
    1 packets28 bytes
    0 pkts/sec0 bytes/sec
          1 minute input rate 0 pkts/sec,  0 bytes/sec
          1 minute output rate 0 pkts/sec,  0 bytes/sec
          1 minute drop rate, 0 pkts/sec
          5 minute input rate 0 pkts/sec,  0 bytes/sec
          5 minute output rate 0 pkts/sec,  0 bytes/sec
          5 minute drop rate, 0 pkts/sec
    Aggregated Traffic on Physical Interface
    <--- More --->
    Ethernet0/0:
    received (in 51431.740 secs):
    10758462 packets10640075825 bytes
    42 pkts/sec206042 bytes/sec
    transmitted (in 51431.740 secs):
    7548383 packets1029818127 bytes
    63 pkts/sec20023 bytes/sec
          1 minute input rate 485 pkts/sec,  537048 bytes/sec
          1 minute output rate 395 pkts/sec,  54546 bytes/sec
          1 minute drop rate, 0 pkts/sec
          5 minute input rate 485 pkts/sec,  511723 bytes/sec
          5 minute output rate 387 pkts/sec,  65495 bytes/sec
          5 minute drop rate, 0 pkts/sec
    Ethernet0/1:
    received (in 51433.570 secs):
    7749780 packets1066328930 bytes
    67 pkts/sec20064 bytes/sec
    transmitted (in 51433.570 secs):
    10653359 packets10552787020 bytes
    40 pkts/sec205006 bytes/sec
          1 minute input rate 419 pkts/sec,  59621 bytes/sec
          1 minute output rate 480 pkts/sec,  533950 bytes/sec
          1 minute drop rate, 0 pkts/sec
          5 minute input rate 399 pkts/sec,  67618 bytes/sec
    <--- More --->
          5 minute output rate 482 pkts/sec,  511073 bytes/sec
          5 minute drop rate, 0 pkts/sec
    Ethernet0/2:
    received (in 51434.730 secs):
    0 packets0 bytes
    0 pkts/sec0 bytes/sec
    transmitted (in 51434.730 secs):
    0 packets0 bytes
    0 pkts/sec0 bytes/sec
          1 minute input rate 0 pkts/sec,  0 bytes/sec
          1 minute output rate 0 pkts/sec,  0 bytes/sec
          1 minute drop rate, 0 pkts/sec
          5 minute input rate 0 pkts/sec,  0 bytes/sec
          5 minute output rate 0 pkts/sec,  0 bytes/sec
          5 minute drop rate, 0 pkts/sec
    Ethernet0/3:
    received (in 51434.730 secs):
    0 packets0 bytes
    0 pkts/sec0 bytes/sec
    transmitted (in 51434.730 secs):
    0 packets0 bytes
    0 pkts/sec0 bytes/sec
          1 minute input rate 0 pkts/sec,  0 bytes/sec
          1 minute output rate 0 pkts/sec,  0 bytes/sec
    <--- More --->
          1 minute drop rate, 0 pkts/sec
          5 minute input rate 0 pkts/sec,  0 bytes/sec
          5 minute output rate 0 pkts/sec,  0 bytes/sec
          5 minute drop rate, 0 pkts/sec
    Ethernet0/4:
    received (in 51434.870 secs):
    0 packets0 bytes
    0 pkts/sec0 bytes/sec
    transmitted (in 51434.870 secs):
    0 packets0 bytes
    0 pkts/sec0 bytes/sec
          1 minute input rate 0 pkts/sec,  0 bytes/sec
          1 minute output rate 0 pkts/sec,  0 bytes/sec
          1 minute drop rate, 0 pkts/sec
          5 minute input rate 0 pkts/sec,  0 bytes/sec
          5 minute output rate 0 pkts/sec,  0 bytes/sec
          5 minute drop rate, 0 pkts/sec
    Ethernet0/5:
    received (in 51434.870 secs):
    0 packets0 bytes
    0 pkts/sec0 bytes/sec
    transmitted (in 51434.870 secs):
    0 packets0 bytes
    0 pkts/sec0 bytes/sec
    <--- More --->
          1 minute input rate 0 pkts/sec,  0 bytes/sec
          1 minute output rate 0 pkts/sec,  0 bytes/sec
          1 minute drop rate, 0 pkts/sec
          5 minute input rate 0 pkts/sec,  0 bytes/sec
          5 minute output rate 0 pkts/sec,  0 bytes/sec
          5 minute drop rate, 0 pkts/sec
    Ethernet0/6:
    received (in 51435.010 secs):
    0 packets0 bytes
    0 pkts/sec0 bytes/sec
    transmitted (in 51435.010 secs):
    0 packets0 bytes
    0 pkts/sec0 bytes/sec
          1 minute input rate 0 pkts/sec,  0 bytes/sec
          1 minute output rate 0 pkts/sec,  0 bytes/sec
          1 minute drop rate, 0 pkts/sec
          5 minute input rate 0 pkts/sec,  0 bytes/sec
          5 minute output rate 0 pkts/sec,  0 bytes/sec
          5 minute drop rate, 0 pkts/sec
    Ethernet0/7:
    received (in 51435.010 secs):
    0 packets0 bytes
    0 pkts/sec0 bytes/sec
    transmitted (in 51435.010 secs):
    <--- More --->
    0 packets0 bytes
    0 pkts/sec0 bytes/sec
          1 minute input rate 0 pkts/sec,  0 bytes/sec
          1 minute output rate 0 pkts/sec,  0 bytes/sec
          1 minute drop rate, 0 pkts/sec
          5 minute input rate 0 pkts/sec,  0 bytes/sec
          5 minute output rate 0 pkts/sec,  0 bytes/sec
          5 minute drop rate, 0 pkts/sec
    Internal-Data0/0:
    received (in 51435.510 secs):
    18513901 packets11784250044 bytes
    25 pkts/sec229023 bytes/sec
    transmitted (in 51435.510 secs):
    18207269 packets11641332179 bytes
    19 pkts/sec226078 bytes/sec
          1 minute input rate 891 pkts/sec,  595715 bytes/sec
          1 minute output rate 863 pkts/sec,  588935 bytes/sec
          1 minute drop rate, 0 pkts/sec
          5 minute input rate 885 pkts/sec,  584035 bytes/sec
          5 minute output rate 870 pkts/sec,  580393 bytes/sec
          5 minute drop rate, 0 pkts/sec
    Internal-Data0/1:
    received (in 51436.010 secs):
    18207323 packets11641364184 bytes
    <--- More --->
    19 pkts/sec226076 bytes/sec
    transmitted (in 51436.010 secs):
    18513954 packets11784281987 bytes
    25 pkts/sec229022 bytes/sec
          1 minute input rate 855 pkts/sec,  575808 bytes/sec
          1 minute output rate 884 pkts/sec,  582339 bytes/sec
          1 minute drop rate, 0 pkts/sec
          5 minute input rate 869 pkts/sec,  578350 bytes/sec
          5 minute output rate 883 pkts/sec,  581924 bytes/sec
          5 minute drop rate, 0 pkts/sec
    ------------------ show perfmon ------------------
    PERFMON STATS:    Current      Average
    Xlates               0/s          0/s
    Connections         17/s          6/s
    TCP Conns            8/s          2/s
    UDP Conns            7/s          2/s
    URL Access           0/s          0/s
    URL Server Req       0/s          0/s
    TCP Fixup            0/s          0/s
    TCP Intercept        0/s          0/s
    HTTP Fixup           0/s          0/s
    <--- More --->
    FTP Fixup            0/s          0/s
    AAA Authen           0/s          0/s
    AAA Author           0/s          0/s
    AAA Account          0/s          0/s
    ------------------ show counters ------------------
    Protocol     Counter                     Value   Context
    IP           IN_PKTS                  168960   Summary
    IP           OUT_PKTS                 169304   Summary
    IP           TO_ARP                       61   Summary
    ------------------ show history ------------------
    ------------------ show firewall ------------------
    Firewall mode: Transparent
    ------------------ show running-config ------------------
    <--- More --->
    : Saved
    ASA Version 7.2(3)
    firewall transparent
    hostname ciscoasa
    enable password
    names
    interface Vlan1
    nameif inside
    security-level 100
    interface Vlan2
    nameif outside
    security-level 0
    interface Ethernet0/0
    switchport access vlan 2
    interface Ethernet0/1
    interface Ethernet0/2
    <--- More --->
    interface Ethernet0/3
    interface Ethernet0/4
    interface Ethernet0/5
    interface Ethernet0/6
    interface Ethernet0/7
    passwd
    regex domain1 ".facebook\.com"
    regex domain2 ".fb\.com"
    regex domain3 ".youtube\.com"
    ftp mode passive
    access-list ACL_IN extended permit ip any any
    pager lines 24
    mtu inside 1500
    mtu outside 1500
    ip address 192.168.1.254 255.255.255.0
    icmp unreachable rate-limit 1 burst-size 1
    asdm image disk0:/asdm-523.bin
    no asdm history enable
    <--- More --->
    arp timeout 14400
    access-group ACL_IN in interface outside
    timeout xlate 3:00:00
    timeout conn 1:00:00 half-closed 0:10:00 udp 0:02:00 icmp 0:00:02
    timeout sunrpc 0:10:00 h323 0:05:00 h225 1:00:00 mgcp 0:05:00 mgcp-pat 0:05:00
    timeout sip 0:30:00 sip_media 0:02:00 sip-invite 0:03:00 sip-disconnect 0:02:00
    timeout uauth 0:05:00 absolute
    no snmp-server location
    no snmp-server contact
    snmp-server enable traps snmp authentication linkup linkdown coldstart
    telnet timeout 5
    ssh timeout 5
    console timeout 0
    class-map type regex match-any DomainBlockList
    match regex domain1
    match regex domain2
    match regex domain3
    class-map inspection_default
    match default-inspection-traffic
    policy-map type inspect dns preset_dns_map
    parameters
    <--- More --->
      message-length maximum 512
    match domain-name regex class DomainBlockList
      drop-connection log
    policy-map global_policy
    class inspection_default
      inspect dns preset_dns_map
      inspect ftp
      inspect h323 h225
      inspect h323 ras
      inspect rsh
      inspect rtsp
      inspect esmtp
      inspect sqlnet
      inspect skinny
      inspect sunrpc
      inspect xdmcp
      inspect sip
      inspect netbios
      inspect tftp
    service-policy global_policy global
    prompt hostname context
    Cryptochecksum:bb5115ea1d14ee42e7961ef0c9aaed86
    : end
    <--- More --->
    ------------------ show startup-config errors ------------------
    INFO: No configuration errors
    ------------------ console logs ------------------
    Message #1 : Message #2 : Message #3 : Message #4 : Message #5 : Message #6 : Message #7 : Message #8 : Message #9 : Message #10 : Message #11 : Message #12 : Message #13 : Message #14 :
    Total SSMs found: 0
    Message #15 :
    Total NICs found: 10
    Message #16 : 88E6095 rev 2 Gigabit Ethernet @ index 09Message #17 :  MAC: 0000.0003.0002
    Message #18 : 88E6095 rev 2 Ethernet @ index 08Message #19 :  MAC: 001f.9ee8.ffa1
    Message #20 : 88E6095 rev 2 Ethernet @ index 07Message #21 :  MAC: 001f.9ee8.ffa0
    Message #22 : 88E6095 rev 2 Ethernet @ index 06Message #23 :  MAC: 001f.9ee8.ff9f
    Message #24 : 88E6095 rev 2 Ethernet @ index 05Message #25 :  MAC: 001f.9ee8.ff9e
    Message #26 : 88E6095 rev 2 Ethernet @ index 04Message #27 :  MAC: 001f.9ee8.ff9d
    Message #28 : 88E6095 rev 2 Ethernet @ index 03Message #29 :  MAC: 001f.9ee8.ff9c
    Message #30 : 88E6095 rev 2 Ethernet @ index 02Message #31 :  MAC: 001f.9ee8.ff9b
    Message #32 : 88E6095 rev 2 Ethernet @ index 01Message #33 :  MAC: 001f.9ee8.ff9a
    Message #34 : y88acs06 rev16 Gigabit Ethernet @ index 00 MAC: 001f.9ee8.ffa2
    Message #35 :
    Licensed features for this platform:
    Message #36 : Maximum Physical Interfaces : 8        
    <--- More --->
    Message #37 : VLANs                       : 3, DMZ Restricted
    Message #38 : Inside Hosts                : Unlimited
    Message #39 : Failover                    : Disabled
    Message #40 : VPN-DES                     : Enabled  
    Message #41 : VPN-3DES-AES                : Enabled  
    Message #42 : VPN Peers                   : 10       
    Message #43 : WebVPN Peers                : 2        
    Message #44 : Dual ISPs                   : Disabled 
    Message #45 : VLAN Trunk Ports            : 0        
    Message #46 :
    This platform has a Base license.
    Message #47 :
    Message #48 : Encryption hardware device : Cisco ASA-5505 on-board accelerator (revision 0x0)
    Message #49 :                              Boot microcode   : CNlite-MC-Boot-Cisco-1.2
    Message #50 :                              SSL/IKE microcode: CNlite-MC-IPSEC-Admin-3.03
    Message #51 :                              IPSec microcode  : CNlite-MC-IPSECm-MAIN-2.04
    Message #52 :   --------------------------------------------------------------------------
    Message #53 :                                  .            .                            
    Message #54 :                                  |            |                            
    Message #55 :                                 |||          |||                           
    Message #56 :                               .|| ||.      .|| ||.                         
    Message #57 :                            .:||| | |||:..:||| | |||:.                      
    Message #58 :                             C i s c o  S y s t e m s                       
    Message #59 :   --------------------------------------------------------------------------
    <--- More --->
    Message #60 :
    Cisco Adaptive Security Appliance Software Version 7.2(3)
    Message #61 :
    Message #62 :   ****************************** Warning *******************************
    Message #63 :   This product contains cryptographic features and is
    Message #64 :   subject to United States and local country laws
    Message #65 :   governing, import, export, transfer, and use.
    Message #66 :   Delivery of Cisco cryptographic products does not
    Message #67 :   imply third-party authority to import, export,
    Message #68 :   distribute, or use encryption. Importers, exporters,
    Message #69 :   distributors and users are responsible for compliance
    Message #70 :   with U.S. and local country laws. By using this
    Message #71 :   product you agree to comply with applicable laws and
    Message #72 :   regulations. If you are unable to comply with U.S.
    Message #73 :   and local laws, return the enclosed items immediately.
    Message #74 :
    Message #75 :   A summary of U.S. laws governing Cisco cryptographic
    Message #76 :   products may be found at:
    Message #77 :   http://www.cisco.com/wwl/export/crypto/tool/stqrg.html
    Message #78 :
    Message #79 :   If you require further assistance please contact us by
    Message #80 :   sending email to [email protected].
    Message #81 :   ******************************* Warning *******************************
    Message #82 :
    <--- More --->
    Message #83 : Copyright (c) 1996-2007 by Cisco Systems, Inc.
    Message #84 :                 Restricted Rights Legend
    Message #85 : Use, duplication, or disclosure by the Government is
    Message #86 : subject to restrictions as set forth in subparagraph
    Message #87 : (c) of the Commercial Computer Software - Restricted
    Message #88 : Rights clause at FAR sec. 52.227-19 and subparagraph
    Message #89 : (c) (1) (ii) of the Rights in Technical Data and Computer
    Message #90 : Software clause at DFARS sec. 252.227-7013.
    Message #91 :                 Cisco Systems, Inc.
    Message #92 :                 170 West Tasman Drive
    Message #93 :                 San Jose, California 95134-1706
    ciscoasa#   

  • Cisco ASA 5505 not able to access flash

    Hi All:
    I have searched and searched all over the net for an answer to this question and have decided to just post it. I have a 5505 that was given to me by my job to use for working on my CCNA Sec. cert and did the following:
    I plugged it in and booted it up just fine. Made config changes as I followed along with the examples in my CCNA Security book. Got to the point in chapter 14 where the initial setup happens to configure it for working with ASDM. I never did a write mem on it and decided to take it back to square one by unplugging it to allow it to lose the changes that I made. This is where things got ugly.
    When it booted back up it got stuck in a bootup loop and couldn't find an IOS. After following all kinds of steps to boot to rommon and tftp another IOS and such (several times) I decided to follow another posting that said that the flash could be corrupted and to just delete it and start anew. Did that and through rommon as it would not boot up normally any more. After trying this over and over for the last couple hours I realized that it would boot from tftp so I did that in hopes of fixing the flash issue.
    I've tried deleting it, and re-initializing it and formating it. But the thing is that it no longer SEES the disk0: mount point. I've used two different flash cards...the one that came with it and the one that I already had. With the cover off I can see that there is no activity light next to the flash drive when I issue a delete or initialize or format command.
    Here is a copy of some of the output file. Any help or suggestions are greatly appreciated.
    CISCO SYSTEMS
    Embedded BIOS Version 1.0(12)11 04/30/08 15:45:41.19
    Low Memory: 632 KB
    High Memory: 507 MB
    PCI Device Table.
    Bus Dev Func VendID DevID Class              Irq
    00  01  00   1022   2080  Host Bridge       
    00  01  02   1022   2082  Chipset En/Decrypt 11
    00  0C  00   1148   4320  Ethernet           11
    00  0D  00   177D   0003  Network En/Decrypt 10
    00  0F  00   1022   2090  ISA Bridge        
    00  0F  02   1022   2092  IDE Controller    
    00  0F  03   1022   2093  Audio              10
    00  0F  04   1022   2094  Serial Bus         9
    00  0F  05   1022   2095  Serial Bus         9
    Evaluating BIOS Options ...
    Launch BIOS Extension to setup ROMMON
    Cisco Systems ROMMON Version (1.0(12)11) #4: Thu May  1 14:50:05 PDT 2008
    Platform ASA5505
    Use BREAK or ESC to interrupt boot.
    Use SPACE to begin boot immediately.
    Ethernet0/0
    MAC Address: 0023.339e.2a91
    Link is UP
    Please set ADDRESS Variable.
    Please set SERVER Variable.
    Please set IMAGE Variable.
    Launching BootLoader...
    Default configuration file contains 1 entry.
    Boot mode is 1. Default entry is 1.
    Searching / for images to boot.
    No images in /
    Error 15: File not found
    unable to boot an image
    Default configuration file contains 1 entry.
    Searching / for images to boot.
    No images in /
    Error 15: File not found
    unable to boot an image
    Failsafe booting engaged.
    Default configuration file contains 1 entry.
    Searching / for images to boot.
    No images in /
    Error 15: File not found
    unable to boot an image
    CISCO SYSTEMS
    Embedded BIOS Version 1.0(12)11 04/30/08 15:45:41.19
    Low Memory: 632 KB
    High Memory: 507 MB
    PCI Device Table.
    Bus Dev Func VendID DevID Class              Irq
    00  01  00   1022   2080  Host Bridge       
    00  01  02   1022   2082  Chipset En/Decrypt 11
    00  0C  00   1148   4320  Ethernet           11
    00  0D  00   177D   0003  Network En/Decrypt 10
    00  0F  00   1022   2090  ISA Bridge        
    00  0F  02   1022   2092  IDE Controller    
    00  0F  03   1022   2093  Audio              10
    00  0F  04   1022   2094  Serial Bus         9
    00  0F  05   1022   2095  Serial Bus         9
    Evaluating BIOS Options ...
    Launch BIOS Extension to setup ROMMON
    Cisco Systems ROMMON Version (1.0(12)11) #4: Thu May  1 14:50:05 PDT 2008
    Platform ASA5505
    Use BREAK or ESC to interrupt boot.
    Use SPACE to begin boot immediately.
    Ethernet0/0
    MAC Address: 0023.339e.2a91
    Link is UP
    Please set ADDRESS Variable.
    Please set SERVER Variable.
    Please set IMAGE Variable.
    Launching BootLoader...
    Default configuration file contains 1 entry.
    Boot mode is 1. Default entry is 1.
    Searching / for images to boot.
    No images in /
    Error 15: File not found
    unable to boot an image
    Default configuration file contains 1 entry.
    Searching / for images to boot.
    No images in /
    Error 15: File not found
    unable to boot an image
    Failsafe booting engaged.
    Default configuration file contains 1 entry.
    Searching / for images to boot.
    No images in /
    Error 15: File not found
    unable to boot an image
    CISCO SYSTEMS
    Embedded BIOS Version 1.0(12)11 04/30/08 15:45:41.19
    Low Memory: 632 KB
    High Memory: 507 MB
    PCI Device Table.
    Bus Dev Func VendID DevID Class              Irq
    00  01  00   1022   2080  Host Bridge       
    00  01  02   1022   2082  Chipset En/Decrypt 11
    00  0C  00   1148   4320  Ethernet           11
    00  0D  00   177D   0003  Network En/Decrypt 10
    00  0F  00   1022   2090  ISA Bridge        
    00  0F  02   1022   2092  IDE Controller    
    00  0F  03   1022   2093  Audio              10
    00  0F  04   1022   2094  Serial Bus         9
    00  0F  05   1022   2095  Serial Bus         9
    Evaluating BIOS Options ...
    Launch BIOS Extension to setup ROMMON
    Cisco Systems ROMMON Version (1.0(12)11) #4: Thu May  1 14:50:05 PDT 2008
    Platform ASA5505
    Use BREAK or ESC to interrupt boot.
    Use SPACE to begin boot immediately.
    Ethernet0/0
    MAC Address: 0023.339e.2a91
    Link is UP
    Please set ADDRESS Variable.
    Please set SERVER Variable.
    Please set IMAGE Variable.
    Launching BootLoader...
    Default configuration file contains 1 entry.
    Boot mode is 1. Default entry is 1.
    Searching / for images to boot.
    No images in /
    Error 15: File not found
    unable to boot an image
    Default configuration file contains 1 entry.
    Searching / for images to boot.
    No images in /
    Error 15: File not found
    unable to boot an image
    Failsafe booting engaged.
    Default configuration file contains 1 entry.
    Searching / for images to boot.
    No images in /
    Error 15: File not found
    unable to boot an image
    CISCO SYSTEMS
    Embedded BIOS Version 1.0(12)11 04/30/08 15:45:41.19
    Low Memory: 632 KB
    High Memory: 507 MB
    PCI Device Table.
    Bus Dev Func VendID DevID Class              Irq
    00  01  00   1022   2080  Host Bridge       
    00  01  02   1022   2082  Chipset En/Decrypt 11
    00  0C  00   1148   4320  Ethernet           11
    00  0D  00   177D   0003  Network En/Decrypt 10
    00  0F  00   1022   2090  ISA Bridge        
    00  0F  02   1022   2092  IDE Controller    
    00  0F  03   1022   2093  Audio              10
    00  0F  04   1022   2094  Serial Bus         9
    00  0F  05   1022   2095  Serial Bus         9
    Evaluating BIOS Options ...
    Launch BIOS Extension to setup ROMMON
    Cisco Systems ROMMON Version (1.0(12)11) #4: Thu May  1 14:50:05 PDT 2008
    Platform ASA5505
    Use BREAK or ESC to interrupt boot.
    Use SPACE to begin boot immediately.
    Ethernet0/0
    MAC Address: 0023.339e.2a91
    Link is UP
    Please set ADDRESS Variable.
    Please set SERVER Variable.
    Please set IMAGE Variable.
    Launching BootLoader...
    Default configuration file contains 1 entry.
    Boot mode is 1. Default entry is 1.
    Searching / for images to boot.
    No images in /
    Error 15: File not found
    unable to boot an image
    Default configuration file contains 1 entry.
    Searching / for images to boot.
    No images in /
    Error 15: File not found
    unable to boot an image
    Failsafe booting engaged.
    Default configuration file contains 1 entry.
    Searching / for images to boot.
    No images in /
    Error 15: File not found
    unable to boot an image
    CISCO SYSTEMS
    Embedded BIOS Version 1.0(12)11 04/30/08 15:45:41.19
    Low Memory: 632 KB
    High Memory: 507 MB
    PCI Device Table.
    Bus Dev Func VendID DevID Class              Irq
    00  01  00   1022   2080  Host Bridge       
    00  01  02   1022   2082  Chipset En/Decrypt 11
    00  0C  00   1148   4320  Ethernet           11
    00  0D  00   177D   0003  Network En/Decrypt 10
    00  0F  00   1022   2090  ISA Bridge        
    00  0F  02   1022   2092  IDE Controller    
    00  0F  03   1022   2093  Audio              10
    00  0F  04   1022   2094  Serial Bus         9
    00  0F  05   1022   2095  Serial Bus         9
    Evaluating BIOS Options ...
    Launch BIOS Extension to setup ROMMON
    Cisco Systems ROMMON Version (1.0(12)11) #4: Thu May  1 14:50:05 PDT 2008
    Platform ASA5505
    Use BREAK or ESC to interrupt boot.
    Use SPACE to begin boot immediately.
    Ethernet0/0
    MAC Address: 0023.339e.2a91
    Link is UP
    Please set ADDRESS Variable.
    Please set SERVER Variable.
    Please set IMAGE Variable.
    Launching BootLoader...
    Default configuration file contains 1 entry.
    Boot mode is 1. Default entry is 1.
    Searching / for images to boot.
    No images in /
    Error 15: File not found
    unable to boot an image
    Default configuration file contains 1 entry.
    Searching / for images to boot.
    No images in /
    Error 15: File not found
    unable to boot an image
    Failsafe booting engaged.
    Default configuration file contains 1 entry.
    Searching / for images to boot.
    No images in /
    Error 15: File not found
    unable to boot an image
    CISCO SYSTEMS
    Embedded BIOS Version 1.0(12)11 04/30/08 15:45:41.19
    Low Memory: 632 KB
    High Memory: 507 MB
    PCI Device Table.
    Bus Dev Func VendID DevID Class              Irq
    00  01  00   1022   2080  Host Bridge       
    00  01  02   1022   2082  Chipset En/Decrypt 11
    00  0C  00   1148   4320  Ethernet           11
    00  0D  00   177D   0003  Network En/Decrypt 10
    00  0F  00   1022   2090  ISA Bridge        
    00  0F  02   1022   2092  IDE Controller    
    00  0F  03   1022   2093  Audio              10
    00  0F  04   1022   2094  Serial Bus         9
    00  0F  05   1022   2095  Serial Bus         9
    Evaluating BIOS Options ...
    Launch BIOS Extension to setup ROMMON
    Cisco Systems ROMMON Version (1.0(12)11) #4: Thu May  1 14:50:05 PDT 2008
    Platform ASA5505
    Use BREAK or ESC to interrupt boot.
    Use SPACE to begin boot immediately.
    Ethernet0/0
    MAC Address: 0023.339e.2a91
    Link is UP
    Please set ADDRESS Variable.
    Please set SERVER Variable.
    Please set IMAGE Variable.
    Launching BootLoader...
    Default configuration file contains 1 entry.
    Boot mode is 1. Default entry is 1.
    Searching / for images to boot.
    No images in /
    Error 15: File not found
    unable to boot an image
    Default configuration file contains 1 entry.
    Searching / for images to boot.
    No images in /
    Error 15: File not found
    unable to boot an image
    Failsafe booting engaged.
    Default configuration file contains 1 entry.
    Searching / for images to boot.
    No images in /
    Error 15: File not found
    unable to boot an image
    CISCO SYSTEMS
    Embedded BIOS Version 1.0(12)11 04/30/08 15:45:41.19
    Low Memory: 632 KB
    High Memory: 507 MB
    PCI Device Table.
    Bus Dev Func VendID DevID Class              Irq
    00  01  00   1022   2080  Host Bridge       
    00  01  02   1022   2082  Chipset En/Decrypt 11
    00  0C  00   1148   4320  Ethernet           11
    00  0D  00   177D   0003  Network En/Decrypt 10
    00  0F  00   1022   2090  ISA Bridge        
    00  0F  02   1022   2092  IDE Controller    
    00  0F  03   1022   2093  Audio              10
    00  0F  04   1022   2094  Serial Bus         9
    00  0F  05   1022   2095  Serial Bus         9
    Evaluating BIOS Options ...
    Launch BIOS Extension to setup ROMMON
    Cisco Systems ROMMON Version (1.0(12)11) #4: Thu May  1 14:50:05 PDT 2008
    Platform ASA5505
    Use BREAK or ESC to interrupt boot.
    Use SPACE to begin boot immediately.
    Boot interrupted.                              
    Ethernet0/0
    MAC Address: 0023.339e.2a91
    Link is UP
    Use ? for help.
    rommon #0> format disk0:
    Invalid or incorrect command.  Use 'help' for help.
    rommon #0> ADDRESS=10.10.10.110
    rommon #1> GATEWAY=10.10.10.1
    rommon #2> SERVER=10.10.10.98
    rommon #3> IMAGE=asa914-k8.bin
    rommon #4> tftp
    ROMMON Variable Settings:
      ADDRESS=10.10.10.110
      SERVER=10.10.10.98
      GATEWAY=10.10.10.1
      PORT=Ethernet0/0
      VLAN=untagged
      IMAGE=asa914-k8.bin
      CONFIG=
      LINKTIMEOUT=20
      PKTTIMEOUT=4
      RETRY=20
    tftp [email protected] via 10.10.10.1
    Received 27076608 bytes
    Launching TFTP Image...
    Cisco Security Appliance admin loader (3.0) #0: Thu Dec  5 19:38:43 PST 2013
    Platform ASA5505
    Loading...
    IO memory blocks requested from bigphys 32bit: 9956
    Àdosfsck 2.11, 12 Mar 2005, FAT32, LFN
    Currently, only 1 or 2 FATs are supported, not 42.
    dosfsck(/dev/hda1) returned 1
    mount: mounting /dev/hda1 on /mnt/disk0 failed: Invalid argument
    mount: mounting /dev/hda1 on /mnt/disk0 failed: Invalid argument
    Processor memory 343932928, Reserved memory: 62914560
    Total SSMs found: 0
    Total NICs found: 10
    88E6095 rev 2 Gigabit Ethernet @ index 09 MAC: 0000.0003.0002
    88E6095 rev 2 Ethernet @ index 08 MAC: 0023.339e.2a90
    88E6095 rev 2 Ethernet @ index 07 MAC: 0023.339e.2a8f
    88E6095 rev 2 Ethernet @ index 06 MAC: 0023.339e.2a8e
    88E6095 rev 2 Ethernet @ index 05 MAC: 0023.339e.2a8d
    88E6095 rev 2 Ethernet @ index 04 MAC: 0023.339e.2a8c
    88E6095 rev 2 Ethernet @ index 03 MAC: 0023.339e.2a8b
    88E6095 rev 2 Ethernet @ index 02 MAC: 0023.339e.2a8a
    88E6095 rev 2 Ethernet @ index 01 MAC: 0023.339e.2a89
    y88acs06 rev16 Gigabit Ethernet @ index 00 MAC: 0023.339e.2a91
    INFO: Unable to read firewall mode from flash
           Writing default firewall mode (single) to flash
    INFO: Unable to read cluster interface-mode from flash
           Writing default mode "None" to flash
    Verify the activation-key, it might take a while...
    Failed to retrieve permanent activation key.
    Running Permanent Activation Key: 0x00000000 0x00000000 0x00000000 0x00000000 0x00000000
    The Running Activation Key is not valid, using default settings:
    Licensed features for this platform:
    Maximum Physical Interfaces       : 8              perpetual
    VLANs                             : 3              DMZ Restricted
    Dual ISPs                         : Disabled       perpetual
    VLAN Trunk Ports                  : 0              perpetual
    Inside Hosts                      : 10             perpetual
    Failover                          : Disabled       perpetual
    Encryption-DES                    : Enabled        perpetual
    Encryption-3DES-AES               : Disabled       perpetual
    AnyConnect Premium Peers          : 2              perpetual
    AnyConnect Essentials             : Disabled       perpetual
    Other VPN Peers                   : 10             perpetual
    Total VPN Peers                   : 12             perpetual
    Shared License                    : Disabled       perpetual
    AnyConnect for Mobile             : Disabled       perpetual
    AnyConnect for Cisco VPN Phone    : Disabled       perpetual
    Advanced Endpoint Assessment      : Disabled       perpetual
    UC Phone Proxy Sessions           : 2              perpetual
    Total UC Proxy Sessions           : 2              perpetual
    Botnet Traffic Filter             : Disabled       perpetual
    Intercompany Media Engine         : Disabled       perpetual
    Cluster                           : Disabled       perpetual
    This platform has a Base license.
    Encryption hardware device : Cisco ASA-5505 on-board accelerator (revision 0x0)
                                 Boot microcode        : CN1000-MC-BOOT-2.00
                                 SSL/IKE microcode     : CNLite-MC-SSLm-PLUS-2_05
                                 IPSec microcode       : CNlite-MC-IPSECm-MAIN-2.09
    Cisco Adaptive Security Appliance Software Version 9.1(4)
      ****************************** Warning *******************************
      This product contains cryptographic features and is
      subject to United States and local country laws
      governing, import, export, transfer, and use.
      Delivery of Cisco cryptographic products does not
      imply third-party authority to import, export,
      distribute, or use encryption. Importers, exporters,
      distributors and users are responsible for compliance
      with U.S. and local country laws. By using this
      product you agree to comply with applicable laws and
      regulations. If you are unable to comply with U.S.
      and local laws, return the enclosed items immediately.
      A summary of U.S. laws governing Cisco cryptographic
      products may be found at:
      http://www.cisco.com/wwl/export/crypto/tool/stqrg.html
      If you require further assistance please contact us by
      sending email to [email protected].
      ******************************* Warning *******************************
    This product includes software developed by the OpenSSL Project
    for use in the OpenSSL Toolkit (http://www.openssl.org/)
    Copyright (C) 1995-1998 Eric Young ([email protected])
    All rights reserved.
    Copyright (c) 1998-2011 The OpenSSL Project.
    All rights reserved.
    This product includes software developed at the University of
    California, Irvine for use in the DAV Explorer project
    (http://www.ics.uci.edu/~webdav/)
    Copyright (c) 1999-2005 Regents of the University of California.
    All rights reserved.
    Busybox, version 1.16.1, Copyright (C) 1989, 1991 Free Software Foundation, Inc.
    51 Franklin St, Fifth Floor, Boston, MA 02110-1301  USA
    Busybox comes with ABSOLUTELY NO WARRANTY.
    This is free software, and you are welcome to redistribute it under the General
    Public License v.2 (http://www.gnu.org/licenses/gpl-2.0.html)
    See User Manual (''Licensing'') for details.
    DOSFSTOOLS, version 2.11, Copyright (C) 1989, 1991 Free Software Foundation, Inc.
    59 Temple Place, Suite 330, Boston, MA 02111-1307
    675 Mass Ave, Cambridge, MA 02139
    DOSFSTOOLS comes with ABSOLUTELY NO WARRANTY.
    This is free software, and you are welcome to redistribute it under the General
    Public License v.2 (http://www.gnu.org/licenses/gpl-2.0.html)
    See User Manual (''Licensing'') for details.
    grub, version 0.94, Copyright (C) 1989, 1991 Free Software Foundation, Inc.
    59 Temple Place, Suite 330, Boston, MA 02111-1307
    grub comes with ABSOLUTELY NO WARRANTY.
    This is free software, and you are welcome to redistribute it under the General
    Public License v.2 (http://www.gnu.org/licenses/gpl-2.0.html)
    See User Manual (''Licensing'') for details.
    libgcc, version 4.3, Copyright (C) 2007 Free Software Foundation, Inc.
    libgcc comes with ABSOLUTELY NO WARRANTY.
    This is free software, and you are welcome to redistribute it under the General
    Public License v.2 (http://www.gnu.org/licenseSee User Manual (''Licensing'') for details.
    libstdc++, version 4.3, Copyright (C) 2007 Free Software Foundation, Inc.
    libstdc++ comes with ABSOLUTELY NO WARRANTY.
    This is free software, and you are welcome to redistribute it under the General
    Public License v.2 (http://www.gnu.org/licenses/gpl-2.0.html)
    See User Manual (''Licensing'') for details.
    Linux kernel, version 2.6.29.6, Copyright (C) 1989, 1991 Free Software
    Foundation, Inc.
    51 Franklin St, Fifth Floor, Boston, MA 02110-1301  USA
    Linux kernel comes with ABSOLUTELY NO WARRANTY.
    This is free software, and you are welcome to redistribute it under the General
    Public License v.2 (http://www.gnu.org/licenses/gpl-2.0.html)
    See User Manual (''Licensing'') for details.
    module-init-tools, version 3.10, Copyright (C) 1989, 1991 Free Software
    Foundation, Inc.
    59 Temple Place, Suite 330, Boston, MA 02111-1307 USA
    module-init-tools comes with ABSOLUTELY NO WARRANTY.
    This is free software, and you are welcome to redistribute it under the General
    Public License v.2 (http://www.gnu.org/licenses/gpl-2.0.html)
    See User Manual (''Licensing'') for details.
    numactl, version 2.0.3, Copyright (C) 2008 SGI.
    Author: Andi Kleen, SUSE Labs
    Version 2.0.0 by Cliff Wickman, Chritopher Lameter and Lee Schermerhorn
    numactl comes with ABSOLUTELY NO WARRANTY.
    This is free software, and you are welcome to redistribute it under the General
    Public License v.2 (http://www.gnu.org/licenses/gpl-2.0.html)
    See User Manual (''Licensing'') for details.
    pciutils, version 3.1.4, Copyright (C) 1989, 1991 Free Software Foundation, Inc.
    51 Franklin St, Fifth Floor, Boston, MA 02110-1301  USA
    pciutils comes with ABSOLUTELY NO WARRANTY.
    This is free software, and you are welcome to redistribute it under the General
    Public License v.2 (http://www.gnu.org/licenses/gpl-2.0.html)
    See User Manual (''Licensing'') for details.
    readline, version 5.2, Copyright (C) 1989, 1991 Free Software Foundation, Inc.
    59 Temple Place, Suite 330, Boston, MA 02111 USA
    readline comes with ABSOLUTELY NO WARRANTY.
    This is free software, and you are welcome to redistribute it under the General
    Public License v.2 (http://www.gnu.org/licenses/gpl-2.0.html)
    See User Manual (''Licensing'') for details.
    udev, version 146, Copyright (C) 1989, 1991 Free Software Foundation, Inc.
    51 Franklin St, Fifth Floor, Boston, MA 02110-1301  USA
    udev comes with ABSOLUTELY NO WARRANTY.
    This is free software, and you are welcome to redistribute it under the General
    Public License v.2 (http://www.gnu.org/licenses/gpl-2.0.html)
    See User Manual (''Licensing'') for details.
    Cisco Adapative Security Appliance Software, version 9.1,
    Copyright (c) 1996-2013 by Cisco Systems, Inc.
    Certain components of Cisco ASA Software, Version 9.1 are licensed under the GNU
    Lesser Public License (LGPL) Version 2.1.  The software code licensed under LGPL
    Version 2.1 is free software that comes with ABSOLUTELY NO WARRANTY.  You can
    redistribute and/or modify such LGPL code under the terms of LGPL Version 2.1
    (http://www.gnu.org/licenses/lgpl-2.1.html).  See User Manual for licensing
    details.
                    Restricted Rights Legend
    Use, duplication, or disclosure by the Government is
    subject to restrictions as set forth in subparagraph
    (c) of the Commercial Computer Software - Restricted
    Rights clause at FAR sec. 52.227-19 and subparagraph
    (c) (1) (ii) of the Rights in Technical Data and Computer
    Software clause at DFARS sec. 252.227-7013.
                    Cisco Systems, Inc.
                    170 West Tasman Drive
                    San Jose, California 95134-1706
    Insufficient flash space available for this request:
      Size info: request:32 free:0  delta:32
    Could not initialize system files in flash.
    config_fetcher: channel open failed
    ERROR: MIGRATION - Could not get the startup configuration.
    INFO: Power-On Self-Test in process.
    INFO: Power-On Self-Test complete.
    INFO: MIGRATION - Saving the startup errors to file 'flash:upgrade_startup_errors_200804300128.log'
    Pre-configure Firewall now through interactive prompts [yes]? n
    Type help or '?' for a list of available commands.
    ciscoasa> en
    Password:
    ciscoasa# format disk0:
    Format operation may take a while. Continue? [confirm]
    Format operation will destroy all data in "disk0:".  Continue? [confirm]
    Initializing partition - done!
    Creating FAT16 filesystem
    mkdosfs 2.11 (12 Mar 2005)
    System tables written to disk
    Format of disk0 complete
    ciscoasa# format disk:
                     ^
    ERROR: % Invalid input detected at '^' marker.
    ciscoasa# format flash:
    Format operation may take a while. Continue? [confirm]
    Format operation will destroy all data in "flash:".  Continue? [confirm]
    Initializing partition - done!

    Yeah...I think I found that one out the hard way already. I'll cross that bridge when I get to it. I want to get this issue fixed before I start thinking about the license issue.
    ciscoasa#
    ciscoasa#
    ciscoasa#
    ciscoasa# sh flash
    --#--  --length--  -----date/time------  path
    2403  0           Apr 30 2008 02:00:56  test
    2285  196         Apr 30 2008 01:28:20  upgrade_startup_errors_200804300128.log
    2283  0           Apr 30 2008 01:28:20  coredumpinfo
    2284  59          Apr 30 2008 01:28:20  coredumpinfo/coredump.cfg
    2280  0           Apr 30 2008 01:27:56  crypto_archive
    2267  0           Apr 30 2008 01:27:38  log
    0 bytes total (0 bytes free)
    ciscoasa#
    ciscoasa#
    ciscoasa#
    ciscoasa# sh disk0
    --#--  --length--  -----date/time------  path
    2403  0           Apr 30 2008 02:00:56  test
    2285  196         Apr 30 2008 01:28:20  upgrade_startup_errors_200804300128.log
    2283  0           Apr 30 2008 01:28:20  coredumpinfo
    2284  59          Apr 30 2008 01:28:20  coredumpinfo/coredump.cfg
    2280  0           Apr 30 2008 01:27:56  crypto_archive
    2267  0           Apr 30 2008 01:27:38  log
    0 bytes total (0 bytes free)
    ciscoasa#

  • Cisco ASA 5505 doesn't forware incoming connection to LAN

    Hello everybody.
    I just got a Cisco asa 5505 with the next OS and ASDM info
    ASA 5505 OS 8.4(3) ASDM 6.47
    I configured and enter all rules to allow incoming traffic to LAN but it's not working also, I have one host inside that is configured in a second IP and create the rule to allow traffic to it but it doesn't work too.
    Problem 1
    I have VNC running in port 5900 tcp and I want to connect from Internet using port 6001 and this has to forware the connection to the real VNC port. In the configuration I have a few host with the same configuration but I use different outside port to get it.
    Problem 2.
    I have a second IP with services: SMTP, HTTP, HTTPS and port 444 all TCP forwarding to a server in the LAN.
    Facts:
    SMTP.
    Every time that I do telnet to the second IP looking for the SMTP port, the firewall doesn't let the incoming connection goes through and the LOGGING screen doesn't how that connection.
    PORT 6001 (outside)
    this port is configured to work with the IP in the outside internface and it was to send the incoming connection to a host inside to the real port 5900.
    Can any one check my configuration if I'm missing anything? for sure I'm but I didn't find it. Bellow is the configuration, I masked the Public IPs just left the last number in the IP, also I left the LAN network to see better the configuration.
    I will appreciate any help.
    Thanks a lot..
    CONFIGURATION.
    : Saved
    ASA Version 8.4(3)
    hostname saturn1
    domain-name mydominio.com
    enable password SOMEPASS encrypted
    passwd SOMEPASS encrypted
    names
    name 192.168.250.11 CAPITOLA-LAN
    name 192.168.250.15 OBIi110-LAN
    name 192.168.250.21 DRP1260-LAN
    name 192.168.250.22 HPOJ8500-LAN
    name 192.168.250.30 AP-W77-NG-LAN
    name 192.168.250.97 AJ-DTOP-PC-LAN
    name 192.168.250.96 SWEETHEART-PC-LAN
    name 192.168.250.94 KIDS-PC-LAN
    name XX.YY.ZZ.250 EXTERNALIP
    name XX.YY.ZZ.251 EXTERNALIP2
    name XX.YY.ZZ.1 GTWAY
    dns-guard
    interface Ethernet0/0
    switchport access vlan 2
    interface Ethernet0/1
    interface Ethernet0/2
    interface Ethernet0/3
    interface Ethernet0/4
    interface Ethernet0/5
    interface Ethernet0/6
    interface Ethernet0/7
    interface Vlan1
    nameif inside
    security-level 100
    ip address 192.168.250.2 255.255.255.0
    interface Vlan2
    nameif outside
    security-level 0
    ip address EXTERNALIP 255.255.255.0
    boot system disk0:/asa843-k8.bin
    ftp mode passive
    dns server-group DefaultDNS
    domain-name mydominio.com
    object network CAPITOLA-LAN
    host 192.168.250.11
    object network EXTERNALIP
    host XX.YY.ZZ.250
    description Created during name migration
    object network CAPITOLA-PUBLIC
    host XX.YY.ZZ.251
    object network capitola-int
    host 192.168.250.11
    object network capitola-int-vnc
    host 192.168.250.11
    object network aj-dtop-int-vnc
    host 192.168.250.97
    object network sweetheart-int-vnc
    host 192.168.250.96
    object network kids-int-vnc
    host 192.168.250.94
    object network VPNNetwork
    subnet 10.10.20.0 255.255.255.0
    object network InsideNetwork
    subnet 192.168.250.0 255.255.255.0
    object network obj_any
    subnet 0.0.0.0 0.0.0.0
    object network capitola-int-smtp
    host 192.168.250.11
    object-group service capitola-int-smtp-service tcp
    port-object eq smtp
    object-group service capitola-int-services tcp
    port-object eq smtp
    port-object eq https
    port-object eq www
    port-object eq 444
    object-group service capitola-int-vnc-service tcp
    port-object eq 6001
    object-group service aj-dtop-int-vnc-service tcp
    port-object eq 6002
    object-group service sweetheart-int-vnc-service tcp
    port-object eq 6003
    object-group service kids-int-vnc-service tcp
    port-object eq 6004
    access-list incoming extended permit icmp any any
    access-list incoming extended permit tcp any object capitola-int object-group capitola-int-services
    access-list incoming extended permit tcp any object capitola-int-vnc object-group capitola-int-vnc-service
    access-list incoming extended permit tcp any object aj-dtop-int-vnc object-group aj-dtop-int-vnc-service
    access-list incoming extended permit tcp any object sweetheart-int-vnc object-group sweetheart-int-vnc-service
    access-list incoming extended permit tcp any object kids-int-vnc object-group kids-int-vnc-service
    access-list incoming extended permit tcp any object capitola-int-smtp object-group capitola-int-smtp-service
    access-list split-tunnel standard permit 192.168.250.0 255.255.255.0
    access-list inside_nat0_outbound extended permit ip any object VPNNetwork
    pager lines 24
    logging enable
    logging asdm informational
    mtu inside 1500
    mtu outside 1500
    ip local pool vpnpool 10.10.20.1-10.10.20.50 mask 255.255.255.0
    icmp unreachable rate-limit 1 burst-size 1
    asdm image disk0:/asdm-647.bin
    no asdm history enable
    arp timeout 14400
    nat (inside,any) source static any any destination static VPNNetwork VPNNetwork no-proxy-arp
    object network capitola-int
    nat (any,any) static XX.YY.ZZ.251
    object network capitola-int-vnc
    nat (inside,outside) static interface service tcp 5900 6001
    object network aj-dtop-int-vnc
    nat (inside,outside) static interface service tcp 5900 6002
    object network sweetheart-int-vnc
    nat (inside,outside) static interface service tcp 5900 6003
    object network kids-int-vnc
    nat (inside,outside) static interface service tcp 5900 6004
    object network obj_any
    nat (inside,outside) dynamic interface
    object network capitola-int-smtp
    nat (any,outside) static interface service tcp smtp smtp
    access-group incoming in interface outside
    route outside 0.0.0.0 0.0.0.0 GTWAY 1
    timeout xlate 3:00:00
    timeout pat-xlate 0:00:30
    timeout conn 1:00:00 half-closed 0:10:00 udp 0:02:00 icmp 0:00:02
    timeout sunrpc 0:10:00 h323 0:05:00 h225 1:00:00 mgcp 0:05:00 mgcp-pat 0:05:00
    timeout sip 0:30:00 sip_media 0:02:00 sip-invite 0:03:00 sip-disconnect 0:02:00
    timeout sip-provisional-media 0:02:00 uauth 0:05:00 absolute
    timeout tcp-proxy-reassembly 0:01:00
    timeout floating-conn 0:00:00
    dynamic-access-policy-record DfltAccessPolicy
    user-identity default-domain LOCAL
    aaa authentication ssh console LOCAL
    http server enable
    http server idle-timeout 2
    http server session-timeout 1
    http 192.168.1.0 255.255.255.0 inside
    http CAPITOLA-LAN 255.255.255.255 inside
    http AJ-DTOP-PC-LAN 255.255.255.255 inside
    no snmp-server location
    no snmp-server contact
    snmp-server enable traps snmp authentication linkup linkdown coldstart
    telnet timeout 5
    ssh CAPITOLA-LAN 255.255.255.255 inside
    ssh AJ-DTOP-PC-LAN 255.255.255.255 inside
    ssh timeout 15
    console timeout 0
    vpn-addr-assign local reuse-delay 2
    dhcpd auto_config outside
    threat-detection basic-threat
    threat-detection statistics access-list
    no threat-detection statistics tcp-intercept
    webvpn
    username admin password SOMEPASS encrypted privilege 15
    class-map inspection_default
    match default-inspection-traffic
    policy-map type inspect dns preset_dns_map
    parameters
      message-length maximum 512
    policy-map global_policy
    class inspection_default
      inspect dns preset_dns_map
      inspect ftp
      inspect h323 h225
      inspect h323 ras
      inspect rsh
      inspect rtsp
      inspect sqlnet
      inspect skinny 
      inspect sunrpc
      inspect xdmcp
      inspect sip 
      inspect netbios
      inspect tftp
      inspect pptp
      inspect ip-options
    service-policy global_policy global
    prompt hostname context
    no call-home reporting anonymous
    call-home
    profile CiscoTAC-1
      no active
      destination address http https://tools.cisco.com/its/service/oddce/services/DDCEService
      destination address email [email protected]
      destination transport-method http
      subscribe-to-alert-group diagnostic
      subscribe-to-alert-group environment
      subscribe-to-alert-group inventory periodic monthly
      subscribe-to-alert-group configuration periodic monthly
      subscribe-to-alert-group telemetry periodic daily
    Cryptochecksum:036b82d3eb5cffc1c65a3b381246d043
    : end
    asdm image disk0:/asdm-647.bin
    no asdm history enable

    Jose, your fix to problem 1 allows all access from the outside, assuming you applied the extended list to the outside interface.  Try to be more restrictive than an '...ip any any' rule for outside_in connections.  For instance, this is what I have for incoming VOIP (access list and nat rules):
    access list rule:
    access-list outside_access_in extended permit udp any object server range 9000 9049 log errors
    nat rule:
    nat (inside,outside) source static server interface service voip-range voip-range
    - 'server' is a network object *
    - 'voip-range' is a service group range
    I'd assume you can do something similar here in combination with my earlier comment:
    access-list incoming extended permit tcp any any eq 5900
    Can you explain your forwarding methodology a little more?  I'm by no means an expert on forwarding, but the way I read what you're trying to do is that you have an inbound VNC request coming in on 5900 and you want the firewall to figure out which host the request should go to.  Or is it vice-versa, the inbound VNC request can be on port 6001-6004 ?

  • Remote Access VPN Problem with ASA 5505

    After about ~1 year of having the Cisco VPN Client connecting to a ASA 5505 without any problems, suddenly one day it stops working. The client is able to get a connection to the ASA and browse the local network for only about 30 seconds after connection. After that, no access is available to the network behind the ASA. I tried everything that I can think of to try and troubleshoot the problem, but at this point I am just banging my head against a wall. Does anyone know what could cause this?
    Here is the running cfg of the ASA
    : Saved
    ASA Version 8.4(1)
    hostname NCHCO
    enable password xxxxxxxxxxxxxxx encrypted
    passwd xxxxxxxxxxx encrypted
    names
    name 192.168.2.0 NCHCO description City Offices
    name 192.168.2.80 VPN_End
    name 192.168.2.70 VPN_Start
    interface Vlan1
    nameif inside
    security-level 100
    ip address 192.168.2.1 255.255.255.0
    interface Vlan2
    nameif outside
    security-level 0
    ip address **.**.***.*** 255.255.255.248
    interface Ethernet0/0
    switchport access vlan 2
    speed 100
    duplex full
    interface Ethernet0/1
    interface Ethernet0/2
    interface Ethernet0/3
    interface Ethernet0/4
    interface Ethernet0/5
    interface Ethernet0/6
    interface Ethernet0/7
    boot system disk0:/asa841-k8.bin
    ftp mode passive
    object network NCHCO
    subnet 192.168.2.0 255.255.255.0
    object network obj-192.168.1.0
    subnet 192.168.1.0 255.255.255.0
    object network obj-192.168.2.64
    subnet 192.168.2.64 255.255.255.224
    object network obj-0.0.0.0
    subnet 0.0.0.0 255.255.255.0
    object network obj_any
    subnet 0.0.0.0 0.0.0.0
    object network Webserver
    object network FINX
    host 192.168.2.11
    object service rdp
    service tcp source range 1 65535 destination eq 3389
    description rdp
    access-list outside_nat0_outbound extended permit ip object NCHCO 192.168.1.0 255.255.255.0
    access-list outside_nat0_outbound extended permit ip object NCHCO 192.168.2.0 255.255.255.0
    access-list inside_nat0_outbound extended permit ip object NCHCO 192.168.1.0 255.255.255.0
    access-list inside_nat0_outbound extended permit ip any 192.168.2.64 255.255.255.224
    access-list inside_nat0_outbound extended permit ip 0.0.0.0 255.255.255.0 192.168.2.64 255.255.255.224
    access-list outside_1_cryptomap extended permit ip object NCHCO 192.168.1.0 255.255.255.0
    access-list outside_1_cryptomap_1 extended permit ip object NCHCO 192.168.1.0 255.255.255.0
    access-list LAN_Access standard permit 192.168.2.0 255.255.255.0
    access-list LAN_Access standard permit 0.0.0.0 255.255.255.0
    access-list NCHCO_splitTunnelAcl_1 standard permit 192.168.2.0 255.255.255.0
    access-list AnyConnect_Client_Local_Print extended deny ip any any
    access-list AnyConnect_Client_Local_Print extended permit tcp any any eq lpd
    access-list AnyConnect_Client_Local_Print remark IPP: Internet Printing Protocol
    access-list AnyConnect_Client_Local_Print extended permit tcp any any eq 631
    access-list AnyConnect_Client_Local_Print remark Windows' printing port
    access-list AnyConnect_Client_Local_Print extended permit tcp any any eq 9100
    access-list AnyConnect_Client_Local_Print remark mDNS: multicast DNS protocol
    access-list AnyConnect_Client_Local_Print extended permit udp any host 224.0.0.251 eq 5353
    access-list AnyConnect_Client_Local_Print remark LLMNR: Link Local Multicast Name Resolution protocol
    access-list AnyConnect_Client_Local_Print extended permit udp any host 224.0.0.252 eq 5355
    access-list AnyConnect_Client_Local_Print remark TCP/NetBIOS protocol
    access-list AnyConnect_Client_Local_Print extended permit tcp any any eq 137
    access-list AnyConnect_Client_Local_Print extended permit udp any any eq netbios-ns
    access-list outside_access_in extended permit tcp any object FINX eq 3389
    access-list outside_access_in_1 extended permit object rdp any object FINX
    pager lines 24
    logging enable
    logging asdm informational
    mtu inside 1500
    mtu outside 1500
    ip local pool VPN_Pool VPN_Start-VPN_End mask 255.255.255.0
    icmp unreachable rate-limit 1 burst-size 1
    asdm image disk0:/asdm-649.bin
    no asdm history enable
    arp timeout 14400
    nat (inside,any) source static NCHCO NCHCO destination static obj-192.168.1.0 obj-192.168.1.0
    nat (inside,any) source static any any destination static obj-192.168.2.64 obj-192.168.2.64
    nat (inside,any) source static obj-0.0.0.0 obj-0.0.0.0 destination static obj-192.168.2.64 obj-192.168.2.64
    object network obj_any
    nat (inside,outside) dynamic interface
    object network FINX
    nat (inside,outside) static interface service tcp 3389 3389
    access-group outside_access_in_1 in interface outside
    route outside 0.0.0.0 0.0.0.0 69.61.228.177 1
    timeout xlate 3:00:00
    timeout conn 1:00:00 half-closed 0:10:00 udp 0:02:00 icmp 0:00:02
    timeout sunrpc 0:10:00 h323 0:05:00 h225 1:00:00 mgcp 0:05:00 mgcp-pat 0:05:00
    timeout sip 0:30:00 sip_media 0:02:00 sip-invite 0:03:00 sip-disconnect 0:02:00
    timeout sip-provisional-media 0:02:00 uauth 0:05:00 absolute
    timeout tcp-proxy-reassembly 0:01:00
    dynamic-access-policy-record DfltAccessPolicy
    network-acl outside_nat0_outbound
    webvpn
      svc ask enable default svc
    http server enable
    http 192.168.1.0 255.255.255.0 inside
    http **.**.***.*** 255.255.255.255 outside
    http **.**.***.*** 255.255.255.255 outside
    http NCHCO 255.255.255.0 inside
    http 96.11.251.186 255.255.255.255 outside
    no snmp-server location
    no snmp-server contact
    snmp-server enable traps snmp authentication linkup linkdown coldstart warmstart
    crypto ipsec ikev1 transform-set ESP-AES-128-SHA esp-aes esp-sha-hmac
    crypto ipsec ikev1 transform-set ESP-AES-128-MD5 esp-aes esp-md5-hmac
    crypto ipsec ikev1 transform-set ESP-AES-192-SHA esp-aes-192 esp-sha-hmac
    crypto ipsec ikev1 transform-set ESP-AES-192-MD5 esp-aes-192 esp-md5-hmac
    crypto ipsec ikev1 transform-set ESP-AES-256-SHA esp-aes-256 esp-sha-hmac
    crypto ipsec ikev1 transform-set ESP-AES-256-MD5 esp-aes-256 esp-md5-hmac
    crypto ipsec ikev1 transform-set ESP-3DES-MD5 esp-3des esp-md5-hmac
    crypto ipsec ikev1 transform-set ESP-DES-SHA esp-des esp-sha-hmac
    crypto ipsec ikev1 transform-set ESP-DES-MD5 esp-des esp-md5-hmac
    crypto ipsec ikev1 transform-set l2tp-transform esp-3des esp-sha-hmac
    crypto ipsec ikev1 transform-set l2tp-transform mode transport
    crypto ipsec ikev1 transform-set vpn-transform esp-aes-256 esp-sha-hmac
    crypto ipsec ikev1 transform-set TRANS_ESP_3DES_SHA esp-3des esp-sha-hmac
    crypto ipsec ikev1 transform-set TRANS_ESP_3DES_SHA mode transport
    crypto ipsec ikev1 transform-set TRANS_ESP_3DES_MD5 esp-3des esp-md5-hmac
    crypto ipsec ikev1 transform-set TRANS_ESP_3DES_MD5 mode transport
    crypto ipsec ikev1 transform-set ESP-3DES-SHA esp-3des esp-sha-hmac
    crypto dynamic-map SYSTEM_DEFAULT_CRYPTO_MAP 65535 set pfs group1
    crypto dynamic-map SYSTEM_DEFAULT_CRYPTO_MAP 65535 set ikev1 transform-set ESP-AES-128-SHA ESP-AES-128-MD5 ESP-AES-192-SHA ESP-AES-192-MD5 ESP-AES-256-SHA ESP-AES-256-MD5 ESP-3DES-SHA ESP-3DES-MD5 ESP-DES-SHA ESP-DES-MD5
    crypto dynamic-map dyn-map 10 set pfs group1
    crypto dynamic-map dyn-map 10 set ikev1 transform-set l2tp-transform vpn-transform
    crypto dynamic-map dyn-map 10 set reverse-route
    crypto dynamic-map outside_dyn_map 20 set ikev1 transform-set ESP-3DES-SHA
    crypto dynamic-map outside_dyn_map 20 set reverse-route
    crypto map outside_map 1 match address outside_1_cryptomap
    crypto map outside_map 1 set pfs group1
    crypto map outside_map 1 set peer 74.219.208.50
    crypto map outside_map 1 set ikev1 transform-set ESP-3DES-SHA
    crypto map outside_map 20 ipsec-isakmp dynamic outside_dyn_map
    crypto map outside_map 65535 ipsec-isakmp dynamic SYSTEM_DEFAULT_CRYPTO_MAP
    crypto map outside_map interface outside
    crypto map inside_map 65535 ipsec-isakmp dynamic SYSTEM_DEFAULT_CRYPTO_MAP
    crypto map inside_map interface inside
    crypto map vpn-map 1 match address outside_1_cryptomap_1
    crypto map vpn-map 1 set pfs group1
    crypto map vpn-map 1 set peer 74.219.208.50
    crypto map vpn-map 1 set ikev1 transform-set ESP-3DES-SHA
    crypto map vpn-map 10 ipsec-isakmp dynamic dyn-map
    crypto isakmp identity address
    crypto ikev1 enable inside
    crypto ikev1 enable outside
    crypto ikev1 ipsec-over-tcp port 10000
    crypto ikev1 policy 10
    authentication pre-share
    encryption 3des
    hash md5
    group 2
    lifetime 86400
    crypto ikev1 policy 15
    authentication pre-share
    encryption aes-256
    hash sha
    group 2
    lifetime 86400
    crypto ikev1 policy 35
    authentication pre-share
    encryption 3des
    hash sha
    group 2
    lifetime 86400
    client-update enable
    telnet 192.168.1.0 255.255.255.0 inside
    telnet NCHCO 255.255.255.0 inside
    telnet timeout 5
    ssh 192.168.1.0 255.255.255.0 inside
    ssh NCHCO 255.255.255.0 inside
    ssh timeout 5
    console timeout 0
    dhcpd address 192.168.2.150-192.168.2.225 inside
    dhcpd dns 216.68.4.10 216.68.5.10 interface inside
    dhcpd lease 64000 interface inside
    threat-detection basic-threat
    threat-detection statistics access-list
    no threat-detection statistics tcp-intercept
    webvpn
    group-policy DefaultRAGroup internal
    group-policy DefaultRAGroup attributes
    dns-server value 192.168.2.1
    vpn-tunnel-protocol ikev1 l2tp-ipsec
    default-domain value nchco.local
    group-policy DfltGrpPolicy attributes
    dns-server value 192.168.2.1
    vpn-tunnel-protocol ikev1 l2tp-ipsec ssl-client ssl-clientless
    password-storage enable
    ipsec-udp enable
    intercept-dhcp 255.255.255.0 enable
    address-pools value VPN_Pool
    group-policy NCHCO internal
    group-policy NCHCO attributes
    dns-server value 192.168.2.1 8.8.8.8
    vpn-tunnel-protocol ikev1
    split-tunnel-policy tunnelspecified
    split-tunnel-network-list value NCHCO_splitTunnelAcl_1
    default-domain value NCHCO.local
    username admin password LbMiJuAJjDaFb2uw encrypted privilege 15
    username 8njferg password yB1lHEVmHZGj5C2Z encrypted privilege 15
    username NCHvpn99 password dhn.JzttvRmMbHsP encrypted
    tunnel-group DefaultRAGroup general-attributes
    address-pool (inside) VPN_Pool
    address-pool VPN_Pool
    authentication-server-group (inside) LOCAL
    authentication-server-group (outside) LOCAL
    authorization-server-group LOCAL
    authorization-server-group (inside) LOCAL
    authorization-server-group (outside) LOCAL
    default-group-policy DefaultRAGroup
    strip-realm
    strip-group
    tunnel-group DefaultRAGroup ipsec-attributes
    ikev1 pre-shared-key *****
    peer-id-validate nocheck
    tunnel-group DefaultRAGroup ppp-attributes
    no authentication chap
    no authentication ms-chap-v1
    authentication ms-chap-v2
    tunnel-group DefaultWEBVPNGroup ppp-attributes
    authentication pap
    authentication ms-chap-v2
    tunnel-group 74.219.208.50 type ipsec-l2l
    tunnel-group 74.219.208.50 ipsec-attributes
    ikev1 pre-shared-key *****
    tunnel-group NCHCO type remote-access
    tunnel-group NCHCO general-attributes
    address-pool VPN_Pool
    default-group-policy NCHCO
    tunnel-group NCHCO ipsec-attributes
    ikev1 pre-shared-key *****
    class-map inspection_default
    match default-inspection-traffic
    policy-map type inspect dns preset_dns_map
    parameters
      message-length maximum client auto
      message-length maximum 512
    policy-map global_policy
    class inspection_default
      inspect dns preset_dns_map
      inspect ftp
      inspect h323 h225
      inspect h323 ras
      inspect rsh
      inspect rtsp
      inspect esmtp
      inspect sqlnet
      inspect skinny
      inspect sunrpc
      inspect xdmcp
      inspect sip
      inspect netbios
      inspect tftp
      inspect ip-options
    service-policy global_policy global
    prompt hostname context
    call-home
    profile CiscoTAC-1
      no active
      destination address http https://tools.cisco.com/its/service/oddce/services/DDCEService
      destination address email [email protected]
      destination transport-method http
      subscribe-to-alert-group diagnostic
      subscribe-to-alert-group environment
      subscribe-to-alert-group inventory periodic monthly
      subscribe-to-alert-group configuration periodic monthly
      subscribe-to-alert-group telemetry periodic daily
    Cryptochecksum:a2110206e1af06974c858fb40c6de2fc
    : end
    asdm image disk0:/asdm-649.bin
    asdm location VPN_Start 255.255.255.255 inside
    asdm location VPN_End 255.255.255.255 inside
    no asdm history enable
    And here is the logs from the Cisco VPN Client when it browses, then fails to browse the network behind the ASA:
    Cisco Systems VPN Client Version 5.0.07.0440
    Copyright (C) 1998-2010 Cisco Systems, Inc. All Rights Reserved.
    Client Type(s): Windows, WinNT
    Running on: 6.1.7601 Service Pack 1
    Config file directory: C:\Program Files (x86)\Cisco Systems\VPN Client\
    1      09:44:55.677  10/01/13  Sev=Info/6    CERT/0x63600026
    Attempting to find a Certificate using Serial Hash.
    2      09:44:55.677  10/01/13  Sev=Info/6    CERT/0x63600027
    Found a Certificate using Serial Hash.
    3      09:44:55.693  10/01/13  Sev=Info/6    GUI/0x63B00011
    Reloaded the Certificates in all Certificate Stores successfully.
    4      09:45:02.802  10/01/13  Sev=Info/4    CM/0x63100002
    Begin connection process
    5      09:45:02.802  10/01/13  Sev=Info/4    CM/0x63100004
    Establish secure connection
    6      09:45:02.802  10/01/13  Sev=Info/4    CM/0x63100024
    Attempt connection with server "**.**.***.***"
    7      09:45:02.802  10/01/13  Sev=Info/6    IKE/0x6300003B
    Attempting to establish a connection with **.**.***.***.
    8      09:45:02.818  10/01/13  Sev=Info/4    IKE/0x63000001
    Starting IKE Phase 1 Negotiation
    9      09:45:02.865  10/01/13  Sev=Info/4    IKE/0x63000013
    SENDING >>> ISAKMP OAK AG (SA, KE, NON, ID, VID(Xauth), VID(dpd), VID(Frag), VID(Nat-T), VID(Unity)) to **.**.***.***
    10     09:45:02.896  10/01/13  Sev=Info/5    IKE/0x6300002F
    Received ISAKMP packet: peer = **.**.***.***
    11     09:45:02.896  10/01/13  Sev=Info/4    IKE/0x63000014
    RECEIVING <<< ISAKMP OAK AG (SA, KE, NON, ID, HASH, VID(Unity), VID(Xauth), VID(dpd), VID(Nat-T), NAT-D, NAT-D, VID(Frag), VID(?)) from **.**.***.***
    12     09:45:02.896  10/01/13  Sev=Info/5    IKE/0x63000001
    Peer is a Cisco-Unity compliant peer
    13     09:45:02.896  10/01/13  Sev=Info/5    IKE/0x63000001
    Peer supports XAUTH
    14     09:45:02.896  10/01/13  Sev=Info/5    IKE/0x63000001
    Peer supports DPD
    15     09:45:02.896  10/01/13  Sev=Info/5    IKE/0x63000001
    Peer supports NAT-T
    16     09:45:02.896  10/01/13  Sev=Info/5    IKE/0x63000001
    Peer supports IKE fragmentation payloads
    17     09:45:02.927  10/01/13  Sev=Info/6    IKE/0x63000001
    IOS Vendor ID Contruction successful
    18     09:45:02.927  10/01/13  Sev=Info/4    IKE/0x63000013
    SENDING >>> ISAKMP OAK AG *(HASH, NOTIFY:STATUS_INITIAL_CONTACT, NAT-D, NAT-D, VID(?), VID(Unity)) to **.**.***.***
    19     09:45:02.927  10/01/13  Sev=Info/4    IKE/0x63000083
    IKE Port in use - Local Port =  0xDD3B, Remote Port = 0x01F4
    20     09:45:02.927  10/01/13  Sev=Info/5    IKE/0x63000072
    Automatic NAT Detection Status:
       Remote end is NOT behind a NAT device
       This   end is NOT behind a NAT device
    21     09:45:02.927  10/01/13  Sev=Info/4    CM/0x6310000E
    Established Phase 1 SA.  1 Crypto Active IKE SA, 0 User Authenticated IKE SA in the system
    22     09:45:02.943  10/01/13  Sev=Info/5    IKE/0x6300002F
    Received ISAKMP packet: peer = **.**.***.***
    23     09:45:02.943  10/01/13  Sev=Info/4    IKE/0x63000014
    RECEIVING <<< ISAKMP OAK TRANS *(HASH, ATTR) from **.**.***.***
    24     09:45:02.943  10/01/13  Sev=Info/4    CM/0x63100015
    Launch xAuth application
    25     09:45:03.037  10/01/13  Sev=Info/6    GUI/0x63B00012
    Authentication request attributes is 6h.
    26     09:45:03.037  10/01/13  Sev=Info/4    CM/0x63100017
    xAuth application returned
    27     09:45:03.037  10/01/13  Sev=Info/4    IKE/0x63000013
    SENDING >>> ISAKMP OAK TRANS *(HASH, ATTR) to **.**.***.***
    28     09:45:03.037  10/01/13  Sev=Info/4    IPSEC/0x63700008
    IPSec driver successfully started
    29     09:45:03.037  10/01/13  Sev=Info/4    IPSEC/0x63700014
    Deleted all keys
    30     09:45:03.083  10/01/13  Sev=Info/5    IKE/0x6300002F
    Received ISAKMP packet: peer = **.**.***.***
    31     09:45:03.083  10/01/13  Sev=Info/4    IKE/0x63000014
    RECEIVING <<< ISAKMP OAK TRANS *(HASH, ATTR) from **.**.***.***
    32     09:45:03.083  10/01/13  Sev=Info/4    IKE/0x63000013
    SENDING >>> ISAKMP OAK TRANS *(HASH, ATTR) to **.**.***.***
    33     09:45:03.083  10/01/13  Sev=Info/4    CM/0x6310000E
    Established Phase 1 SA.  1 Crypto Active IKE SA, 1 User Authenticated IKE SA in the system
    34     09:45:03.083  10/01/13  Sev=Info/5    IKE/0x6300005E
    Client sending a firewall request to concentrator
    35     09:45:03.083  10/01/13  Sev=Info/4    IKE/0x63000013
    SENDING >>> ISAKMP OAK TRANS *(HASH, ATTR) to **.**.***.***
    36     09:45:03.146  10/01/13  Sev=Info/5    IKE/0x6300002F
    Received ISAKMP packet: peer = **.**.***.***
    37     09:45:03.146  10/01/13  Sev=Info/4    IKE/0x63000014
    RECEIVING <<< ISAKMP OAK TRANS *(HASH, ATTR) from **.**.***.***
    38     09:45:03.146  10/01/13  Sev=Info/5    IKE/0x63000010
    MODE_CFG_REPLY: Attribute = INTERNAL_IPV4_ADDRESS: , value = 192.168.2.70
    39     09:45:03.146  10/01/13  Sev=Info/5    IKE/0x63000010
    MODE_CFG_REPLY: Attribute = INTERNAL_IPV4_NETMASK: , value = 255.255.255.0
    40     09:45:03.146  10/01/13  Sev=Info/5    IKE/0x63000010
    MODE_CFG_REPLY: Attribute = INTERNAL_IPV4_DNS(1): , value = 192.168.2.1
    41     09:45:03.146  10/01/13  Sev=Info/5    IKE/0x63000010
    MODE_CFG_REPLY: Attribute = INTERNAL_IPV4_DNS(2): , value = 8.8.8.8
    42     09:45:03.146  10/01/13  Sev=Info/5    IKE/0x6300000D
    MODE_CFG_REPLY: Attribute = MODECFG_UNITY_SAVEPWD: , value = 0x00000001
    43     09:45:03.146  10/01/13  Sev=Info/5    IKE/0x6300000D
    MODE_CFG_REPLY: Attribute = MODECFG_UNITY_SPLIT_INCLUDE (# of split_nets), value = 0x00000001
    44     09:45:03.146  10/01/13  Sev=Info/5    IKE/0x6300000F
    SPLIT_NET #1
        subnet = 192.168.2.0
        mask = 255.255.255.0
        protocol = 0
        src port = 0
        dest port=0
    45     09:45:03.146  10/01/13  Sev=Info/5    IKE/0x6300000E
    MODE_CFG_REPLY: Attribute = MODECFG_UNITY_DEFDOMAIN: , value = NCHCO.local
    46     09:45:03.146  10/01/13  Sev=Info/5    IKE/0x6300000D
    MODE_CFG_REPLY: Attribute = MODECFG_UNITY_UDP_NAT_PORT, value = 0x00002710
    47     09:45:03.146  10/01/13  Sev=Info/5    IKE/0x6300000D
    MODE_CFG_REPLY: Attribute = MODECFG_UNITY_PFS: , value = 0x00000000
    48     09:45:03.146  10/01/13  Sev=Info/5    IKE/0x6300000E
    MODE_CFG_REPLY: Attribute = APPLICATION_VERSION, value = Cisco Systems, Inc ASA5505 Version 8.4(1) built by builders on Mon 31-Jan-11 02:11
    49     09:45:03.146  10/01/13  Sev=Info/5    IKE/0x6300000D
    MODE_CFG_REPLY: Attribute = MODECFG_UNITY_SMARTCARD_REMOVAL_DISCONNECT: , value = 0x00000001
    50     09:45:03.146  10/01/13  Sev=Info/4    CM/0x63100019
    Mode Config data received
    51     09:45:03.146  10/01/13  Sev=Info/4    IKE/0x63000056
    Received a key request from Driver: Local IP = 192.168.2.70, GW IP = **.**.***.***, Remote IP = 0.0.0.0
    52     09:45:03.146  10/01/13  Sev=Info/4    IKE/0x63000013
    SENDING >>> ISAKMP OAK QM *(HASH, SA, NON, ID, ID) to **.**.***.***
    53     09:45:03.177  10/01/13  Sev=Info/5    IKE/0x6300002F
    Received ISAKMP packet: peer = **.**.***.***
    54     09:45:03.177  10/01/13  Sev=Info/4    IKE/0x63000014
    RECEIVING <<< ISAKMP OAK INFO *(HASH, NOTIFY:STATUS_RESP_LIFETIME) from **.**.***.***
    55     09:45:03.177  10/01/13  Sev=Info/5    IKE/0x63000045
    RESPONDER-LIFETIME notify has value of 86400 seconds
    56     09:45:03.177  10/01/13  Sev=Info/5    IKE/0x63000047
    This SA has already been alive for 1 seconds, setting expiry to 86399 seconds from now
    57     09:45:03.193  10/01/13  Sev=Info/5    IKE/0x6300002F
    Received ISAKMP packet: peer = **.**.***.***
    58     09:45:03.193  10/01/13  Sev=Info/4    IKE/0x63000014
    RECEIVING <<< ISAKMP OAK QM *(HASH, SA, NON, ID, ID, NOTIFY:STATUS_RESP_LIFETIME) from **.**.***.***
    59     09:45:03.193  10/01/13  Sev=Info/5    IKE/0x63000045
    RESPONDER-LIFETIME notify has value of 28800 seconds
    60     09:45:03.193  10/01/13  Sev=Info/4    IKE/0x63000013
    SENDING >>> ISAKMP OAK QM *(HASH) to **.**.***.***
    61     09:45:03.193  10/01/13  Sev=Info/5    IKE/0x63000059
    Loading IPsec SA (MsgID=967A3C93 OUTBOUND SPI = 0xAAAF4C1C INBOUND SPI = 0x3EBEBFC5)
    62     09:45:03.193  10/01/13  Sev=Info/5    IKE/0x63000025
    Loaded OUTBOUND ESP SPI: 0xAAAF4C1C
    63     09:45:03.193  10/01/13  Sev=Info/5    IKE/0x63000026
    Loaded INBOUND ESP SPI: 0x3EBEBFC5
    64     09:45:03.193  10/01/13  Sev=Info/5    CVPND/0x63400013
        Destination           Netmask           Gateway         Interface   Metric
            0.0.0.0           0.0.0.0       96.11.251.1     96.11.251.149      261
        96.11.251.0     255.255.255.0     96.11.251.149     96.11.251.149      261
      96.11.251.149   255.255.255.255     96.11.251.149     96.11.251.149      261
      96.11.251.255   255.255.255.255     96.11.251.149     96.11.251.149      261
          127.0.0.0         255.0.0.0         127.0.0.1         127.0.0.1      306
          127.0.0.1   255.255.255.255         127.0.0.1         127.0.0.1      306
    127.255.255.255   255.255.255.255         127.0.0.1         127.0.0.1      306
        192.168.1.0     255.255.255.0       192.168.1.3       192.168.1.3      261
        192.168.1.3   255.255.255.255       192.168.1.3       192.168.1.3      261
      192.168.1.255   255.255.255.255       192.168.1.3       192.168.1.3      261
          224.0.0.0         240.0.0.0         127.0.0.1         127.0.0.1      306
          224.0.0.0         240.0.0.0     96.11.251.149     96.11.251.149      261
          224.0.0.0         240.0.0.0       192.168.1.3       192.168.1.3      261
    255.255.255.255   255.255.255.255         127.0.0.1         127.0.0.1      306
    255.255.255.255   255.255.255.255     96.11.251.149     96.11.251.149      261
    255.255.255.255   255.255.255.255       192.168.1.3       192.168.1.3      261
    65     09:45:03.521  10/01/13  Sev=Info/6    CVPND/0x63400001
    Launch VAInst64 to control IPSec Virtual Adapter
    66     09:45:03.896  10/01/13  Sev=Info/4    CM/0x63100034
    The Virtual Adapter was enabled:
        IP=192.168.2.70/255.255.255.0
        DNS=192.168.2.1,8.8.8.8
        WINS=0.0.0.0,0.0.0.0
        Domain=NCHCO.local
        Split DNS Names=
    67     09:45:03.912  10/01/13  Sev=Info/5    CVPND/0x63400013
        Destination           Netmask           Gateway         Interface   Metric
            0.0.0.0           0.0.0.0       96.11.251.1     96.11.251.149      261
        96.11.251.0     255.255.255.0     96.11.251.149     96.11.251.149      261
      96.11.251.149   255.255.255.255     96.11.251.149     96.11.251.149      261
      96.11.251.255   255.255.255.255     96.11.251.149     96.11.251.149      261
          127.0.0.0         255.0.0.0         127.0.0.1         127.0.0.1      306
          127.0.0.1   255.255.255.255         127.0.0.1         127.0.0.1      306
    127.255.255.255   255.255.255.255         127.0.0.1         127.0.0.1      306
        192.168.1.0     255.255.255.0       192.168.1.3       192.168.1.3      261
        192.168.1.3   255.255.255.255       192.168.1.3       192.168.1.3      261
      192.168.1.255   255.255.255.255       192.168.1.3       192.168.1.3      261
          224.0.0.0         240.0.0.0         127.0.0.1         127.0.0.1      306
          224.0.0.0         240.0.0.0     96.11.251.149     96.11.251.149      261
          224.0.0.0         240.0.0.0       192.168.1.3       192.168.1.3      261
          224.0.0.0         240.0.0.0           0.0.0.0           0.0.0.0      261
    255.255.255.255   255.255.255.255         127.0.0.1         127.0.0.1      306
    255.255.255.255   255.255.255.255     96.11.251.149     96.11.251.149      261
    255.255.255.255   255.255.255.255       192.168.1.3       192.168.1.3      261
    255.255.255.255   255.255.255.255           0.0.0.0           0.0.0.0      261
    68     09:45:07.912  10/01/13  Sev=Info/4    CM/0x63100038
    Successfully saved route changes to file.
    69     09:45:07.912  10/01/13  Sev=Info/5    CVPND/0x63400013
        Destination           Netmask           Gateway         Interface   Metric
            0.0.0.0           0.0.0.0       96.11.251.1     96.11.251.149      261
      **.**.***.***   255.255.255.255       96.11.251.1     96.11.251.149      100
        96.11.251.0     255.255.255.0     96.11.251.149     96.11.251.149      261
      96.11.251.149   255.255.255.255     96.11.251.149     96.11.251.149      261
      96.11.251.255   255.255.255.255     96.11.251.149     96.11.251.149      261
          127.0.0.0         255.0.0.0         127.0.0.1         127.0.0.1      306
          127.0.0.1   255.255.255.255         127.0.0.1         127.0.0.1      306
    127.255.255.255   255.255.255.255         127.0.0.1         127.0.0.1      306
        192.168.1.0     255.255.255.0       192.168.1.3       192.168.1.3      261
        192.168.1.3   255.255.255.255       192.168.1.3       192.168.1.3      261
      192.168.1.255   255.255.255.255       192.168.1.3       192.168.1.3      261
        192.168.2.0     255.255.255.0      192.168.2.70      192.168.2.70      261
        192.168.2.0     255.255.255.0       192.168.2.1      192.168.2.70      100
       192.168.2.70   255.255.255.255      192.168.2.70      192.168.2.70      261
      192.168.2.255   255.255.255.255      192.168.2.70      192.168.2.70      261
          224.0.0.0         240.0.0.0         127.0.0.1         127.0.0.1      306
          224.0.0.0         240.0.0.0     96.11.251.149     96.11.251.149      261
          224.0.0.0         240.0.0.0       192.168.1.3       192.168.1.3      261
          224.0.0.0         240.0.0.0      192.168.2.70      192.168.2.70      261
    255.255.255.255   255.255.255.255         127.0.0.1         127.0.0.1      306
    255.255.255.255   255.255.255.255     96.11.251.149     96.11.251.149      261
    255.255.255.255   255.255.255.255       192.168.1.3       192.168.1.3      261
    255.255.255.255   255.255.255.255      192.168.2.70      192.168.2.70      261
    70     09:45:07.912  10/01/13  Sev=Info/6    CM/0x63100036
    The routing table was updated for the Virtual Adapter
    71     09:45:07.912  10/01/13  Sev=Info/4    CM/0x6310001A
    One secure connection established
    72     09:45:07.943  10/01/13  Sev=Info/4    CM/0x6310003B
    Address watch added for 96.11.251.149.  Current hostname: psaserver, Current address(es): 192.168.2.70, 96.11.251.149, 192.168.1.3.
    73     09:45:07.943  10/01/13  Sev=Info/4    CM/0x6310003B
    Address watch added for 192.168.2.70.  Current hostname: psaserver, Current address(es): 192.168.2.70, 96.11.251.149, 192.168.1.3.
    74     09:45:07.943  10/01/13  Sev=Info/5    CM/0x63100001
    Did not find the Smartcard to watch for removal
    75     09:45:07.943  10/01/13  Sev=Info/4    IPSEC/0x63700014
    Deleted all keys
    76     09:45:07.943  10/01/13  Sev=Info/4    IPSEC/0x63700010
    Created a new key structure
    77     09:45:07.943  10/01/13  Sev=Info/4    IPSEC/0x6370000F
    Added key with SPI=0x1c4cafaa into key list
    78     09:45:07.943  10/01/13  Sev=Info/4    IPSEC/0x63700010
    Created a new key structure
    79     09:45:07.943  10/01/13  Sev=Info/4    IPSEC/0x6370000F
    Added key with SPI=0xc5bfbe3e into key list
    80     09:45:07.943  10/01/13  Sev=Info/4    IPSEC/0x6370002F
    Assigned VA private interface addr 192.168.2.70
    81     09:45:07.943  10/01/13  Sev=Info/4    IPSEC/0x63700037
    Configure public interface: 96.11.251.149. SG: **.**.***.***
    82     09:45:07.943  10/01/13  Sev=Info/6    CM/0x63100046
    Set tunnel established flag in registry to 1.
    83     09:45:13.459  10/01/13  Sev=Info/4    IKE/0x63000013
    SENDING >>> ISAKMP OAK INFO *(HASH, NOTIFY:DPD_REQUEST) to **.**.***.***
    84     09:45:13.459  10/01/13  Sev=Info/6    IKE/0x6300003D
    Sending DPD request to **.**.***.***, our seq# = 107205276
    85     09:45:13.474  10/01/13  Sev=Info/5    IKE/0x6300002F
    Received ISAKMP packet: peer = **.**.***.***
    86     09:45:13.474  10/01/13  Sev=Info/4    IKE/0x63000014
    RECEIVING <<< ISAKMP OAK INFO *(HASH, NOTIFY:DPD_ACK) from **.**.***.***
    87     09:45:13.474  10/01/13  Sev=Info/5    IKE/0x63000040
    Received DPD ACK from **.**.***.***, seq# received = 107205276, seq# expected = 107205276
    88     09:45:15.959  10/01/13  Sev=Info/4    IPSEC/0x63700019
    Activate outbound key with SPI=0x1c4cafaa for inbound key with SPI=0xc5bfbe3e
    89     09:46:00.947  10/01/13  Sev=Info/4    IKE/0x63000013
    SENDING >>> ISAKMP OAK INFO *(HASH, NOTIFY:DPD_REQUEST) to **.**.***.***
    90     09:46:00.947  10/01/13  Sev=Info/6    IKE/0x6300003D
    Sending DPD request to **.**.***.***, our seq# = 107205277
    91     09:46:01.529  10/01/13  Sev=Info/5    IKE/0x6300002F
    Received ISAKMP packet: peer = **.**.***.***
    92     09:46:01.529  10/01/13  Sev=Info/4    IKE/0x63000014
    RECEIVING <<< ISAKMP OAK INFO *(HASH, NOTIFY:DPD_ACK) from **.**.***.***
    93     09:46:01.529  10/01/13  Sev=Info/5    IKE/0x63000040
    Received DPD ACK from **.**.***.***, seq# received = 107205277, seq# expected = 107205277
    94     09:46:11.952  10/01/13  Sev=Info/4    IKE/0x63000013
    SENDING >>> ISAKMP OAK INFO *(HASH, NOTIFY:DPD_REQUEST) to **.**.***.***
    95     09:46:11.952  10/01/13  Sev=Info/6    IKE/0x6300003D
    Sending DPD request to **.**.***.***, our seq# = 107205278
    96     09:46:11.979  10/01/13  Sev=Info/5    IKE/0x6300002F
    Received ISAKMP packet: peer = **.**.***.***
    97     09:46:11.979  10/01/13  Sev=Info/4    IKE/0x63000014
    RECEIVING <<< ISAKMP OAK INFO *(HASH, NOTIFY:DPD_ACK) from **.**.***.***
    98     09:46:11.979  10/01/13  Sev=Info/5    IKE/0x63000040
    Received DPD ACK from **.**.***.***, seq# received = 107205278, seq# expected = 107205278
    Any help would be appreciated, thanks!

    I made the change that you requested by moving the VPN pool to the 192.168.3.0 network. Unfortunately, now traffic isn't flowing to the inside network at all. I was going to make a specific route as you suggested, but as far as I can see the routes are already being created correctly on the VPN client's end.
    Here is the route print off of the computer behind the (test) client:
    ===========================================================================
    Interface List
    21...00 05 9a 3c 78 00 ......Cisco Systems VPN Adapter for 64-bit Windows
    10...00 15 5d 01 02 01 ......Microsoft Hyper-V Network Adapter
    15...00 15 5d 01 02 02 ......Microsoft Hyper-V Network Adapter #2
      1...........................Software Loopback Interface 1
    13...00 00 00 00 00 00 00 e0 Microsoft ISATAP Adapter
    11...00 00 00 00 00 00 00 e0 Teredo Tunneling Pseudo-Interface
    14...00 00 00 00 00 00 00 e0 Microsoft 6to4 Adapter
    16...00 00 00 00 00 00 00 e0 Microsoft ISATAP Adapter #2
    23...00 00 00 00 00 00 00 e0 Microsoft ISATAP Adapter #3
    ===========================================================================
    IPv4 Route Table
    ===========================================================================
    Active Routes:
    Network Destination        Netmask          Gateway       Interface  Metric
              0.0.0.0          0.0.0.0      96.11.251.1    96.11.251.149    261
        69.61.228.178  255.255.255.255      96.11.251.1    96.11.251.149    100
          96.11.251.0    255.255.255.0         On-link     96.11.251.149    261
        96.11.251.149  255.255.255.255         On-link     96.11.251.149    261
        96.11.251.255  255.255.255.255         On-link     96.11.251.149    261
            127.0.0.0        255.0.0.0         On-link         127.0.0.1    306
            127.0.0.1  255.255.255.255         On-link         127.0.0.1    306
      127.255.255.255  255.255.255.255         On-link         127.0.0.1    306
          192.168.1.0    255.255.255.0         On-link       192.168.1.3    261
          192.168.1.3  255.255.255.255         On-link       192.168.1.3    261
        192.168.1.255  255.255.255.255         On-link       192.168.1.3    261
          192.168.2.0    255.255.255.0      192.168.3.1     192.168.3.70    100
          192.168.3.0    255.255.255.0         On-link      192.168.3.70    261
         192.168.3.70  255.255.255.255         On-link      192.168.3.70    261
        192.168.3.255  255.255.255.255         On-link      192.168.3.70    261
            224.0.0.0        240.0.0.0         On-link         127.0.0.1    306
            224.0.0.0        240.0.0.0         On-link       192.168.1.3    261
            224.0.0.0        240.0.0.0         On-link     96.11.251.149    261
            224.0.0.0        240.0.0.0         On-link      192.168.3.70    261
      255.255.255.255  255.255.255.255         On-link         127.0.0.1    306
      255.255.255.255  255.255.255.255         On-link       192.168.1.3    261
      255.255.255.255  255.255.255.255         On-link     96.11.251.149    261
      255.255.255.255  255.255.255.255         On-link      192.168.3.70    261
    ===========================================================================
    Persistent Routes:
      Network Address          Netmask  Gateway Address  Metric
              0.0.0.0          0.0.0.0      96.11.251.1  Default
    ===========================================================================
    IPv6 Route Table
    ===========================================================================
    Active Routes:
    If Metric Network Destination      Gateway
    14   1020 ::/0                     2002:c058:6301::c058:6301
    14   1020 ::/0                     2002:c058:6301::1
      1    306 ::1/128                  On-link
    14   1005 2002::/16                On-link
    14    261 2002:600b:fb95::600b:fb95/128
                                        On-link
    15    261 fe80::/64                On-link
    10    261 fe80::/64                On-link
    21    261 fe80::/64                On-link
    10    261 fe80::64ae:bae7:3dc0:c8c4/128
                                        On-link
    21    261 fe80::e9f7:e24:3147:bd/128
                                        On-link
    15    261 fe80::f116:2dfd:1771:125a/128
                                        On-link
      1    306 ff00::/8                 On-link
    15    261 ff00::/8                 On-link
    10    261 ff00::/8                 On-link
    21    261 ff00::/8                 On-link
    ===========================================================================
    Persistent Routes:
      None
    And here is the updated running config in case you need it:
    : Saved
    ASA Version 8.4(1)
    hostname NCHCO
    enable password hTjwXz/V8EuTw9p9 encrypted
    passwd hTjwXz/V8EuTw9p9 encrypted
    names
    name 192.168.2.0 NCHCO description City Offices
    name 192.168.2.80 VPN_End
    name 192.168.2.70 VPN_Start
    interface Vlan1
    nameif inside
    security-level 100
    ip address 192.168.2.1 255.255.255.0
    interface Vlan2
    nameif outside
    security-level 0
    ip address 69.61.228.178 255.255.255.248
    interface Ethernet0/0
    switchport access vlan 2
    speed 100
    duplex full
    interface Ethernet0/1
    interface Ethernet0/2
    interface Ethernet0/3
    interface Ethernet0/4
    interface Ethernet0/5
    interface Ethernet0/6
    interface Ethernet0/7
    boot system disk0:/asa841-k8.bin
    ftp mode passive
    object network NCHCO
    subnet 192.168.2.0 255.255.255.0
    object network obj-192.168.1.0
    subnet 192.168.1.0 255.255.255.0
    object network obj-192.168.2.64
    subnet 192.168.2.64 255.255.255.224
    object network obj-0.0.0.0
    subnet 0.0.0.0 255.255.255.0
    object network obj_any
    subnet 0.0.0.0 0.0.0.0
    object network Webserver
    object network FINX
    host 192.168.2.11
    object service rdp
    service tcp source range 1 65535 destination eq 3389
    description rdp 
    object network obj-192.168.3.0
    subnet 192.168.3.0 255.255.255.0
    object network obj-192.168.2.0
    subnet 192.168.2.0 255.255.255.0
    access-list outside_nat0_outbound extended permit ip object NCHCO 192.168.1.0 255.255.255.0
    access-list outside_nat0_outbound extended permit ip object NCHCO 192.168.2.0 255.255.255.0
    access-list inside_nat0_outbound extended permit ip object NCHCO 192.168.1.0 255.255.255.0
    access-list inside_nat0_outbound extended permit ip any 192.168.2.64 255.255.255.224
    access-list inside_nat0_outbound extended permit ip 0.0.0.0 255.255.255.0 192.168.2.64 255.255.255.224
    access-list outside_1_cryptomap extended permit ip object NCHCO 192.168.1.0 255.255.255.0
    access-list outside_1_cryptomap_1 extended permit ip object NCHCO 192.168.1.0 255.255.255.0
    access-list LAN_Access standard permit 192.168.2.0 255.255.255.0
    access-list LAN_Access standard permit 0.0.0.0 255.255.255.0
    access-list NCHCO_splitTunnelAcl_1 standard permit 192.168.2.0 255.255.255.0
    access-list AnyConnect_Client_Local_Print extended permit tcp any any eq lpd
    access-list AnyConnect_Client_Local_Print remark IPP: Internet Printing Protocol
    access-list AnyConnect_Client_Local_Print extended permit tcp any any eq 631
    access-list AnyConnect_Client_Local_Print remark Windows' printing port
    access-list AnyConnect_Client_Local_Print extended permit tcp any any eq 9100
    access-list AnyConnect_Client_Local_Print remark mDNS: multicast DNS protocol
    access-list AnyConnect_Client_Local_Print extended permit udp any host 224.0.0.251 eq 5353
    access-list AnyConnect_Client_Local_Print remark LLMNR: Link Local Multicast Name Resolution protocol
    access-list AnyConnect_Client_Local_Print extended permit udp any host 224.0.0.252 eq 5355
    access-list AnyConnect_Client_Local_Print remark TCP/NetBIOS protocol
    access-list AnyConnect_Client_Local_Print extended permit tcp any any eq 137
    access-list AnyConnect_Client_Local_Print extended permit udp any any eq netbios-ns
    access-list AnyConnect_Client_Local_Print extended deny ip any any
    access-list outside_access_in extended permit tcp any object FINX eq 3389
    access-list outside_access_in_1 extended permit object rdp any object FINX
    access-list outside_specific_blocks extended deny ip host 121.168.66.35 any
    pager lines 24
    logging enable
    logging asdm informational
    mtu inside 1500
    mtu outside 1500
    ip local pool VPN_Pool VPN_Start-VPN_End mask 255.255.255.0
    ip local pool VPN_Split_Pool 192.168.3.70-192.168.3.80 mask 255.255.255.0
    icmp unreachable rate-limit 1 burst-size 1
    asdm image disk0:/asdm-649.bin
    no asdm history enable
    arp timeout 14400
    nat (inside,any) source static NCHCO NCHCO destination static obj-192.168.1.0 obj-192.168.1.0
    nat (inside,any) source static any any destination static obj-192.168.2.64 obj-192.168.2.64
    nat (inside,any) source static obj-0.0.0.0 obj-0.0.0.0 destination static obj-192.168.2.64 obj-192.168.2.64
    object network obj_any
    nat (inside,outside) dynamic interface
    object network FINX
    nat (inside,outside) static interface service tcp 3389 3389
    access-group outside_access_in_1 in interface outside
    route outside 0.0.0.0 0.0.0.0 69.61.228.177 1
    timeout xlate 3:00:00
    timeout conn 1:00:00 half-closed 0:10:00 udp 0:02:00 icmp 0:00:02
    timeout sunrpc 0:10:00 h323 0:05:00 h225 1:00:00 mgcp 0:05:00 mgcp-pat 0:05:00
    timeout sip 0:30:00 sip_media 0:02:00 sip-invite 0:03:00 sip-disconnect 0:02:00
    timeout sip-provisional-media 0:02:00 uauth 0:05:00 absolute
    timeout tcp-proxy-reassembly 0:01:00
    dynamic-access-policy-record DfltAccessPolicy
    network-acl outside_nat0_outbound
    webvpn
      svc ask enable default svc
    http server enable
    http 192.168.1.0 255.255.255.0 inside
    http 69.61.228.178 255.255.255.255 outside
    http 74.218.158.238 255.255.255.255 outside
    http NCHCO 255.255.255.0 inside
    http 96.11.251.186 255.255.255.255 outside
    no snmp-server location
    no snmp-server contact
    snmp-server enable traps snmp authentication linkup linkdown coldstart warmstart
    crypto ipsec ikev1 transform-set ESP-AES-128-SHA esp-aes esp-sha-hmac
    crypto ipsec ikev1 transform-set ESP-AES-128-MD5 esp-aes esp-md5-hmac
    crypto ipsec ikev1 transform-set ESP-AES-192-SHA esp-aes-192 esp-sha-hmac
    crypto ipsec ikev1 transform-set ESP-AES-192-MD5 esp-aes-192 esp-md5-hmac
    crypto ipsec ikev1 transform-set ESP-AES-256-SHA esp-aes-256 esp-sha-hmac
    crypto ipsec ikev1 transform-set ESP-AES-256-MD5 esp-aes-256 esp-md5-hmac
    crypto ipsec ikev1 transform-set ESP-3DES-MD5 esp-3des esp-md5-hmac
    crypto ipsec ikev1 transform-set ESP-DES-SHA esp-des esp-sha-hmac
    crypto ipsec ikev1 transform-set ESP-DES-MD5 esp-des esp-md5-hmac
    crypto ipsec ikev1 transform-set l2tp-transform esp-3des esp-sha-hmac
    crypto ipsec ikev1 transform-set l2tp-transform mode transport
    crypto ipsec ikev1 transform-set vpn-transform esp-aes-256 esp-sha-hmac
    crypto ipsec ikev1 transform-set TRANS_ESP_3DES_SHA esp-3des esp-sha-hmac
    crypto ipsec ikev1 transform-set TRANS_ESP_3DES_SHA mode transport
    crypto ipsec ikev1 transform-set TRANS_ESP_3DES_MD5 esp-3des esp-md5-hmac
    crypto ipsec ikev1 transform-set TRANS_ESP_3DES_MD5 mode transport
    crypto ipsec ikev1 transform-set ESP-3DES-SHA esp-3des esp-sha-hmac
    crypto dynamic-map SYSTEM_DEFAULT_CRYPTO_MAP 65535 set pfs group1
    crypto dynamic-map SYSTEM_DEFAULT_CRYPTO_MAP 65535 set ikev1 transform-set ESP-AES-128-SHA ESP-AES-128-MD5 ESP-AES-192-SHA ESP-AES-192-MD5 ESP-AES-256-SHA ESP-AES-256-MD5 ESP-3DES-SHA ESP-3DES-MD5 ESP-DES-SHA ESP-DES-MD5
    crypto dynamic-map dyn-map 10 set pfs group1
    crypto dynamic-map dyn-map 10 set ikev1 transform-set l2tp-transform vpn-transform
    crypto dynamic-map dyn-map 10 set reverse-route
    crypto dynamic-map outside_dyn_map 20 set ikev1 transform-set ESP-3DES-SHA
    crypto dynamic-map outside_dyn_map 20 set reverse-route
    crypto map outside_map 1 match address outside_1_cryptomap
    crypto map outside_map 1 set pfs group1
    crypto map outside_map 1 set peer 74.219.208.50
    crypto map outside_map 1 set ikev1 transform-set ESP-3DES-SHA
    crypto map outside_map 20 ipsec-isakmp dynamic outside_dyn_map
    crypto map outside_map 65535 ipsec-isakmp dynamic SYSTEM_DEFAULT_CRYPTO_MAP
    crypto map outside_map interface outside
    crypto map inside_map 65535 ipsec-isakmp dynamic SYSTEM_DEFAULT_CRYPTO_MAP
    crypto map inside_map interface inside
    crypto map vpn-map 1 match address outside_1_cryptomap_1
    crypto map vpn-map 1 set pfs group1
    crypto map vpn-map 1 set peer 74.219.208.50
    crypto map vpn-map 1 set ikev1 transform-set ESP-3DES-SHA
    crypto map vpn-map 10 ipsec-isakmp dynamic dyn-map
    crypto isakmp identity address
    crypto ikev1 enable inside
    crypto ikev1 enable outside
    crypto ikev1 ipsec-over-tcp port 10000
    crypto ikev1 policy 10
    authentication pre-share
    encryption 3des
    hash md5
    group 2
    lifetime 86400
    crypto ikev1 policy 15
    authentication pre-share
    encryption aes-256
    hash sha
    group 2
    lifetime 86400
    crypto ikev1 policy 35
    authentication pre-share
    encryption 3des
    hash sha
    group 2
    lifetime 86400
    client-update enable
    telnet 192.168.1.0 255.255.255.0 inside
    telnet NCHCO 255.255.255.0 inside
    telnet timeout 5
    ssh 192.168.1.0 255.255.255.0 inside
    ssh NCHCO 255.255.255.0 inside
    ssh 96.11.251.186 255.255.255.255 outside
    ssh timeout 5
    console timeout 0
    dhcpd address 192.168.2.150-192.168.2.225 inside
    dhcpd dns 216.68.4.10 216.68.5.10 interface inside
    dhcpd lease 64000 interface inside
    threat-detection basic-threat
    threat-detection statistics host
    threat-detection statistics port
    threat-detection statistics protocol
    threat-detection statistics access-list
    no threat-detection statistics tcp-intercept
    webvpn
    group-policy DefaultRAGroup internal
    group-policy DefaultRAGroup attributes
    dns-server value 192.168.2.1
    vpn-tunnel-protocol ikev1 l2tp-ipsec
    default-domain value nchco.local
    group-policy DfltGrpPolicy attributes
    dns-server value 192.168.2.1
    vpn-tunnel-protocol ikev1 l2tp-ipsec ssl-client ssl-clientless
    password-storage enable
    ipsec-udp enable
    intercept-dhcp 255.255.255.0 enable
    address-pools value VPN_Split_Pool
    group-policy NCHCO internal
    group-policy NCHCO attributes
    dns-server value 192.168.2.1 8.8.8.8
    vpn-tunnel-protocol ikev1
    split-tunnel-policy tunnelspecified
    split-tunnel-network-list value NCHCO_splitTunnelAcl_1
    default-domain value NCHCO.local
    username admin password LbMiJuAJjDaFb2uw encrypted privilege 15
    username 8njferg password yB1lHEVmHZGj5C2Z encrypted privilege 15
    username NCHvpn99 password dhn.JzttvRmMbHsP encrypted
    tunnel-group DefaultRAGroup general-attributes
    address-pool (inside) VPN_Pool
    address-pool VPN_Split_Pool
    authentication-server-group (inside) LOCAL
    authentication-server-group (outside) LOCAL
    authorization-server-group LOCAL
    authorization-server-group (inside) LOCAL
    authorization-server-group (outside) LOCAL
    default-group-policy DefaultRAGroup
    strip-realm
    strip-group
    tunnel-group DefaultRAGroup ipsec-attributes
    ikev1 pre-shared-key *****
    peer-id-validate nocheck
    tunnel-group DefaultRAGroup ppp-attributes
    no authentication chap
    no authentication ms-chap-v1
    authentication ms-chap-v2
    tunnel-group DefaultWEBVPNGroup ppp-attributes
    authentication pap
    authentication ms-chap-v2
    tunnel-group 74.219.208.50 type ipsec-l2l
    tunnel-group 74.219.208.50 ipsec-attributes
    ikev1 pre-shared-key *****
    tunnel-group NCHCO type remote-access
    tunnel-group NCHCO general-attributes
    address-pool VPN_Split_Pool
    default-group-policy NCHCO
    tunnel-group NCHCO ipsec-attributes
    ikev1 pre-shared-key *****
    class-map inspection_default
    match default-inspection-traffic
    policy-map type inspect dns preset_dns_map
    parameters
      message-length maximum client auto
      message-length maximum 512
    policy-map global_policy
    class inspection_default
      inspect dns preset_dns_map
      inspect ftp
      inspect h323 h225
      inspect h323 ras
      inspect rsh
      inspect rtsp
      inspect esmtp
      inspect sqlnet
      inspect skinny 
      inspect sunrpc
      inspect xdmcp
      inspect sip 
      inspect netbios
      inspect tftp
      inspect ip-options
    service-policy global_policy global
    prompt hostname context
    call-home
    profile CiscoTAC-1
      no active
      destination address http https://tools.cisco.com/its/service/oddce/services/DDCEService
      destination address email [email protected]
      destination transport-method http
      subscribe-to-alert-group diagnostic
      subscribe-to-alert-group environment
      subscribe-to-alert-group inventory periodic monthly
      subscribe-to-alert-group configuration periodic monthly
      subscribe-to-alert-group telemetry periodic daily
    Cryptochecksum:9e8466cd318c0bd35bc660fa65ba7a03
    : end
    asdm image disk0:/asdm-649.bin
    asdm location VPN_Start 255.255.255.255 inside
    asdm location VPN_End 255.255.255.255 inside
    no asdm history enable
    Thanks again for your help,
    Matthew

  • Site to Site VPN Problems With 2801 Router and ASA 5505

    Hello,
    I am having some issue setting up a site to site ipsec VPN between a Cisco 2801 router and a Cisco ASA 5505. I was told there was a vpn previously setup with an old hosting provider, but those connections have been servered. Right now I am trying to get the sites to talk to the 2801. Here ere are my current configs, please let me know if you need anything else. Im stumped on this one. Thanks.
    IP scheme at SIte A:
    IP    172.19.3.x
    sub 255.255.255.128
    GW 172.19.3.129
    Site A Ciscso 2801 Router
    Current configuration : 11858 bytes
    version 12.4
    service timestamps debug datetime localtime
    service timestamps log datetime localtime show-timezone
    service password-encryption
    hostname router-2801
    boot-start-marker
    boot-end-marker
    logging message-counter syslog
    logging buffered 4096
    aaa new-model
    aaa authentication login userauthen group radius local
    aaa authorization network groupauthor local
    aaa session-id common
    clock timezone est -5
    clock summer-time zone recurring last Sun Mar 2:00 1 Sun Nov 2:00
    dot11 syslog
    ip source-route
    ip dhcp excluded-address 172.19.3.129 172.19.3.149
    ip dhcp excluded-address 172.19.10.1 172.19.10.253
    ip dhcp excluded-address 172.19.3.140
    ip dhcp ping timeout 900
    ip dhcp pool DHCP
       network 172.19.3.128 255.255.255.128
       default-router 172.19.3.129
       domain-name domain.local
       netbios-name-server 172.19.3.7
       option 66 ascii 172.19.3.225
       dns-server 172.19.3.140 208.67.220.220 208.67.222.222
    ip dhcp pool VoiceDHCP
       network 172.19.10.0 255.255.255.0
       default-router 172.19.10.1
       dns-server 208.67.220.220 8.8.8.8
       option 66 ascii 172.19.10.2
       lease 2
    ip cef
    ip inspect name SDM_LOW cuseeme
    ip inspect name SDM_LOW dns
    ip inspect name SDM_LOW ftp
    ip inspect name SDM_LOW h323
    ip inspect name SDM_LOW https
    ip inspect name SDM_LOW icmp
    ip inspect name SDM_LOW imap
    ip inspect name SDM_LOW pop3
    ip inspect name SDM_LOW netshow
    ip inspect name SDM_LOW rcmd
    ip inspect name SDM_LOW realaudio
    ip inspect name SDM_LOW rtsp
    ip inspect name SDM_LOW esmtp
    ip inspect name SDM_LOW sqlnet
    ip inspect name SDM_LOW streamworks
    ip inspect name SDM_LOW tftp
    ip inspect name SDM_LOW tcp
    ip inspect name SDM_LOW udp
    ip inspect name SDM_LOW vdolive
    no ip domain lookup
    ip domain name domain.local
    multilink bundle-name authenticated
    key chain key1
    key 1
       key-string 7 06040033484B1B484557
    crypto pki trustpoint TP-self-signed-3448656681
    enrollment selfsigned
    subject-name cn=IOS-Self-Signed-Certificate-3448bb6681
    revocation-check none
    rsakeypair TP-self-signed-344bbb56681
    crypto pki certificate chain TP-self-signed-3448656681
    certificate self-signed 01
      3082024F
                quit
    username admin privilege 15 password 7 F55
    archive
    log config
      hidekeys
    crypto isakmp policy 10
    encr 3des
    hash md5
    authentication pre-share
    group 2
    crypto isakmp key XXXXX address 209.118.0.1
    crypto isakmp key xxxxx address SITE B Public IP
    crypto isakmp keepalive 40 5
    crypto isakmp nat keepalive 20
    crypto isakmp client configuration group IISVPN
    key 1nsur3m3
    dns 172.19.3.140
    wins 172.19.3.140
    domain domain.local
    pool VPN_Pool
    acl 198
    crypto isakmp profile IISVPNClient
       description VPN clients profile
       match identity group IISVPN
       client authentication list userauthen
       isakmp authorization list groupauthor
       client configuration address respond
    crypto ipsec transform-set myset esp-3des esp-md5-hmac
    crypto dynamic-map Dynamic 5
    set transform-set myset
    set isakmp-profile IISVPNClient
    qos pre-classify
    crypto map VPN 10 ipsec-isakmp
    set peer 209.118.0.1
    set peer SITE B Public IP
    set transform-set myset
    match address 101
    qos pre-classify
    crypto map VPN 65535 ipsec-isakmp dynamic Dynamic
    track 123 ip sla 1 reachability
    delay down 15 up 10
    class-map match-any VoiceTraffic
    match protocol rtp audio
    match protocol h323
    match protocol rtcp
    match access-group name VOIP
    match protocol sip
    class-map match-any RDP
    match access-group 199
    policy-map QOS
    class VoiceTraffic
        bandwidth 512
    class RDP
        bandwidth 768
    policy-map MainQOS
    class class-default
        shape average 1500000
      service-policy QOS
    interface FastEthernet0/0
    description $ETH-LAN$$ETH-SW-LAUNCH$$INTF-INFO-FE 0$$FW_INSIDE$
    ip address 172.19.3.129 255.255.255.128
    ip access-group 100 in
    ip inspect SDM_LOW in
    ip nat inside
    ip virtual-reassembly
    duplex auto
    speed auto
    interface FastEthernet0/0.10
    description $ETH-VoiceVLAN$$
    encapsulation dot1Q 10
    ip address 172.19.10.1 255.255.255.0
    ip inspect SDM_LOW in
    ip nat inside
    ip virtual-reassembly
    interface FastEthernet0/1
    description "Comcast"
    ip address PUB IP 255.255.255.248
    ip access-group 102 in
    ip inspect SDM_LOW out
    ip nat outside
    ip virtual-reassembly
    duplex auto
    speed auto
    crypto map VPN
    interface Serial0/1/0
    description "Verizon LEC Circuit ID: w0w13908 Site ID: U276420-1"
    bandwidth 1536
    no ip address
    encapsulation frame-relay IETF
    frame-relay lmi-type ansi
    interface Serial0/1/0.1 point-to-point
    bandwidth 1536
    ip address 152.000.000.18 255.255.255.252
    ip access-group 102 in
    ip verify unicast reverse-path
    ip inspect SDM_LOW out
    ip nat outside
    ip virtual-reassembly
    frame-relay interface-dlci 500 IETF 
    crypto map VPN
    service-policy output MainQOS
    interface Serial0/2/0
    description "PAETEC 46.HCGS.788446.CV (Verizon ID) / 46.HCGS.3 (PAETEC ID)"
    ip address 123.252.123.102 255.255.255.252
    ip access-group 102 in
    ip inspect SDM_LOW out
    ip nat outside
    ip virtual-reassembly
    encapsulation ppp
    crypto map VPN
    service-policy output MainQOS
    ip local pool VPN_Pool 172.20.3.130 172.20.3.254
    ip forward-protocol nd
    ip route 0.0.0.0 0.0.0.0 50.00.000.110 track 123
    ip route 0.0.0.0 0.0.0.0 111.252.237.000 254
    ip route 122.112.197.20 255.255.255.255 209.252.237.101
    ip route 208.67.220.220 255.255.255.255 50.78.233.110
    no ip http server
    no ip http secure-server
    ip http timeout-policy idle 60 life 86400 requests 10000
    ip flow-top-talkers
    top 20
    sort-by bytes
    ip nat inside source route-map COMCAST interface FastEthernet0/1 overload
    ip nat inside source route-map PAETEC interface Serial0/2/0 overload
    ip nat inside source route-map VERIZON interface Serial0/1/0.1 overload
    ip nat inside source static tcp 172.19.3.140 21 PUB IP 21 extendable
    ip access-list extended VOIP
    permit ip 172.20.3.0 0.0.0.127 host 172.19.3.190
    permit ip host 172.19.3.190 172.20.3.0 0.0.0.127
    ip radius source-interface FastEthernet0/0
    ip sla 1
    icmp-echo 000.67.220.220 source-interface FastEthernet0/1
    timeout 10000
    frequency 15
    ip sla schedule 1 life forever start-time now
    access-list 23 permit 172.19.3.0 0.0.0.127
    access-list 23 permit 172.19.3.128 0.0.0.127
    access-list 23 permit 173.189.251.192 0.0.0.63
    access-list 23 permit 107.0.197.0 0.0.0.63
    access-list 23 permit 173.163.157.32 0.0.0.15
    access-list 23 permit 72.55.33.0 0.0.0.255
    access-list 23 permit 172.19.5.0 0.0.0.63
    access-list 100 remark "Outgoing Traffic"
    access-list 100 deny   ip 67.128.87.156 0.0.0.3 any
    access-list 100 deny   ip host 255.255.255.255 any
    access-list 100 deny   ip 127.0.0.0 0.255.255.255 any
    access-list 100 permit tcp host 172.19.3.190 any eq smtp
    access-list 100 permit tcp host 172.19.3.137 any eq smtp
    access-list 100 permit tcp any host 66.251.35.131 eq smtp
    access-list 100 permit tcp any host 173.201.193.101 eq smtp
    access-list 100 permit ip any any
    access-list 100 permit tcp any any eq ftp
    access-list 101 remark "Interesting VPN Traffic"
    access-list 101 permit ip 172.19.3.128 0.0.0.127 172.19.3.0 0.0.0.127
    access-list 101 permit ip 172.20.3.128 0.0.0.127 172.19.3.0 0.0.0.127
    access-list 101 permit ip 172.19.3.128 0.0.0.127 host 172.19.250.10
    access-list 101 permit ip 172.19.3.128 0.0.0.127 host 172.19.250.11
    access-list 101 permit tcp any any eq ftp
    access-list 101 permit tcp any any eq ftp-data
    access-list 102 remark "Inbound Access"
    access-list 102 permit udp any host 152.179.53.18 eq non500-isakmp
    access-list 102 permit udp any host 152.179.53.18 eq isakmp
    access-list 102 permit esp any host 152.179.53.18
    access-list 102 permit ahp any host 152.179.53.18
    access-list 102 permit udp any host 209.000.000.102 eq non500-isakmp
    access-list 102 permit udp any host 209.000.000.102 eq isakmp
    access-list 102 permit esp any host 209.000.000.102
    access-list 102 permit ahp any host 209.000.000.102
    access-list 102 permit udp any host PUB IP eq non500-isakmp
    access-list 102 permit udp any host PUB IP eq isakmp
    access-list 102 permit esp any host PUB IP
    access-list 102 permit ahp any host PUB IP
    access-list 102 permit ip 72.55.33.0 0.0.0.255 any
    access-list 102 permit ip 107.0.197.0 0.0.0.63 any
    access-list 102 deny   ip 172.19.3.128 0.0.0.127 any
    access-list 102 permit icmp any any echo-reply
    access-list 102 permit icmp any any time-exceeded
    access-list 102 permit icmp any any unreachable
    access-list 102 permit icmp any any
    access-list 102 deny   ip any any log
    access-list 102 permit tcp any host 172.19.3.140 eq ftp
    access-list 102 permit tcp any host 172.19.3.140 eq ftp-data established
    access-list 102 permit udp any host SITE B Public IP  eq non500-isakmp
    access-list 102 permit udp any host SITE B Public IP  eq isakmp
    access-list 102 permit esp any host SITE B Public IP
    access-list 102 permit ahp any host SITE B Public IP
    access-list 110 remark "Outbound NAT Rule"
    access-list 110 remark "Deny VPN Traffic NAT"
    access-list 110 deny   ip 172.19.3.128 0.0.0.127 172.19.3.0 0.0.0.127
    access-list 110 deny   ip 172.19.3.128 0.0.0.127 172.19.10.0 0.0.0.255
    access-list 110 deny   ip 172.19.10.0 0.0.0.255 172.19.3.128 0.0.0.127
    access-list 110 deny   ip 172.20.3.128 0.0.0.127 172.19.3.0 0.0.0.127
    access-list 110 deny   ip 172.19.3.128 0.0.0.127 172.20.3.128 0.0.0.127
    access-list 110 deny   ip 172.19.3.128 0.0.0.127 host 172.19.250.11
    access-list 110 deny   ip 172.19.3.128 0.0.0.127 host 172.19.250.10
    access-list 110 permit ip 172.19.3.128 0.0.0.127 any
    access-list 110 permit ip 172.19.10.0 0.0.0.255 any
    access-list 198 remark "Networks for IISVPN Client"
    access-list 198 permit ip 172.19.3.0 0.0.0.127 172.20.3.128 0.0.0.127
    access-list 198 permit ip 172.19.3.128 0.0.0.127 172.20.3.128 0.0.0.127
    access-list 199 permit tcp any any eq 3389
    route-map PAETEC permit 10
    match ip address 110
    match interface Serial0/2/0
    route-map COMCAST permit 10
    match ip address 110
    match interface FastEthernet0/1
    route-map VERIZON permit 10
    match ip address 110
    match interface Serial0/1/0.1
    snmp-server community 123 RO
    radius-server host 172.19.3.7 auth-port 1645 acct-port 1646 key 7 000000000000000
    control-plane
    line con 0
    line aux 0
    line vty 0 4
    access-class 23 in
    privilege level 15
    transport input telnet ssh
    line vty 5 15
    access-class 23 in
    privilege level 15
    transport input telnet ssh
    scheduler allocate 20000 1000
    ntp server 128.118.25.3
    ntp server 217.150.242.8
    end
    IP scheme at site B:
    ip     172.19.5.x
    sub  255.255.255.292
    gw   172.19.5.65
    Cisco ASA 5505 at Site B
    ASA Version 8.2(5)
    hostname ASA5505
    domain-name domain.com
    enable password b04DSH2HQqXwS8wi encrypted
    passwd b04DSH2HQqXwS8wi encrypted
    names
    interface Ethernet0/0
    switchport access vlan 2
    interface Ethernet0/1
    interface Ethernet0/2
    interface Ethernet0/3
    interface Ethernet0/4
    interface Ethernet0/5
    interface Ethernet0/6
    interface Ethernet0/7
    interface Vlan1
    nameif inside
    security-level 100
    ip address 172.19.5.65 255.255.255.192
    interface Vlan2
    nameif outside
    security-level 0
    ip address SITE B public IP 255.255.255.224
    boot system disk0:/asa825-k8.bin
    ftp mode passive
    clock timezone est -5
    clock summer-time zone recurring last Sun Mar 2:00 last Sun Oct 2:00
    dns server-group DefaultDNS
    domain-name iis-usa.com
    same-security-traffic permit intra-interface
    object-group network old hosting provider
    network-object 72.55.34.64 255.255.255.192
    network-object 72.55.33.0 255.255.255.0
    network-object 173.189.251.192 255.255.255.192
    network-object 173.163.157.32 255.255.255.240
    network-object 66.11.1.64 255.255.255.192
    network-object 107.0.197.0 255.255.255.192
    object-group network old hosting provider
    network-object host 172.19.250.10
    network-object host 172.19.250.11
    access-list 100 extended permit ip 172.19.5.64 255.255.255.192 object-group old hosting provider
    access-list 100 extended permit ip 172.19.5.64 255.255.255.192 172.19.3.128 255.255.255.128
    access-list 10 extended deny ip 0.0.0.0 255.0.0.0 any
    access-list 10 extended deny ip 127.0.0.0 255.0.0.0 any
    access-list 10 extended deny ip 169.254.0.0 255.255.0.0 any
    access-list 10 extended deny ip 172.16.0.0 255.255.0.0 any
    access-list 10 extended deny ip 224.0.0.0 224.0.0.0 any
    access-list 10 extended permit icmp any any echo-reply
    access-list 10 extended permit icmp any any time-exceeded
    access-list 10 extended permit icmp any any unreachable
    access-list 10 extended permit icmp any any traceroute
    access-list 10 extended permit icmp any any source-quench
    access-list 10 extended permit icmp any any
    access-list 10 extended permit tcp object-group old hosting provider any eq 3389
    access-list 10 extended permit tcp any any eq https
    access-list 10 extended permit tcp any any eq www
    access-list 110 extended permit ip 172.19.5.64 255.255.255.192 172.19.3.0 255.255.255.128
    access-list 110 extended permit ip 172.19.5.64 255.255.255.192 object-group old hosting provider
    pager lines 24
    logging enable
    logging timestamp
    logging console emergencies
    logging monitor emergencies
    logging buffered warnings
    logging trap debugging
    logging history debugging
    logging asdm informational
    mtu inside 1500
    mtu outside 1500
    ip verify reverse-path interface inside
    ip verify reverse-path interface outside
    ip audit name jab attack action alarm drop reset
    ip audit name probe info action alarm drop reset
    ip audit interface outside probe
    ip audit interface outside jab
    ip audit info action alarm drop reset
    ip audit attack action alarm drop reset
    ip audit signature 2000 disable
    ip audit signature 2001 disable
    ip audit signature 2004 disable
    ip audit signature 2005 disable
    icmp unreachable rate-limit 1 burst-size 1
    icmp permit 75.150.169.48 255.255.255.240 outside
    icmp permit 72.44.134.16 255.255.255.240 outside
    icmp permit 72.55.33.0 255.255.255.0 outside
    icmp permit any outside
    icmp permit 173.163.157.32 255.255.255.240 outside
    icmp permit 107.0.197.0 255.255.255.192 outside
    icmp permit 66.11.1.64 255.255.255.192 outside
    icmp deny any outside
    asdm image disk0:/asdm-645.bin
    no asdm history enable
    arp timeout 14400
    global (outside) 1 interface
    nat (inside) 0 access-list 100
    nat (inside) 1 0.0.0.0 0.0.0.0
    access-group 10 in interface outside
    route outside 0.0.0.0 0.0.0.0 174.78.151.225 1
    timeout xlate 3:00:00
    timeout conn 24:00:00 half-closed 0:10:00 udp 0:10:00 icmp 0:00:02
    timeout sunrpc 0:10:00 h323 0:05:00 h225 1:00:00 mgcp 0:05:00 mgcp-pat 0:05:00
    timeout sip 24:00:00 sip_media 0:02:00 sip-invite 0:03:00 sip-disconnect 0:02:00
    timeout sip-provisional-media 0:02:00 uauth 0:05:00 absolute
    timeout tcp-proxy-reassembly 0:01:00
    timeout floating-conn 0:00:00
    dynamic-access-policy-record DfltAccessPolicy
    http 107.0.197.0 255.255.255.192 outside
    http 66.11.1.64 255.255.255.192 outside
    snmp-server host outside 107.0.197.29 community *****
    snmp-server host outside 107.0.197.30 community *****
    snmp-server host inside 172.19.250.10 community *****
    snmp-server host outside 172.19.250.10 community *****
    snmp-server host inside 172.19.250.11 community *****
    snmp-server host outside 172.19.250.11 community *****
    snmp-server host outside 68.82.122.239 community *****
    snmp-server host outside 72.55.33.37 community *****
    snmp-server host outside 72.55.33.38 community *****
    snmp-server host outside 75.150.169.50 community *****
    snmp-server host outside 75.150.169.51 community *****
    no snmp-server location
    no snmp-server contact
    snmp-server community *****
    snmp-server enable traps snmp authentication linkup linkdown coldstart
    crypto ipsec transform-set ESP-3DES-MD5 esp-3des esp-md5-hmac
    crypto ipsec security-association lifetime seconds 28800
    crypto ipsec security-association lifetime kilobytes 4608000
    crypto map VPNMAP 10 match address 110
    crypto map VPNMAP 10 set peer 72.00.00.7 old vpn public ip Site B Public IP
    crypto map VPNMAP 10 set transform-set ESP-3DES-MD5
    crypto map VPNMAP 10 set security-association lifetime seconds 86400
    crypto map VPNMAP 10 set security-association lifetime kilobytes 4608000
    crypto map VPNMAP interface outside
    crypto isakmp identity address
    crypto isakmp enable outside
    crypto isakmp policy 20
    authentication pre-share
    encryption 3des
    hash md5
    group 2
    lifetime 86400
    telnet 172.19.5.64 255.255.255.192 inside
    telnet 172.19.3.0 255.255.255.128 outside
    telnet timeout 60
    ssh 0.0.0.0 0.0.0.0 inside
    ssh 0.0.0.0 0.0.0.0 outside
    ssh timeout 60
    console timeout 0
    management-access inside
    dhcpd dns 172.19.3.140
    dhcpd wins 172.19.3.140
    dhcpd ping_timeout 750
    dhcpd domain iis-usa.com
    dhcpd address 172.19.5.80-172.19.5.111 inside
    dhcpd enable inside
    threat-detection basic-threat
    threat-detection scanning-threat shun except object-group old hosting provider
    threat-detection statistics
    threat-detection statistics tcp-intercept rate-interval 30 burst-rate 400 average-rate 200
    ntp server 128.118.25.3 source outside
    ntp server 217.150.242.8 source outside
    tunnel-group 72.00.00.7 type ipsec-l2l
    tunnel-group 72.00.00.7 ipsec-attributes
    pre-shared-key *****
    tunnel-group old vpn public ip type ipsec-l2l
    tunnel-group old vpn public ip ipsec-attributes
    pre-shared-key *****
    tunnel-group SITE A Public IP  type ipsec-l2l
    tunnel-group SITE A Public IP  ipsec-attributes
    pre-shared-key *****
    class-map inspection_default
    match default-inspection-traffic
    policy-map type inspect dns preset_dns_map
    parameters
      message-length maximum 512
    policy-map global_policy
    class inspection_default
      inspect dns preset_dns_map
      inspect ftp
      inspect h323 h225
      inspect h323 ras
      inspect rsh
      inspect rtsp
      inspect esmtp
      inspect sqlnet
      inspect skinny 
      inspect sunrpc
      inspect xdmcp
      inspect netbios
      inspect tftp
      inspect pptp
      inspect sip 
    service-policy global_policy global
    prompt hostname context
    no call-home reporting anonymous
    call-home
    profile CiscoTAC-1
      no active
      destination address http https://tools.cisco.com/its/service/oddce/services/DDCEService
      destination address email [email protected]
      destination transport-method http
      subscribe-to-alert-group diagnostic
      subscribe-to-alert-group environment
      subscribe-to-alert-group inventory periodic monthly
      subscribe-to-alert-group configuration periodic monthly
      subscribe-to-alert-group telemetry periodic daily
    Cryptochecksum:
    : end

    I have removed the old "set peer" and have added:
    IOS router:
    access-list 101 permit ip 172.19.3.128 0.0.0.127 172.19.5.64 0.0.0.65
    ASA fw:
    access-list 110 extended permit ip 172.19.5.64 255.255.255.192 172.19.3.128 255.255.255.128
    on the router I have also added;
    access-list 110 deny  ip 172.19.3.128 0.0.0.127 172.19.5.64 0.0.0.63
    Here is my acl :
    access-list 110 remark "Outbound NAT Rule"
    access-list 110 remark "Deny VPN Traffic NAT"
    access-list 110 deny   ip 172.19.3.128 0.0.0.127 172.19.3.0 0.0.0.127
    access-list 110 deny   ip 172.19.3.128 0.0.0.127 172.19.10.0 0.0.0.255
    access-list 110 deny   ip 172.19.10.0 0.0.0.255 172.19.3.128 0.0.0.127
    access-list 110 deny   ip 172.20.3.128 0.0.0.127 172.19.3.0 0.0.0.127
    access-list 110 deny   ip 172.19.3.128 0.0.0.127 172.20.3.128 0.0.0.127
    access-list 110 deny   ip 172.19.3.128 0.0.0.127 host 172.19.250.11
    access-list 110 deny   ip 172.19.3.128 0.0.0.127 host 172.19.250.10
    access-list 110 permit ip 172.19.3.128 0.0.0.127 any
    access-list 110 permit ip 172.19.10.0 0.0.0.255 any
    access-list 110 deny   ip 172.19.3.128 0.0.0.127 172.19.5.64 0.0.0.63
    access-list 198 remark "Networks for IISVPN Client"
    access-list 198 permit ip 172.19.3.0 0.0.0.127 172.20.3.128 0.0.0.127
    access-list 198 permit ip 172.19.3.128 0.0.0.127 172.20.3.128 0.0.0.127
    Still no ping tothe other site.

  • Cannot connect to ASDM on ASA 5505 over https

    Problem: Cannot connect to ASDM on ASA 5505 when vlan1 network is changed from the factory default.
    Hi all. I am just getting started on a new ASA 5505, working it in a test lab environment. I ran thru the initial setup wizard. During that time I specified a name for Vlan1 (changed from 'inside' to 'INTR-NET'), modified the Vlan1 IP address to use DHCP, and then populated the Device Config Access table with entries corresponding to the entire Class B network here on the local intranet. I don't recall if the factory-default network was already populated, but if it wasn't I added it as 192.168.1.0/255.255.255.0
    I then saved the config, and verified that the ASA got a dhcp address using the RS-232 console. I then reconfigured the laptop I have plugged into port 0/1 with it's normal address on the intranet and discovered that I couldn't reconnect to ASDM. The ASDM client times out, and a web browser opened to https://(ASA5505's dhcp addr) fails as well.
    I then used the console to add another http IP address matching the specific IP address (xxx.240.113.129/255.255.255.255) which the laptop is set for, to the list of permissible admin connections, but saw no difference.
    This issue is much the same as was reported in this prior forum posting:
    http://forums.cisco.com/eforum/servlet/NetProf?page=netprof&forum=Security&topic=General&topicID=.ee6e1f8&CommCmd=MB%3Fcmd%3Dpass_through%26location%3Doutline%40%5E1%40%40.2cc16cb8/4
    EXCEPT that I was already aware the admin IP address(es) needed to be registered to enable access via SSH/Telnet/HTTPS.
    And, I did that step, but it is not working. I have tried adding various combinations of network ranges in the device config access list, including the specific subnet that the lab's dhcp server assigned to the ASA 5505 (xxx.240.112.0/255.255.254.0), but there is no difference. I can traceroute to the laptop and ping the Vlan1 interface from the laptop, but the https ASDM (and ssh connections too) are not successful. This is very frustrating.
    The device is brand new, I see that upon boot it loads asa724-k8.bin, and the software banner says Cisco Adaptive Security Appliance Software Version 7.2(4)
    Note also that, from the RS-232 console, if I reset the IP address to the static, factory default (192.168.1.1) and manually config my laptop on the same subnet, then ASDM makes the connection. Just like out of the box. But when I put it back onto our intranet and verify the DHCP lease, then ASDM is a no go.
    Can you think of what I've missed?

    Good question. Let me add that info plus related Vlan config details:
    ASA5505A# show ip
    System IP Addresses:
    Interface Name IP address Subnet mask Method
    Vlan1 INTR-NET XXX.240.112.92 255.255.254.0 DHCP
    Vlan2 VoIP 172.26.99.1 255.255.255.0 manual
    Vlan3 dmz-unused 192.168.99.1 255.255.255.0 manual
    Current IP Addresses:
    Interface Name IP address Subnet mask Method
    Vlan1 INTR-NET XXX.240.112.92 255.255.254.0 DHCP
    Vlan2 VoIP 172.26.99.1 255.255.255.0 manual
    Vlan3 dmz-unused 192.168.99.1 255.255.255.0 manual
    ASA5505A# show switch vlan
    VLAN Name Status Ports
    1 INTR-NET up Et0/1, Et0/2, Et0/3, Et0/4
    2 VoIP down Et0/5, Et0/6, Et0/7
    3 dmz-unused down Et0/0
    ASA5505A#
    ASA5505A# config t
    ASA5505A(config)# show running-config http
    http server enable
    http XXX.240.0.0 255.255.0.0 INTR-NET
    http 192.168.1.0 255.255.255.0 INTR-NET
    http XXX.240.113.129 255.255.255.255 INTR-NET
    ASA5505A(config)#
    ASA5505A(config)# show running-config ssh
    ssh 192.168.1.0 255.255.255.0 INTR-NET
    ssh XXX.240.0.0 255.255.0.0 INTR-NET
    ssh timeout 5
    SECURITY LEVEL IS 100 ON Vlan1 and Vlan2, 50 on Vlan3, and traffic is restricted from Vlan3 to Vlan1 because this is the basic license.

  • ASA 5505 unable to connect inside or outside

    Hello,
    I'm extremely new to router configurations, and am attempting to configure a backup ASA 5505 to use as a temporary access point in the event that our main ASA becomes unavailable. What I have done is loaded the running config from our main ASA onto the backup, and have made changes to necessary routes, IPs, etc. I can connect to it from a remote computer without problem, but I cannot access any of our servers, nor can I access the internet. I have also tried modifying the access list and NAT rules every which way from Sunday, but I still cannot get this thing to allow any information through. I keep getting "failed to locate egress interface for UDP from outside" errors.
    We are using Cisco AnyConnect to connect , and mind you, since the config for this backup ASA was taken from our main, it still has the original certificate info and profiles. I was told that this wouldn't matter, but I thought I should mention in case I need to remove any of it from the config.
    Here is part of the config file. I took out some information, but tried to keep it understandable. If anyone could point me in the right direction, it would be greatly appreciated!
    ciscoasa# show running-config
    : Saved
    : Serial Number: xxxxxxxxxxx
    : Hardware:   ASA5505, 512 MB RAM, CPU Geode 500 MHz
    ASA Version 9.2(2)
    hostname ciscoasa
    domain-name domain
    enable password encrypted
    xlate per-session deny tcp any4 any4
    xlate per-session deny tcp any4 any6
    xlate per-session deny tcp any6 any4
    xlate per-session deny tcp any6 any6
    xlate per-session deny udp any4 any4 eq domain
    xlate per-session deny udp any4 any6 eq domain
    xlate per-session deny udp any6 any4 eq domain
    xlate per-session deny udp any6 any6 eq domain
    passwd encrypted
    names
    ip local pool pool1 x.x.9.22-x.x.9.254 mask 255.255.255.0
    interface Ethernet0/0
     switchport access vlan 2
    interface Ethernet0/1
    interface Ethernet0/2
    interface Ethernet0/3
    interface Ethernet0/4
    interface Ethernet0/5
    interface Ethernet0/6
    interface Ethernet0/7
    interface Vlan1
     nameif inside
     security-level 100
     ip address x.x.8.10 255.255.255.0
    interface Vlan2
     nameif outside
     security-level 0
     ip address x.x.x.237 255.255.255.248
    boot system disk0:/asa922-k8.bin
    boot config disk0:/startup-config
    ftp mode passive
    clock timezone EST -5
    clock summer-time EDT recurring
    dns domain-lookup inside
    dns domain-lookup outside
    dns server-group Default
     name-server x.x.8.100
     domain-name domain
    same-security-traffic permit intra-interface
    object network obj_any
     subnet 0.0.0.0 0.0.0.0
    object network pool1
     subnet x.x.9.0 255.255.255.0
    object network outside-network
     host x.x.x.237
    object network Remote-Network
     subnet x.x.8.0 255.255.255.0
    object network local
    object network obj-x.x.9.24
     host x.x.9.24
    object-group network Outside-Network-Group
     description Outside Network Group
     network-object x.x.x.232 255.255.255.248
    object-group network Inside-Network-Group
     description Inside Network Group
     network-object x.x.8.0 255.255.255.0
    access-list inside_access_in extended permit icmp any any
    access-list inside_access_in extended permit ip any any
    access-list NONAT extended permit ip x.x.8.0 255.255.255.0 x.x.9.0 255.255.255.0
    pager lines 24
    logging enable
    logging buffer-size 30000
    logging buffered debugging
    logging asdm informational
    no logging message 106015
    no logging message 313001
    no logging message 313008
    no logging message 106023
    no logging message 710003
    no logging message 106100
    no logging message 302015
    no logging message 302014
    no logging message 302013
    no logging message 302018
    no logging message 302017
    no logging message 302016
    no logging message 302021
    no logging message 302020
    flow-export destination inside x.x.8.132 2055
    flow-export template timeout-rate 1
    flow-export delay flow-create 50
    mtu inside 1500
    mtu outside 1500
    icmp unreachable rate-limit 1 burst-size 1
    icmp permit any outside
    asdm image disk0:/asdm-722.bin
    no asdm history enable
    arp timeout 14400
    no arp permit-nonconnected
    nat (inside,outside) source static any any destination static pool1 pool1 no-proxy-arp route-lookup
    nat (inside,outside) source static any any destination static Remote-Network Remote-Network no-proxy-arp route-lookup
    nat (outside,outside) source dynamic pool1 interface
    object network obj_any
     nat (inside,outside) dynamic interface
    access-group inside_access_in in interface inside
    route outside 0.0.0.0 0.0.0.0 x.x.x.232 1
    route inside x.x.11.0 255.255.255.0 x.x.11.1 1
    If you have any questions, or need any other information, please let me know.
    Thanks!

    Am I posting this in the wrong section? Anyone?

  • Problem with Remote Access VPN on ASA 5505

    I am currently having an issue configuring an ASA 5505 to connect via remote access VPN using the Cisco VPN Client 5.0.07.0440 running on Windows 8 Pro x64. The VPN client prompts for the username and password during the connect process, but fails soon after.
    The VPN client logs are as follows:
    Cisco Systems VPN Client Version 5.0.07.0440
    Copyright (C) 1998-2010 Cisco Systems, Inc. All Rights Reserved.
    Client Type(s): Windows, WinNT
    Running on: 6.2.9200
    2      15:09:21.240  12/11/12  Sev=Info/4    CM/0x63100002
    Begin connection process
    3      15:09:21.287  12/11/12  Sev=Info/4    CM/0x63100004
    Establish secure connection
    4      15:09:21.287  12/11/12  Sev=Info/4    CM/0x63100024
    Attempt connection with server "**.**.***.***"
    5      15:09:21.287  12/11/12  Sev=Info/6    IKE/0x6300003B
    Attempting to establish a connection with **.**.***.***.
    6      15:09:21.287  12/11/12  Sev=Info/4    IKE/0x63000001
    Starting IKE Phase 1 Negotiation
    7      15:09:21.303  12/11/12  Sev=Info/4    IKE/0x63000013
    SENDING >>> ISAKMP OAK AG (SA, KE, NON, ID, VID(Xauth), VID(dpd), VID(Frag), VID(Nat-T), VID(Unity)) to **.**.***.***
    8      15:09:21.365  12/11/12  Sev=Info/6    GUI/0x63B00012
    Authentication request attributes is 6h.
    9      15:09:21.334  12/11/12  Sev=Info/5    IKE/0x6300002F
    Received ISAKMP packet: peer = **.**.***.***
    10     15:09:21.334  12/11/12  Sev=Info/4    IKE/0x63000014
    RECEIVING <<< ISAKMP OAK AG (SA, KE, NON, ID, HASH, VID(Unity), VID(Xauth), VID(dpd), VID(Nat-T), NAT-D, NAT-D, VID(Frag), VID(?)) from **.**.***.***
    11     15:09:21.334  12/11/12  Sev=Info/5    IKE/0x63000001
    Peer is a Cisco-Unity compliant peer
    12     15:09:21.334  12/11/12  Sev=Info/5    IKE/0x63000001
    Peer supports XAUTH
    13     15:09:21.334  12/11/12  Sev=Info/5    IKE/0x63000001
    Peer supports DPD
    14     15:09:21.334  12/11/12  Sev=Info/5    IKE/0x63000001
    Peer supports NAT-T
    15     15:09:21.334  12/11/12  Sev=Info/5    IKE/0x63000001
    Peer supports IKE fragmentation payloads
    16     15:09:21.334  12/11/12  Sev=Info/6    IKE/0x63000001
    IOS Vendor ID Contruction successful
    17     15:09:21.334  12/11/12  Sev=Info/4    IKE/0x63000013
    SENDING >>> ISAKMP OAK AG *(HASH, NOTIFY:STATUS_INITIAL_CONTACT, NAT-D, NAT-D, VID(?), VID(Unity)) to **.**.***.***
    18     15:09:21.334  12/11/12  Sev=Info/6    IKE/0x63000055
    Sent a keepalive on the IPSec SA
    19     15:09:21.334  12/11/12  Sev=Info/4    IKE/0x63000083
    IKE Port in use - Local Port =  0xFBCE, Remote Port = 0x1194
    20     15:09:21.334  12/11/12  Sev=Info/5    IKE/0x63000072
    Automatic NAT Detection Status:
       Remote end is NOT behind a NAT device
       This   end IS behind a NAT device
    21     15:09:21.334  12/11/12  Sev=Info/4    CM/0x6310000E
    Established Phase 1 SA.  1 Crypto Active IKE SA, 0 User Authenticated IKE SA in the system
    22     15:09:21.365  12/11/12  Sev=Info/5    IKE/0x6300002F
    Received ISAKMP packet: peer = **.**.***.***
    23     15:09:21.365  12/11/12  Sev=Info/4    IKE/0x63000014
    RECEIVING <<< ISAKMP OAK TRANS *(HASH, ATTR) from **.**.***.***
    24     15:09:21.365  12/11/12  Sev=Info/4    CM/0x63100015
    Launch xAuth application
    25     15:09:21.474  12/11/12  Sev=Info/4    IPSEC/0x63700008
    IPSec driver successfully started
    26     15:09:21.474  12/11/12  Sev=Info/4    IPSEC/0x63700014
    Deleted all keys
    27     15:09:27.319  12/11/12  Sev=Info/4    CM/0x63100017
    xAuth application returned
    28     15:09:27.319  12/11/12  Sev=Info/4    IKE/0x63000013
    SENDING >>> ISAKMP OAK TRANS *(HASH, ATTR) to **.**.***.***
    29     15:09:27.365  12/11/12  Sev=Info/5    IKE/0x6300002F
    Received ISAKMP packet: peer = **.**.***.***
    30     15:09:27.365  12/11/12  Sev=Info/4    IKE/0x63000014
    RECEIVING <<< ISAKMP OAK TRANS *(HASH, ATTR) from **.**.***.***
    31     15:09:27.365  12/11/12  Sev=Info/4    IKE/0x63000013
    SENDING >>> ISAKMP OAK TRANS *(HASH, ATTR) to **.**.***.***
    32     15:09:27.365  12/11/12  Sev=Info/4    CM/0x6310000E
    Established Phase 1 SA.  1 Crypto Active IKE SA, 1 User Authenticated IKE SA in the system
    33     15:09:27.365  12/11/12  Sev=Info/5    IKE/0x6300005E
    Client sending a firewall request to concentrator
    34     15:09:27.365  12/11/12  Sev=Info/4    IKE/0x63000013
    SENDING >>> ISAKMP OAK TRANS *(HASH, ATTR) to **.**.***.***
    35     15:09:27.397  12/11/12  Sev=Info/5    IKE/0x6300002F
    Received ISAKMP packet: peer = **.**.***.***
    36     15:09:27.397  12/11/12  Sev=Info/4    IKE/0x63000014
    RECEIVING <<< ISAKMP OAK TRANS *(HASH, ATTR) from **.**.***.***
    37     15:09:27.397  12/11/12  Sev=Info/5    IKE/0x63000010
    MODE_CFG_REPLY: Attribute = INTERNAL_IPV4_ADDRESS: , value = 192.168.2.70
    38     15:09:27.397  12/11/12  Sev=Info/5    IKE/0x63000010
    MODE_CFG_REPLY: Attribute = INTERNAL_IPV4_NETMASK: , value = 255.255.255.0
    39     15:09:27.397  12/11/12  Sev=Info/5    IKE/0x63000010
    MODE_CFG_REPLY: Attribute = INTERNAL_IPV4_DNS(1): , value = 192.168.2.1
    40     15:09:27.397  12/11/12  Sev=Info/5    IKE/0x63000010
    MODE_CFG_REPLY: Attribute = INTERNAL_IPV4_DNS(2): , value = 8.8.8.8
    41     15:09:27.397  12/11/12  Sev=Info/5    IKE/0x6300000D
    MODE_CFG_REPLY: Attribute = MODECFG_UNITY_SAVEPWD: , value = 0x00000001
    42     15:09:27.397  12/11/12  Sev=Info/5    IKE/0x6300000E
    MODE_CFG_REPLY: Attribute = MODECFG_UNITY_DEFDOMAIN: , value = NCHCO
    43     15:09:27.397  12/11/12  Sev=Info/5    IKE/0x6300000D
    MODE_CFG_REPLY: Attribute = MODECFG_UNITY_PFS: , value = 0x00000000
    44     15:09:27.397  12/11/12  Sev=Info/5    IKE/0x6300000E
    MODE_CFG_REPLY: Attribute = APPLICATION_VERSION, value = Cisco Systems, Inc ASA5505 Version 8.2(5) built by builders on Fri 20-May-11 16:00
    45     15:09:27.397  12/11/12  Sev=Info/5    IKE/0x6300000D
    MODE_CFG_REPLY: Attribute = MODECFG_UNITY_SMARTCARD_REMOVAL_DISCONNECT: , value = 0x00000001
    46     15:09:27.397  12/11/12  Sev=Info/5    IKE/0x6300000D
    MODE_CFG_REPLY: Attribute = Received and using NAT-T port number , value = 0x00001194
    47     15:09:27.397  12/11/12  Sev=Info/4    CM/0x63100019
    Mode Config data received
    48     15:09:27.412  12/11/12  Sev=Info/4    IKE/0x63000056
    Received a key request from Driver: Local IP = 192.168.2.70, GW IP = **.**.***.***, Remote IP = 0.0.0.0
    49     15:09:27.412  12/11/12  Sev=Info/4    IKE/0x63000013
    SENDING >>> ISAKMP OAK QM *(HASH, SA, NON, ID, ID) to **.**.***.***
    50     15:09:27.444  12/11/12  Sev=Info/5    IKE/0x6300002F
    Received ISAKMP packet: peer = **.**.***.***
    51     15:09:27.444  12/11/12  Sev=Info/4    IKE/0x63000014
    RECEIVING <<< ISAKMP OAK INFO *(HASH, NOTIFY:STATUS_RESP_LIFETIME) from **.**.***.***
    52     15:09:27.444  12/11/12  Sev=Info/5    IKE/0x63000045
    RESPONDER-LIFETIME notify has value of 86400 seconds
    53     15:09:27.444  12/11/12  Sev=Info/5    IKE/0x63000047
    This SA has already been alive for 6 seconds, setting expiry to 86394 seconds from now
    54     15:09:27.459  12/11/12  Sev=Info/5    IKE/0x6300002F
    Received ISAKMP packet: peer = **.**.***.***
    55     15:09:27.459  12/11/12  Sev=Info/4    IKE/0x63000014
    RECEIVING <<< ISAKMP OAK INFO *(HASH, NOTIFY:NO_PROPOSAL_CHOSEN) from **.**.***.***
    56     15:09:27.459  12/11/12  Sev=Info/4    IKE/0x63000013
    SENDING >>> ISAKMP OAK INFO *(HASH, DEL) to **.**.***.***
    57     15:09:27.459  12/11/12  Sev=Info/4    IKE/0x63000049
    Discarding IPsec SA negotiation, MsgID=CE99A8A8
    58     15:09:27.459  12/11/12  Sev=Info/4    IKE/0x63000017
    Marking IKE SA for deletion  (I_Cookie=A3A341F1C7606AD5 R_Cookie=F1F403018625E924) reason = DEL_REASON_IKE_NEG_FAILED
    59     15:09:27.459  12/11/12  Sev=Info/5    IKE/0x6300002F
    Received ISAKMP packet: peer = **.**.***.***
    60     15:09:27.459  12/11/12  Sev=Info/4    IKE/0x63000058
    Received an ISAKMP message for a non-active SA, I_Cookie=A3A341F1C7606AD5 R_Cookie=F1F403018625E924
    61     15:09:27.459  12/11/12  Sev=Info/4    IKE/0x63000014
    RECEIVING <<< ISAKMP OAK INFO *(Dropped) from **.**.***.***
    62     15:09:27.490  12/11/12  Sev=Info/4    IPSEC/0x63700014
    Deleted all keys
    63     15:09:30.475  12/11/12  Sev=Info/4    IKE/0x6300004B
    Discarding IKE SA negotiation (I_Cookie=A3A341F1C7606AD5 R_Cookie=F1F403018625E924) reason = DEL_REASON_IKE_NEG_FAILED
    64     15:09:30.475  12/11/12  Sev=Info/4    CM/0x63100012
    Phase 1 SA deleted before first Phase 2 SA is up cause by "DEL_REASON_IKE_NEG_FAILED".  0 Crypto Active IKE SA, 0 User Authenticated IKE SA in the system
    65     15:09:30.475  12/11/12  Sev=Info/5    CM/0x63100025
    Initializing CVPNDrv
    66     15:09:30.475  12/11/12  Sev=Info/6    CM/0x63100046
    Set tunnel established flag in registry to 0.
    67     15:09:30.475  12/11/12  Sev=Info/4    IKE/0x63000001
    IKE received signal to terminate VPN connection
    68     15:09:30.475  12/11/12  Sev=Info/4    IPSEC/0x63700014
    Deleted all keys
    69     15:09:30.475  12/11/12  Sev=Info/4    IPSEC/0x63700014
    Deleted all keys
    70     15:09:30.475  12/11/12  Sev=Info/4    IPSEC/0x63700014
    Deleted all keys
    71     15:09:30.475  12/11/12  Sev=Info/4    IPSEC/0x6370000A
    IPSec driver successfully stopped
    The running configuration is as follows (there is a site-to-site VPN set up as well to another ASA 5505, but that is working flawlessly):
    : Saved
    ASA Version 8.2(5)
    hostname NCHCO
    enable password hTjwXz/V8EuTw9p9 encrypted
    passwd hTjwXz/V8EuTw9p9 encrypted
    names
    name 192.168.2.0 NCHCO description City Offices
    name 192.168.2.80 VPN_End
    name 192.168.2.70 VPN_Start
    interface Ethernet0/0
    switchport access vlan 2
    speed 100
    duplex full
    interface Ethernet0/1
    interface Ethernet0/2
    interface Ethernet0/3
    interface Ethernet0/4
    interface Ethernet0/5
    interface Ethernet0/6
    interface Ethernet0/7
    interface Vlan1
    nameif inside
    security-level 100
    ip address 192.168.2.1 255.255.255.0
    interface Vlan2
    nameif outside
    security-level 0
    ip address **.**.***.*** 255.255.255.248
    boot system disk0:/asa825-k8.bin
    ftp mode passive
    access-list outside_nat0_outbound extended permit ip NCHCO 255.255.255.0 192.168.1.0 255.255.255.0
    access-list inside_nat0_outbound extended permit ip NCHCO 255.255.255.0 192.168.1.0 255.255.255.0
    access-list inside_nat0_outbound extended permit ip any 192.168.2.64 255.255.255.224
    access-list outside_1_cryptomap extended permit ip NCHCO 255.255.255.0 192.168.1.0 255.255.255.0
    access-list outside_1_cryptomap_1 extended permit ip NCHCO 255.255.255.0 192.168.1.0 255.255.255.0
    access-list LAN_Access standard permit NCHCO 255.255.255.0
    access-list LAN_Access standard permit 0.0.0.0 255.255.255.0
    pager lines 24
    logging enable
    logging asdm informational
    mtu inside 1500
    mtu outside 1500
    ip local pool VPN_Pool VPN_Start-VPN_End mask 255.255.255.0
    icmp unreachable rate-limit 1 burst-size 1
    asdm image disk0:/asdm-645.bin
    no asdm history enable
    arp timeout 14400
    global (outside) 1 interface
    nat (inside) 0 access-list inside_nat0_outbound
    nat (inside) 1 0.0.0.0 0.0.0.0
    nat (outside) 0 access-list outside_nat0_outbound
    route outside 0.0.0.0 0.0.0.0 74.219.208.49 1
    timeout xlate 3:00:00
    timeout conn 1:00:00 half-closed 0:10:00 udp 0:02:00 icmp 0:00:02
    timeout sunrpc 0:10:00 h323 0:05:00 h225 1:00:00 mgcp 0:05:00 mgcp-pat 0:05:00
    timeout sip 0:30:00 sip_media 0:02:00 sip-invite 0:03:00 sip-disconnect 0:02:00
    timeout sip-provisional-media 0:02:00 uauth 0:05:00 absolute
    timeout tcp-proxy-reassembly 0:01:00
    timeout floating-conn 0:00:00
    dynamic-access-policy-record DfltAccessPolicy
    network-acl outside_nat0_outbound
    webvpn
      svc ask enable default svc
    http server enable
    http 192.168.1.0 255.255.255.0 inside
    http **.**.***.*** 255.255.255.255 outside
    http 74.218.158.238 255.255.255.255 outside
    http NCHCO 255.255.255.0 inside
    no snmp-server location
    no snmp-server contact
    snmp-server enable traps snmp authentication linkup linkdown coldstart
    crypto ipsec transform-set ESP-AES-128-SHA esp-aes esp-sha-hmac
    crypto ipsec transform-set ESP-AES-128-MD5 esp-aes esp-md5-hmac
    crypto ipsec transform-set ESP-AES-192-SHA esp-aes-192 esp-sha-hmac
    crypto ipsec transform-set ESP-AES-192-MD5 esp-aes-192 esp-md5-hmac
    crypto ipsec transform-set ESP-AES-256-SHA esp-aes-256 esp-sha-hmac
    crypto ipsec transform-set ESP-AES-256-MD5 esp-aes-256 esp-md5-hmac
    crypto ipsec transform-set ESP-3DES-MD5 esp-3des esp-md5-hmac
    crypto ipsec transform-set ESP-DES-SHA esp-des esp-sha-hmac
    crypto ipsec transform-set ESP-DES-MD5 esp-des esp-md5-hmac
    crypto ipsec transform-set l2tp-transform esp-3des esp-sha-hmac
    crypto ipsec transform-set l2tp-transform mode transport
    crypto ipsec transform-set vpn-transform esp-aes-256 esp-sha-hmac
    crypto ipsec transform-set TRANS_ESP_3DES_SHA esp-3des esp-sha-hmac
    crypto ipsec transform-set TRANS_ESP_3DES_SHA mode transport
    crypto ipsec transform-set TRANS_ESP_3DES_MD5 esp-3des esp-md5-hmac
    crypto ipsec transform-set TRANS_ESP_3DES_MD5 mode transport
    crypto ipsec transform-set ESP-3DES-SHA esp-3des esp-sha-hmac
    crypto ipsec security-association lifetime seconds 28800
    crypto ipsec security-association lifetime kilobytes 4608000
    crypto dynamic-map SYSTEM_DEFAULT_CRYPTO_MAP 65535 set pfs group1
    crypto dynamic-map SYSTEM_DEFAULT_CRYPTO_MAP 65535 set transform-set ESP-AES-128-SHA ESP-AES-128-MD5 ESP-AES-192-SHA ESP-AES-192-MD5 ESP-AES-256-SHA ESP-AES-256-MD5 ESP-3DES-SHA ESP-3DES-MD5 ESP-DES-SHA ESP-DES-MD5
    crypto dynamic-map dyn-map 10 set pfs group1
    crypto dynamic-map dyn-map 10 set transform-set l2tp-transform vpn-transform
    crypto dynamic-map dyn-map 10 set reverse-route
    crypto dynamic-map outside_dyn_map 20 set transform-set TRANS_ESP_3DES_MD5
    crypto map outside_map 1 match address outside_1_cryptomap
    crypto map outside_map 1 set pfs group1
    crypto map outside_map 1 set peer 74.219.208.50
    crypto map outside_map 1 set transform-set ESP-3DES-SHA
    crypto map outside_map 20 ipsec-isakmp dynamic outside_dyn_map
    crypto map outside_map 65535 ipsec-isakmp dynamic SYSTEM_DEFAULT_CRYPTO_MAP
    crypto map outside_map interface outside
    crypto map inside_map 65535 ipsec-isakmp dynamic SYSTEM_DEFAULT_CRYPTO_MAP
    crypto map inside_map interface inside
    crypto map vpn-map 1 match address outside_1_cryptomap_1
    crypto map vpn-map 1 set pfs group1
    crypto map vpn-map 1 set peer 74.219.208.50
    crypto map vpn-map 1 set transform-set ESP-3DES-SHA
    crypto map vpn-map 10 ipsec-isakmp dynamic dyn-map
    crypto isakmp identity address
    crypto isakmp enable inside
    crypto isakmp enable outside
    crypto isakmp policy 10
    authentication pre-share
    encryption 3des
    hash md5
    group 2
    lifetime 86400
    crypto isakmp policy 15
    authentication pre-share
    encryption aes-256
    hash sha
    group 2
    lifetime 86400
    crypto isakmp policy 35
    authentication pre-share
    encryption 3des
    hash sha
    group 2
    lifetime 86400
    crypto isakmp ipsec-over-tcp port 10000
    client-update enable
    telnet 192.168.1.0 255.255.255.0 inside
    telnet NCHCO 255.255.255.0 inside
    telnet timeout 5
    ssh 192.168.1.0 255.255.255.0 inside
    ssh NCHCO 255.255.255.0 inside
    ssh timeout 5
    console timeout 0
    dhcpd address 192.168.2.150-192.168.2.225 inside
    dhcpd dns 216.68.4.10 216.68.5.10 interface inside
    dhcpd lease 64000 interface inside
    threat-detection basic-threat
    threat-detection statistics access-list
    no threat-detection statistics tcp-intercept
    webvpn
    group-policy DefaultRAGroup internal
    group-policy DefaultRAGroup attributes
    dns-server value 192.168.2.1
    vpn-tunnel-protocol IPSec l2tp-ipsec
    default-domain value nchco.local
    group-policy DfltGrpPolicy attributes
    dns-server value 192.168.2.1
    vpn-tunnel-protocol IPSec l2tp-ipsec svc webvpn
    password-storage enable
    ipsec-udp enable
    intercept-dhcp 255.255.255.0 enable
    address-pools value VPN_Pool
    group-policy NCHVPN internal
    group-policy NCHVPN attributes
    dns-server value 192.168.2.1 8.8.8.8
    vpn-tunnel-protocol IPSec l2tp-ipsec
    default-domain value NCHCO
    username admin password LbMiJuAJjDaFb2uw encrypted privilege 15
    username 8njferg password yB1lHEVmHZGj5C2Z encrypted privilege 15
    username NCHvpn99 password QhZZtJfwbnowceB7 encrypted
    tunnel-group DefaultRAGroup general-attributes
    address-pool (inside) VPN_Pool
    address-pool VPN_Pool
    authentication-server-group (inside) LOCAL
    authentication-server-group (outside) LOCAL
    authorization-server-group LOCAL
    authorization-server-group (inside) LOCAL
    authorization-server-group (outside) LOCAL
    default-group-policy DefaultRAGroup
    strip-realm
    strip-group
    tunnel-group DefaultRAGroup ipsec-attributes
    pre-shared-key *****
    peer-id-validate nocheck
    tunnel-group DefaultRAGroup ppp-attributes
    no authentication chap
    no authentication ms-chap-v1
    authentication ms-chap-v2
    tunnel-group DefaultWEBVPNGroup ppp-attributes
    authentication pap
    authentication ms-chap-v2
    tunnel-group 74.219.208.50 type ipsec-l2l
    tunnel-group 74.219.208.50 ipsec-attributes
    pre-shared-key *****
    tunnel-group NCHVPN type remote-access
    tunnel-group NCHVPN general-attributes
    address-pool VPN_Pool
    default-group-policy NCHVPN
    tunnel-group NCHVPN ipsec-attributes
    pre-shared-key *****
    class-map inspection_default
    match default-inspection-traffic
    policy-map type inspect dns preset_dns_map
    parameters
      message-length maximum client auto
      message-length maximum 512
    policy-map global_policy
    class inspection_default
      inspect dns preset_dns_map
      inspect ftp
      inspect h323 h225
      inspect h323 ras
      inspect rsh
      inspect rtsp
      inspect esmtp
      inspect sqlnet
      inspect skinny
      inspect sunrpc
      inspect xdmcp
      inspect sip
      inspect netbios
      inspect tftp
      inspect ip-options
    service-policy global_policy global
    prompt hostname context
    no call-home reporting anonymous
    Cryptochecksum:15852745977ff159ba808c4a4feb61fa
    : end
    asdm image disk0:/asdm-645.bin
    asdm location VPN_Start 255.255.255.255 inside
    asdm location VPN_End 255.255.255.255 inside
    no asdm history enable
    Anyone have any idea why this is happening?
    Thanks!

    Thanks again for your reply, and sorry about the late response, havent gotten back to this issue until just now. I applied the above command as you specified, and unfortunately, it did not resolve the problem. Below are the logs from the VPN Client for the connection + attempted browsing of a network share that is behind the ASA, and the new running configuration.
    VPN Client Log:
    Cisco Systems VPN Client Version 5.0.07.0440
    Copyright (C) 1998-2010 Cisco Systems, Inc. All Rights Reserved.
    Client Type(s): Windows, WinNT
    Running on: 6.2.9200
    331    13:11:41.362  12/17/12  Sev=Info/4    CM/0x63100002
    Begin connection process
    332    13:11:41.362  12/17/12  Sev=Info/4    CM/0x63100004
    Establish secure connection
    333    13:11:41.362  12/17/12  Sev=Info/4    CM/0x63100024
    Attempt connection with server "69.61.228.178"
    334    13:11:41.362  12/17/12  Sev=Info/6    IKE/0x6300003B
    Attempting to establish a connection with 69.61.228.178.
    335    13:11:41.362  12/17/12  Sev=Info/4    IKE/0x63000001
    Starting IKE Phase 1 Negotiation
    336    13:11:41.424  12/17/12  Sev=Info/6    GUI/0x63B00012
    Authentication request attributes is 6h.
    337    13:11:41.362  12/17/12  Sev=Info/4    IKE/0x63000013
    SENDING >>> ISAKMP OAK AG (SA, KE, NON, ID, VID(Xauth), VID(dpd), VID(Frag), VID(Nat-T), VID(Unity)) to 69.61.228.178
    338    13:11:41.393  12/17/12  Sev=Info/5    IKE/0x6300002F
    Received ISAKMP packet: peer = 69.61.228.178
    339    13:11:41.393  12/17/12  Sev=Info/4    IKE/0x63000014
    RECEIVING <<< ISAKMP OAK AG (SA, KE, NON, ID, HASH, VID(Unity), VID(Xauth), VID(dpd), VID(Nat-T), NAT-D, NAT-D, VID(Frag), VID(?)) from 69.61.228.178
    340    13:11:41.393  12/17/12  Sev=Info/5    IKE/0x63000001
    Peer is a Cisco-Unity compliant peer
    341    13:11:41.393  12/17/12  Sev=Info/5    IKE/0x63000001
    Peer supports XAUTH
    342    13:11:41.393  12/17/12  Sev=Info/5    IKE/0x63000001
    Peer supports DPD
    343    13:11:41.393  12/17/12  Sev=Info/5    IKE/0x63000001
    Peer supports NAT-T
    344    13:11:41.393  12/17/12  Sev=Info/5    IKE/0x63000001
    Peer supports IKE fragmentation payloads
    345    13:11:41.393  12/17/12  Sev=Info/6    IKE/0x63000001
    IOS Vendor ID Contruction successful
    346    13:11:41.393  12/17/12  Sev=Info/4    IKE/0x63000013
    SENDING >>> ISAKMP OAK AG *(HASH, NOTIFY:STATUS_INITIAL_CONTACT, NAT-D, NAT-D, VID(?), VID(Unity)) to 69.61.228.178
    347    13:11:41.393  12/17/12  Sev=Info/6    IKE/0x63000055
    Sent a keepalive on the IPSec SA
    348    13:11:41.393  12/17/12  Sev=Info/4    IKE/0x63000083
    IKE Port in use - Local Port =  0xD271, Remote Port = 0x1194
    349    13:11:41.393  12/17/12  Sev=Info/5    IKE/0x63000072
    Automatic NAT Detection Status:
       Remote end is NOT behind a NAT device
       This   end IS behind a NAT device
    350    13:11:41.393  12/17/12  Sev=Info/4    CM/0x6310000E
    Established Phase 1 SA.  1 Crypto Active IKE SA, 0 User Authenticated IKE SA in the system
    351    13:11:41.424  12/17/12  Sev=Info/5    IKE/0x6300002F
    Received ISAKMP packet: peer = 69.61.228.178
    352    13:11:41.424  12/17/12  Sev=Info/4    IKE/0x63000014
    RECEIVING <<< ISAKMP OAK TRANS *(HASH, ATTR) from 69.61.228.178
    353    13:11:41.424  12/17/12  Sev=Info/4    CM/0x63100015
    Launch xAuth application
    354    13:11:41.424  12/17/12  Sev=Info/4    CM/0x63100017
    xAuth application returned
    355    13:11:41.424  12/17/12  Sev=Info/4    IKE/0x63000013
    SENDING >>> ISAKMP OAK TRANS *(HASH, ATTR) to 69.61.228.178
    356    13:11:41.456  12/17/12  Sev=Info/5    IKE/0x6300002F
    Received ISAKMP packet: peer = 69.61.228.178
    357    13:11:41.456  12/17/12  Sev=Info/4    IKE/0x63000014
    RECEIVING <<< ISAKMP OAK TRANS *(HASH, ATTR) from 69.61.228.178
    358    13:11:41.456  12/17/12  Sev=Info/4    IKE/0x63000013
    SENDING >>> ISAKMP OAK TRANS *(HASH, ATTR) to 69.61.228.178
    359    13:11:41.456  12/17/12  Sev=Info/4    CM/0x6310000E
    Established Phase 1 SA.  1 Crypto Active IKE SA, 1 User Authenticated IKE SA in the system
    360    13:11:41.456  12/17/12  Sev=Info/5    IKE/0x6300005E
    Client sending a firewall request to concentrator
    361    13:11:41.456  12/17/12  Sev=Info/4    IKE/0x63000013
    SENDING >>> ISAKMP OAK TRANS *(HASH, ATTR) to 69.61.228.178
    362    13:11:41.502  12/17/12  Sev=Info/5    IKE/0x6300002F
    Received ISAKMP packet: peer = 69.61.228.178
    363    13:11:41.502  12/17/12  Sev=Info/4    IKE/0x63000014
    RECEIVING <<< ISAKMP OAK TRANS *(HASH, ATTR) from 69.61.228.178
    364    13:11:41.502  12/17/12  Sev=Info/5    IKE/0x63000010
    MODE_CFG_REPLY: Attribute = INTERNAL_IPV4_ADDRESS: , value = 192.168.2.70
    365    13:11:41.502  12/17/12  Sev=Info/5    IKE/0x63000010
    MODE_CFG_REPLY: Attribute = INTERNAL_IPV4_NETMASK: , value = 255.255.255.0
    366    13:11:41.502  12/17/12  Sev=Info/5    IKE/0x63000010
    MODE_CFG_REPLY: Attribute = INTERNAL_IPV4_DNS(1): , value = 192.168.2.1
    367    13:11:41.502  12/17/12  Sev=Info/5    IKE/0x63000010
    MODE_CFG_REPLY: Attribute = INTERNAL_IPV4_DNS(2): , value = 8.8.8.8
    368    13:11:41.502  12/17/12  Sev=Info/5    IKE/0x6300000D
    MODE_CFG_REPLY: Attribute = MODECFG_UNITY_SAVEPWD: , value = 0x00000001
    369    13:11:41.502  12/17/12  Sev=Info/5    IKE/0x6300000D
    MODE_CFG_REPLY: Attribute = MODECFG_UNITY_SPLIT_INCLUDE (# of split_nets), value = 0x00000001
    370    13:11:41.502  12/17/12  Sev=Info/5    IKE/0x6300000F
    SPLIT_NET #1
        subnet = 192.168.2.0
        mask = 255.255.255.0
        protocol = 0
        src port = 0
        dest port=0
    371    13:11:41.502  12/17/12  Sev=Info/5    IKE/0x6300000E
    MODE_CFG_REPLY: Attribute = MODECFG_UNITY_DEFDOMAIN: , value = NCHCO.local
    372    13:11:41.502  12/17/12  Sev=Info/5    IKE/0x6300000D
    MODE_CFG_REPLY: Attribute = MODECFG_UNITY_PFS: , value = 0x00000000
    373    13:11:41.502  12/17/12  Sev=Info/5    IKE/0x6300000E
    MODE_CFG_REPLY: Attribute = APPLICATION_VERSION, value = Cisco Systems, Inc ASA5505 Version 8.4(1) built by builders on Mon 31-Jan-11 02:11
    374    13:11:41.502  12/17/12  Sev=Info/5    IKE/0x6300000D
    MODE_CFG_REPLY: Attribute = MODECFG_UNITY_SMARTCARD_REMOVAL_DISCONNECT: , value = 0x00000001
    375    13:11:41.502  12/17/12  Sev=Info/5    IKE/0x6300000D
    MODE_CFG_REPLY: Attribute = Received and using NAT-T port number , value = 0x00001194
    376    13:11:41.502  12/17/12  Sev=Info/4    CM/0x63100019
    Mode Config data received
    377    13:11:41.502  12/17/12  Sev=Info/4    IKE/0x63000056
    Received a key request from Driver: Local IP = 192.168.2.70, GW IP = 69.61.228.178, Remote IP = 0.0.0.0
    378    13:11:41.502  12/17/12  Sev=Info/4    IKE/0x63000013
    SENDING >>> ISAKMP OAK QM *(HASH, SA, NON, ID, ID) to 69.61.228.178
    379    13:11:41.534  12/17/12  Sev=Info/5    IKE/0x6300002F
    Received ISAKMP packet: peer = 69.61.228.178
    380    13:11:41.534  12/17/12  Sev=Info/4    IKE/0x63000014
    RECEIVING <<< ISAKMP OAK INFO *(HASH, NOTIFY:STATUS_RESP_LIFETIME) from 69.61.228.178
    381    13:11:41.534  12/17/12  Sev=Info/5    IKE/0x63000045
    RESPONDER-LIFETIME notify has value of 86400 seconds
    382    13:11:41.534  12/17/12  Sev=Info/5    IKE/0x63000047
    This SA has already been alive for 0 seconds, setting expiry to 86400 seconds from now
    383    13:11:41.549  12/17/12  Sev=Info/5    IKE/0x6300002F
    Received ISAKMP packet: peer = 69.61.228.178
    384    13:11:41.549  12/17/12  Sev=Info/4    IKE/0x63000014
    RECEIVING <<< ISAKMP OAK QM *(HASH, SA, NON, ID, ID, NOTIFY:STATUS_RESP_LIFETIME) from 69.61.228.178
    385    13:11:41.549  12/17/12  Sev=Info/5    IKE/0x63000045
    RESPONDER-LIFETIME notify has value of 28800 seconds
    386    13:11:41.549  12/17/12  Sev=Info/4    IKE/0x63000013
    SENDING >>> ISAKMP OAK QM *(HASH) to 69.61.228.178
    387    13:11:41.549  12/17/12  Sev=Info/5    IKE/0x63000059
    Loading IPsec SA (MsgID=C4F5B5A6 OUTBOUND SPI = 0xD2DBADEA INBOUND SPI = 0x14762837)
    388    13:11:41.549  12/17/12  Sev=Info/5    IKE/0x63000025
    Loaded OUTBOUND ESP SPI: 0xD2DBADEA
    389    13:11:41.549  12/17/12  Sev=Info/5    IKE/0x63000026
    Loaded INBOUND ESP SPI: 0x14762837
    390    13:11:41.549  12/17/12  Sev=Info/5    CVPND/0x63400013
        Destination           Netmask           Gateway         Interface   Metric
            0.0.0.0           0.0.0.0       192.168.1.1     192.168.1.162       10
          127.0.0.0         255.0.0.0         127.0.0.1         127.0.0.1      306
          127.0.0.1   255.255.255.255         127.0.0.1         127.0.0.1      306
    127.255.255.255   255.255.255.255         127.0.0.1         127.0.0.1      306
        192.168.1.0     255.255.255.0     192.168.1.162     192.168.1.162      266
      192.168.1.162   255.255.255.255     192.168.1.162     192.168.1.162      266
      192.168.1.255   255.255.255.255     192.168.1.162     192.168.1.162      266
          224.0.0.0         240.0.0.0         127.0.0.1         127.0.0.1      306
          224.0.0.0         240.0.0.0     192.168.1.162     192.168.1.162      266
    255.255.255.255   255.255.255.255         127.0.0.1         127.0.0.1      306
    255.255.255.255   255.255.255.255     192.168.1.162     192.168.1.162      266
    391    13:11:41.877  12/17/12  Sev=Info/6    CVPND/0x63400001
    Launch VAInst64 to control IPSec Virtual Adapter
    392    13:11:43.455  12/17/12  Sev=Info/4    CM/0x63100034
    The Virtual Adapter was enabled:
        IP=192.168.2.70/255.255.255.0
        DNS=192.168.2.1,8.8.8.8
        WINS=0.0.0.0,0.0.0.0
        Domain=NCHCO.local
        Split DNS Names=
    393    13:11:43.455  12/17/12  Sev=Info/5    CVPND/0x63400013
        Destination           Netmask           Gateway         Interface   Metric
            0.0.0.0           0.0.0.0       192.168.1.1     192.168.1.162       10
          127.0.0.0         255.0.0.0         127.0.0.1         127.0.0.1      306
          127.0.0.1   255.255.255.255         127.0.0.1         127.0.0.1      306
    127.255.255.255   255.255.255.255         127.0.0.1         127.0.0.1      306
        192.168.1.0     255.255.255.0     192.168.1.162     192.168.1.162      266
      192.168.1.162   255.255.255.255     192.168.1.162     192.168.1.162      266
      192.168.1.255   255.255.255.255     192.168.1.162     192.168.1.162      266
          224.0.0.0         240.0.0.0         127.0.0.1         127.0.0.1      306
          224.0.0.0         240.0.0.0     192.168.1.162     192.168.1.162      266
          224.0.0.0         240.0.0.0           0.0.0.0           0.0.0.0      266
    255.255.255.255   255.255.255.255         127.0.0.1         127.0.0.1      306
    255.255.255.255   255.255.255.255     192.168.1.162     192.168.1.162      266
    255.255.255.255   255.255.255.255           0.0.0.0           0.0.0.0      266
    394    13:11:47.517  12/17/12  Sev=Info/4    CM/0x63100038
    Successfully saved route changes to file.
    395    13:11:47.517  12/17/12  Sev=Info/5    CVPND/0x63400013
        Destination           Netmask           Gateway         Interface   Metric
            0.0.0.0           0.0.0.0       192.168.1.1     192.168.1.162       10
      69.61.228.178   255.255.255.255       192.168.1.1     192.168.1.162      100
          127.0.0.0         255.0.0.0         127.0.0.1         127.0.0.1      306
          127.0.0.1   255.255.255.255         127.0.0.1         127.0.0.1      306
    127.255.255.255   255.255.255.255         127.0.0.1         127.0.0.1      306
        192.168.1.0     255.255.255.0     192.168.1.162     192.168.1.162      266
        192.168.1.2   255.255.255.255     192.168.1.162     192.168.1.162      100
      192.168.1.162   255.255.255.255     192.168.1.162     192.168.1.162      266
      192.168.1.255   255.255.255.255     192.168.1.162     192.168.1.162      266
        192.168.2.0     255.255.255.0      192.168.2.70      192.168.2.70      266
        192.168.2.0     255.255.255.0       192.168.2.1      192.168.2.70      100
       192.168.2.70   255.255.255.255      192.168.2.70      192.168.2.70      266
      192.168.2.255   255.255.255.255      192.168.2.70      192.168.2.70      266
          224.0.0.0         240.0.0.0         127.0.0.1         127.0.0.1      306
          224.0.0.0         240.0.0.0     192.168.1.162     192.168.1.162      266
          224.0.0.0         240.0.0.0      192.168.2.70      192.168.2.70      266
    255.255.255.255   255.255.255.255         127.0.0.1         127.0.0.1      306
    255.255.255.255   255.255.255.255     192.168.1.162     192.168.1.162      266
    255.255.255.255   255.255.255.255      192.168.2.70      192.168.2.70      266
    396    13:11:47.517  12/17/12  Sev=Info/6    CM/0x63100036
    The routing table was updated for the Virtual Adapter
    397    13:11:47.517  12/17/12  Sev=Info/4    CM/0x6310001A
    One secure connection established
    398    13:11:47.517  12/17/12  Sev=Info/4    CM/0x6310003B
    Address watch added for 192.168.1.162.  Current hostname: MATT-PC, Current address(es): 192.168.2.70, 192.168.1.162.
    399    13:11:47.517  12/17/12  Sev=Info/4    CM/0x6310003B
    Address watch added for 192.168.2.70.  Current hostname: MATT-PC, Current address(es): 192.168.2.70, 192.168.1.162.
    400    13:11:47.517  12/17/12  Sev=Info/5    CM/0x63100001
    Did not find the Smartcard to watch for removal
    401    13:11:47.517  12/17/12  Sev=Info/4    IPSEC/0x63700008
    IPSec driver successfully started
    402    13:11:47.517  12/17/12  Sev=Info/4    IPSEC/0x63700014
    Deleted all keys
    403    13:11:47.517  12/17/12  Sev=Info/6    IPSEC/0x6370002C
    Sent 109 packets, 0 were fragmented.
    404    13:11:47.517  12/17/12  Sev=Info/4    IPSEC/0x63700014
    Deleted all keys
    405    13:11:47.517  12/17/12  Sev=Info/4    IPSEC/0x63700010
    Created a new key structure
    406    13:11:47.517  12/17/12  Sev=Info/4    IPSEC/0x6370000F
    Added key with SPI=0xeaaddbd2 into key list
    407    13:11:47.517  12/17/12  Sev=Info/4    IPSEC/0x63700010
    Created a new key structure
    408    13:11:47.517  12/17/12  Sev=Info/4    IPSEC/0x6370000F
    Added key with SPI=0x37287614 into key list
    409    13:11:47.517  12/17/12  Sev=Info/4    IPSEC/0x6370002F
    Assigned VA private interface addr 192.168.2.70
    410    13:11:47.517  12/17/12  Sev=Info/4    IPSEC/0x63700037
    Configure public interface: 192.168.1.162. SG: 69.61.228.178
    411    13:11:47.517  12/17/12  Sev=Info/6    CM/0x63100046
    Set tunnel established flag in registry to 1.
    412    13:11:52.688  12/17/12  Sev=Info/4    IKE/0x63000013
    SENDING >>> ISAKMP OAK INFO *(HASH, NOTIFY:DPD_REQUEST) to 69.61.228.178
    413    13:11:52.688  12/17/12  Sev=Info/6    IKE/0x6300003D
    Sending DPD request to 69.61.228.178, our seq# = 2722476009
    414    13:11:52.704  12/17/12  Sev=Info/5    IKE/0x6300002F
    Received ISAKMP packet: peer = 69.61.228.178
    415    13:11:52.704  12/17/12  Sev=Info/4    IKE/0x63000014
    RECEIVING <<< ISAKMP OAK INFO *(HASH, NOTIFY:DPD_ACK) from 69.61.228.178
    416    13:11:52.704  12/17/12  Sev=Info/5    IKE/0x63000040
    Received DPD ACK from 69.61.228.178, seq# received = 2722476009, seq# expected = 2722476009
    417    13:12:03.187  12/17/12  Sev=Info/4    IKE/0x63000013
    SENDING >>> ISAKMP OAK INFO *(HASH, NOTIFY:DPD_REQUEST) to 69.61.228.178
    418    13:12:03.187  12/17/12  Sev=Info/6    IKE/0x6300003D
    Sending DPD request to 69.61.228.178, our seq# = 2722476010
    419    13:12:03.202  12/17/12  Sev=Info/5    IKE/0x6300002F
    Received ISAKMP packet: peer = 69.61.228.178
    420    13:12:03.202  12/17/12  Sev=Info/4    IKE/0x63000014
    RECEIVING <<< ISAKMP OAK INFO *(HASH, NOTIFY:DPD_ACK) from 69.61.228.178
    421    13:12:03.202  12/17/12  Sev=Info/5    IKE/0x63000040
    Received DPD ACK from 69.61.228.178, seq# received = 2722476010, seq# expected = 2722476010
    422    13:12:14.185  12/17/12  Sev=Info/4    IKE/0x63000013
    SENDING >>> ISAKMP OAK INFO *(HASH, NOTIFY:DPD_REQUEST) to 69.61.228.178
    423    13:12:14.185  12/17/12  Sev=Info/6    IKE/0x6300003D
    Sending DPD request to 69.61.228.178, our seq# = 2722476011
    424    13:12:14.201  12/17/12  Sev=Info/5    IKE/0x6300002F
    Received ISAKMP packet: peer = 69.61.228.178
    425    13:12:14.201  12/17/12  Sev=Info/4    IKE/0x63000014
    RECEIVING <<< ISAKMP OAK INFO *(HASH, NOTIFY:DPD_ACK) from 69.61.228.178
    426    13:12:14.201  12/17/12  Sev=Info/5    IKE/0x63000040
    Received DPD ACK from 69.61.228.178, seq# received = 2722476011, seq# expected = 2722476011
    427    13:12:24.762  12/17/12  Sev=Info/4    IKE/0x63000013
    SENDING >>> ISAKMP OAK INFO *(HASH, NOTIFY:DPD_REQUEST) to 69.61.228.178
    428    13:12:24.762  12/17/12  Sev=Info/6    IKE/0x6300003D
    Sending DPD request to 69.61.228.178, our seq# = 2722476012
    429    13:12:24.778  12/17/12  Sev=Info/5    IKE/0x6300002F
    Received ISAKMP packet: peer = 69.61.228.178
    430    13:12:24.778  12/17/12  Sev=Info/4    IKE/0x63000014
    RECEIVING <<< ISAKMP OAK INFO *(HASH, NOTIFY:DPD_ACK) from 69.61.228.178
    431    13:12:24.778  12/17/12  Sev=Info/5    IKE/0x63000040
    Received DPD ACK from 69.61.228.178, seq# received = 2722476012, seq# expected = 2722476012
    New running configuration:
    : Saved
    ASA Version 8.4(1)
    hostname NCHCO
    enable password hTjwXz/V8EuTw9p9 encrypted
    passwd hTjwXz/V8EuTw9p9 encrypted
    names
    name 192.168.2.0 NCHCO description City Offices
    name 192.168.2.80 VPN_End
    name 192.168.2.70 VPN_Start
    interface Vlan1
    nameif inside
    security-level 100
    ip address 192.168.2.1 255.255.255.0
    interface Vlan2
    nameif outside
    security-level 0
    ip address 69.61.228.178 255.255.255.248
    interface Ethernet0/0
    switchport access vlan 2
    speed 100
    duplex full
    interface Ethernet0/1
    interface Ethernet0/2
    interface Ethernet0/3
    interface Ethernet0/4
    interface Ethernet0/5
    interface Ethernet0/6
    interface Ethernet0/7
    boot system disk0:/asa841-k8.bin
    ftp mode passive
    object network NCHCO
    subnet 192.168.2.0 255.255.255.0
    object network obj-192.168.1.0
    subnet 192.168.1.0 255.255.255.0
    object network obj-192.168.2.64
    subnet 192.168.2.64 255.255.255.224
    object network obj-0.0.0.0
    subnet 0.0.0.0 255.255.255.0
    object network obj_any
    subnet 0.0.0.0 0.0.0.0
    access-list outside_nat0_outbound extended permit ip object NCHCO 192.168.1.0 255.255.255.0
    access-list outside_nat0_outbound extended permit ip object NCHCO 192.168.2.0 255.255.255.0
    access-list inside_nat0_outbound extended permit ip object NCHCO 192.168.1.0 255.255.255.0
    access-list inside_nat0_outbound extended permit ip any 192.168.2.64 255.255.255.224
    access-list inside_nat0_outbound extended permit ip 0.0.0.0 255.255.255.0 192.168.2.64 255.255.255.224
    access-list outside_1_cryptomap extended permit ip object NCHCO 192.168.1.0 255.255.255.0
    access-list outside_1_cryptomap_1 extended permit ip object NCHCO 192.168.1.0 255.255.255.0
    access-list LAN_Access standard permit 192.168.2.0 255.255.255.0
    access-list LAN_Access standard permit 0.0.0.0 255.255.255.0
    access-list NCHCO_splitTunnelAcl_1 standard permit 192.168.2.0 255.255.255.0
    access-list AnyConnect_Client_Local_Print extended deny ip any any
    access-list AnyConnect_Client_Local_Print extended permit tcp any any eq lpd
    access-list AnyConnect_Client_Local_Print remark IPP: Internet Printing Protocol
    access-list AnyConnect_Client_Local_Print extended permit tcp any any eq 631
    access-list AnyConnect_Client_Local_Print remark Windows' printing port
    access-list AnyConnect_Client_Local_Print extended permit tcp any any eq 9100
    access-list AnyConnect_Client_Local_Print remark mDNS: multicast DNS protocol
    access-list AnyConnect_Client_Local_Print extended permit udp any host 224.0.0.251 eq 5353
    access-list AnyConnect_Client_Local_Print remark LLMNR: Link Local Multicast Name Resolution protocol
    access-list AnyConnect_Client_Local_Print extended permit udp any host 224.0.0.252 eq 5355
    access-list AnyConnect_Client_Local_Print remark TCP/NetBIOS protocol
    access-list AnyConnect_Client_Local_Print extended permit tcp any any eq 137
    access-list AnyConnect_Client_Local_Print extended permit udp any any eq netbios-ns
    pager lines 24
    logging enable
    logging asdm informational
    mtu inside 1500
    mtu outside 1500
    ip local pool VPN_Pool VPN_Start-VPN_End mask 255.255.255.0
    icmp unreachable rate-limit 1 burst-size 1
    asdm image disk0:/asdm-649.bin
    no asdm history enable
    arp timeout 14400
    nat (inside,any) source static NCHCO NCHCO destination static obj-192.168.1.0 obj-192.168.1.0
    nat (inside,any) source static any any destination static obj-192.168.2.64 obj-192.168.2.64
    nat (inside,any) source static obj-0.0.0.0 obj-0.0.0.0 destination static obj-192.168.2.64 obj-192.168.2.64
    object network obj_any
    nat (inside,outside) dynamic interface
    route outside 0.0.0.0 0.0.0.0 69.61.228.177 1
    timeout xlate 3:00:00
    timeout conn 1:00:00 half-closed 0:10:00 udp 0:02:00 icmp 0:00:02
    timeout sunrpc 0:10:00 h323 0:05:00 h225 1:00:00 mgcp 0:05:00 mgcp-pat 0:05:00
    timeout sip 0:30:00 sip_media 0:02:00 sip-invite 0:03:00 sip-disconnect 0:02:00
    timeout sip-provisional-media 0:02:00 uauth 0:05:00 absolute
    timeout tcp-proxy-reassembly 0:01:00
    dynamic-access-policy-record DfltAccessPolicy
    network-acl outside_nat0_outbound
    webvpn
      svc ask enable default svc
    http server enable
    http 192.168.1.0 255.255.255.0 inside
    http 69.61.228.178 255.255.255.255 outside
    http 74.218.158.238 255.255.255.255 outside
    http NCHCO 255.255.255.0 inside
    no snmp-server location
    no snmp-server contact
    snmp-server enable traps snmp authentication linkup linkdown coldstart
    crypto ipsec ikev1 transform-set ESP-AES-128-SHA esp-aes esp-sha-hmac
    crypto ipsec ikev1 transform-set ESP-AES-128-MD5 esp-aes esp-md5-hmac
    crypto ipsec ikev1 transform-set ESP-AES-192-SHA esp-aes-192 esp-sha-hmac
    crypto ipsec ikev1 transform-set ESP-AES-192-MD5 esp-aes-192 esp-md5-hmac
    crypto ipsec ikev1 transform-set ESP-AES-256-SHA esp-aes-256 esp-sha-hmac
    crypto ipsec ikev1 transform-set ESP-AES-256-MD5 esp-aes-256 esp-md5-hmac
    crypto ipsec ikev1 transform-set ESP-3DES-MD5 esp-3des esp-md5-hmac
    crypto ipsec ikev1 transform-set ESP-DES-SHA esp-des esp-sha-hmac
    crypto ipsec ikev1 transform-set ESP-DES-MD5 esp-des esp-md5-hmac
    crypto ipsec ikev1 transform-set l2tp-transform esp-3des esp-sha-hmac
    crypto ipsec ikev1 transform-set l2tp-transform mode transport
    crypto ipsec ikev1 transform-set vpn-transform esp-aes-256 esp-sha-hmac
    crypto ipsec ikev1 transform-set TRANS_ESP_3DES_SHA esp-3des esp-sha-hmac
    crypto ipsec ikev1 transform-set TRANS_ESP_3DES_SHA mode transport
    crypto ipsec ikev1 transform-set TRANS_ESP_3DES_MD5 esp-3des esp-md5-hmac
    crypto ipsec ikev1 transform-set TRANS_ESP_3DES_MD5 mode transport
    crypto ipsec ikev1 transform-set ESP-3DES-SHA esp-3des esp-sha-hmac
    crypto dynamic-map SYSTEM_DEFAULT_CRYPTO_MAP 65535 set pfs group1
    crypto dynamic-map SYSTEM_DEFAULT_CRYPTO_MAP 65535 set ikev1 transform-set ESP-AES-128-SHA ESP-AES-128-MD5 ESP-AES-192-SHA ESP-AES-192-MD5 ESP-AES-256-SHA ESP-AES-256-MD5 ESP-3DES-SHA ESP-3DES-MD5 ESP-DES-SHA ESP-DES-MD5
    crypto dynamic-map dyn-map 10 set pfs group1
    crypto dynamic-map dyn-map 10 set ikev1 transform-set l2tp-transform vpn-transform
    crypto dynamic-map dyn-map 10 set reverse-route
    crypto dynamic-map outside_dyn_map 20 set ikev1 transform-set ESP-3DES-SHA
    crypto map outside_map 1 match address outside_1_cryptomap
    crypto map outside_map 1 set pfs group1
    crypto map outside_map 1 set peer 74.219.208.50
    crypto map outside_map 1 set ikev1 transform-set ESP-3DES-SHA
    crypto map outside_map 20 ipsec-isakmp dynamic outside_dyn_map
    crypto map outside_map 65535 ipsec-isakmp dynamic SYSTEM_DEFAULT_CRYPTO_MAP
    crypto map outside_map interface outside
    crypto map inside_map 65535 ipsec-isakmp dynamic SYSTEM_DEFAULT_CRYPTO_MAP
    crypto map inside_map interface inside
    crypto map vpn-map 1 match address outside_1_cryptomap_1
    crypto map vpn-map 1 set pfs group1
    crypto map vpn-map 1 set peer 74.219.208.50
    crypto map vpn-map 1 set ikev1 transform-set ESP-3DES-SHA
    crypto map vpn-map 10 ipsec-isakmp dynamic dyn-map
    crypto isakmp identity address
    crypto ikev1 enable inside
    crypto ikev1 enable outside
    crypto ikev1 ipsec-over-tcp port 10000
    crypto ikev1 policy 10
    authentication pre-share
    encryption 3des
    hash md5
    group 2
    lifetime 86400
    crypto ikev1 policy 15
    authentication pre-share
    encryption aes-256
    hash sha
    group 2
    lifetime 86400
    crypto ikev1 policy 35
    authentication pre-share
    encryption 3des
    hash sha
    group 2
    lifetime 86400
    client-update enable
    telnet 192.168.1.0 255.255.255.0 inside
    telnet NCHCO 255.255.255.0 inside
    telnet timeout 5
    ssh 192.168.1.0 255.255.255.0 inside
    ssh NCHCO 255.255.255.0 inside
    ssh timeout 5
    console timeout 0
    dhcpd address 192.168.2.150-192.168.2.225 inside
    dhcpd dns 216.68.4.10 216.68.5.10 interface inside
    dhcpd lease 64000 interface inside
    threat-detection basic-threat
    threat-detection statistics access-list
    no threat-detection statistics tcp-intercept
    webvpn
    group-policy DefaultRAGroup internal
    group-policy DefaultRAGroup attributes
    dns-server value 192.168.2.1
    vpn-tunnel-protocol ikev1 l2tp-ipsec
    default-domain value nchco.local
    group-policy DfltGrpPolicy attributes
    dns-server value 192.168.2.1
    vpn-tunnel-protocol ikev1 l2tp-ipsec ssl-client ssl-clientless
    password-storage enable
    ipsec-udp enable
    intercept-dhcp 255.255.255.0 enable
    address-pools value VPN_Pool
    group-policy NCHCO internal
    group-policy NCHCO attributes
    dns-server value 192.168.2.1 8.8.8.8
    vpn-tunnel-protocol ikev1
    split-tunnel-policy tunnelspecified
    split-tunnel-network-list value NCHCO_splitTunnelAcl_1
    default-domain value NCHCO.local
    username admin password LbMiJuAJjDaFb2uw encrypted privilege 15
    username 8njferg password yB1lHEVmHZGj5C2Z encrypted privilege 15
    username NCHvpn99 password dhn.JzttvRmMbHsP encrypted
    tunnel-group DefaultRAGroup general-attributes
    address-pool (inside) VPN_Pool
    address-pool VPN_Pool
    authentication-server-group (inside) LOCAL
    authentication-server-group (outside) LOCAL
    authorization-server-group LOCAL
    authorization-server-group (inside) LOCAL
    authorization-server-group (outside) LOCAL
    default-group-policy DefaultRAGroup
    strip-realm
    strip-group
    tunnel-group DefaultRAGroup ipsec-attributes
    ikev1 pre-shared-key *****
    peer-id-validate nocheck
    tunnel-group DefaultRAGroup ppp-attributes
    no authentication chap
    no authentication ms-chap-v1
    authentication ms-chap-v2
    tunnel-group DefaultWEBVPNGroup ppp-attributes
    authentication pap
    authentication ms-chap-v2
    tunnel-group 74.219.208.50 type ipsec-l2l
    tunnel-group 74.219.208.50 ipsec-attributes
    ikev1 pre-shared-key *****
    tunnel-group NCHCO type remote-access
    tunnel-group NCHCO general-attributes
    address-pool VPN_Pool
    default-group-policy NCHCO
    tunnel-group NCHCO ipsec-attributes
    ikev1 pre-shared-key *****
    class-map inspection_default
    match default-inspection-traffic
    policy-map type inspect dns preset_dns_map
    parameters
      message-length maximum client auto
      message-length maximum 512
    policy-map global_policy
    class inspection_default
      inspect dns preset_dns_map
      inspect ftp
      inspect h323 h225
      inspect h323 ras
      inspect rsh
      inspect rtsp
      inspect esmtp
      inspect sqlnet
      inspect skinny 
      inspect sunrpc
      inspect xdmcp
      inspect sip 
      inspect netbios
      inspect tftp
      inspect ip-options
    service-policy global_policy global
    prompt hostname context
    call-home
    profile CiscoTAC-1
      no active
      destination address http https://tools.cisco.com/its/service/oddce/services/DDCEService
      destination address email [email protected]
      destination transport-method http
      subscribe-to-alert-group diagnostic
      subscribe-to-alert-group environment
      subscribe-to-alert-group inventory periodic monthly
      subscribe-to-alert-group configuration periodic monthly
      subscribe-to-alert-group telemetry periodic daily
    Cryptochecksum:b6ce58676b6aaeba48caacbeefea53a5
    : end
    asdm image disk0:/asdm-649.bin
    asdm location VPN_Start 255.255.255.255 inside
    asdm location VPN_End 255.255.255.255 inside
    no asdm history enable
    I'm at a loss myself as to why this isn't working, and i'm sure that you are running out of solutions yourself. Any other ideas? I really need to get this working.
    Thanks so much!
    Matthew

  • Site to Site VPN between ASA 5505 and Cisco 800 router

    Evening all,
    Hoping that someboy can see the error of my ways.  It seems very like the problem that i read here: https://supportforums.cisco.com/thread/2016300
    We have a cisco 800 in a remote site which we wanted to use for a site to site vpn.  Went through the steps on the ASA 5505 and the 800 and have got to the stage were the tunnel is up and connected.  Getting traffic through it is another matter.  Remote network is 172.20.224.0/20 and the server network behind the ASA is 192.168.168.0/24. The tunnel does intiate when you send traffic from 172 ......to 192.......  Both the ASA and 800 report the tunnel is up.  If i look at the stats using ccp on the 800 i can see the encapsulation packets graph shooting up but nothing cominbg back.  I did packet captures on the 5505 and could not see anything coming from the tunnel so i dont belive its making it to the ASA.  Here is the config from the 800:
    Building configuration...
    Current configuration : 6488 bytes
    version 12.4
    no service pad
    service timestamps debug uptime
    service timestamps log uptime
    no service password-encryption
    hostname hhp-sty-backup
    boot-start-marker
    boot-end-marker
    logging message-counter syslog
    logging buffered 4096
    enable secret 5 $1$jI1i$/kZbRk2WHD5h0HtfuQVej1
    aaa new-model
    aaa authentication login default local
    aaa authorization exec default local
    aaa authorization auth-proxy default local
    aaa session-id common
    crypto pki trustpoint TP-self-signed-1347488939
    enrollment selfsigned
    subject-name cn=IOS-Self-Signed-Certificate-1347488939
    revocation-check none
    rsakeypair TP-self-signed-1347488939
    crypto pki certificate chain TP-self-signed-1347488939
    certificate self-signed 02
      30820255 308201BE A0030201 02020102 300D0609 2A864886 F70D0101 04050030
      31312F30 2D060355 04031326 494F532D 53656C66 2D536967 6E65642D 43657274
      69666963 6174652D 31333437 34383839 3339301E 170D3032 30333031 30313336
      33375A17 0D323030 31303130 30303030 305A3031 312F302D 06035504 03132649
      4F532D53 656C662D 5369676E 65642D43 65727469 66696361 74652D31 33343734
      38383933 3930819F 300D0609 2A864886 F70D0101 01050003 818D0030 81890281
      8100E714 7B0ADB41 19F60528 A8A5C43B 5CD2D1CD DCCF2E08 8B38D444 36EAB9B7
      0E93CEF7 660F979E E27915B9 E44812A5 794EA03D BA66752B FD0F7EBF D6342513
      D6410E4E 098CE838 C3BADD0A 5F3505FE 22CA776F 89B19510 F0852225 3600F046
      4D57D2E2 FE4AAD1E 8BE4BF80 7B27369E BFA65160 BC769BC9 00A13741 E336D0EA
      8A810203 010001A3 7D307B30 0F060355 1D130101 FF040530 030101FF 30280603
      551D1104 21301F82 1D686870 2D737479 2D626163 6B75702E 796F7572 646F6D61
      696E2E63 6F6D301F 0603551D 23041830 168014FA 4A8C4DF6 629638DE 87D7B60A
      0F5BB40F EA6AED30 1D060355 1D0E0416 0414FA4A 8C4DF662 9638DE87 D7B60A0F
      5BB40FEA 6AED300D 06092A86 4886F70D 01010405 00038181 00BBE577 6EF63FE7
      789766D5 37841812 298D4885 1CD06D07 4C625369 C3403106 89EE1398 73495432
      66C49CB1 36A5B2F8 D77A8C46 5AFE4112 EA5917D9 81542640 80EF2D36 54A85CC6
      C3FFFFB8 39A648DD 2ABA2B13 4137BE07 760E46C0 74401DA7 482E3FA2 A64B70FF
      447AA1B2 52E37240 29987085 532BBE3B C2E2E54A 54CA1D13 0E
                quit
    dot11 syslog
    ip source-route
    ip dhcp excluded-address 10.10.10.1
    ip dhcp pool inside
    ip dhcp pool lan_network
       network 172.20.224.0 255.255.240.0
       dns-server 8.8.8.8 8.8.4.4
       default-router 172.20.224.1
       lease 7
    ip cef
    no ip domain lookup
    ip domain name yourdomain.com
    password encryption aes
    username pix privilege 15 secret 5 $1$Z.wA$lBmj36AJx/cbK1RjmfGJh1
    username admin privilege 15 password 0 434Zaty
    crypto isakmp policy 1
    encr 3des
    authentication pre-share
    group 2
    crypto isakmp key password address 217.36.32.222
    crypto ipsec transform-set ESP-3DES-SHA esp-3des esp-sha-hmac
    crypto map SDM_CMAP_1 1 ipsec-isakmp
    description Tunnel to217.36.32.222
    set peer 217.36.32.222
    set transform-set ESP-3DES-SHA
    match address 100
    archive
    log config
      hidekeys
    interface ATM0
    no ip address
    no atm ilmi-keepalive
    pvc 0/38
      encapsulation aal5mux ppp dialer
      dialer pool-member 1
    dsl operating-mode auto
    interface FastEthernet0
    interface FastEthernet1
    interface FastEthernet2
    interface FastEthernet3
    interface Vlan1
    description $ETH-SW-LAUNCH$$INTF-INFO-HWIC 4ESW$
    ip address 172.20.224.1 255.255.240.0
    ip nat inside
    ip virtual-reassembly
    ip tcp adjust-mss 1452
    interface Dialer0
    ip address negotiated
    ip nat outside
    ip virtual-reassembly
    encapsulation ppp
    dialer pool 1
    no cdp enable
    ppp authentication chap callin
    ppp chap hostname B6*******.btclick.com
    ppp chap password 0 H*******
    crypto map SDM_CMAP_1
    ip forward-protocol nd
    ip route 0.0.0.0 0.0.0.0 Dialer0
    ip http server
    ip http authentication local
    ip http secure-server
    ip http timeout-policy idle 60 life 86400 requests 10000
    ip nat inside source route-map SDM_RMAP_1 interface Dialer0 overload
    access-list 1 remark CCP_ACL Category=16
    access-list 1 permit 172.4.0.0 0.240.255.255
    access-list 10 permit 195.12.1.35
    access-list 10 permit 172.4.0.0 0.240.255.255
    access-list 100 remark CCP_ACL Category=4
    access-list 100 remark IPSec Rule
    access-list 100 permit ip 172.20.224.0 0.0.15.255 192.168.168.0 0.0.0.255
    access-list 101 remark CCP_ACL Category=2
    access-list 101 remark IPSec Rule
    access-list 101 deny   ip 172.20.224.0 0.0.15.255 192.168.168.0 0.0.0.255
    access-list 101 permit ip 172.4.0.0 0.240.255.255 any
    route-map SDM_RMAP_1 permit 1
    match ip address 101
    control-plane
    banner exec ^C
    % Password expiration warning.
    Cisco Configuration Professional (Cisco CP) is installed on this device
    and it provides the default username "cisco" for  one-time use. If you have
    already used the username "cisco" to login to the router and your IOS image
    supports the "one-time" user option, then this username has already expired.
    You will not be able to login to the router with this username after you exit
    this session.
    It is strongly suggested that you create a new username with a privilege level
    of 15 using the following command.
    username <myuser> privilege 15 secret 0 <mypassword>
    Replace <myuser> and <mypassword> with the username and password you
    want to use.
    ^C
    banner login ^C
    Cisco Configuration Professional (Cisco CP) is installed on this device.
    This feature requires the one-time use of the username "cisco" with the
    password "cisco". These default credentials have a privilege level of 15.
    YOU MUST USE CISCO CP or the CISCO IOS CLI TO CHANGE THESE 
    PUBLICLY-KNOWN CREDENTIALS
    Here are the Cisco IOS commands.
    username <myuser>  privilege 15 secret 0 <mypassword>
    no username cisco
    Replace <myuser> and <mypassword> with the username and password you want
    to use.
    IF YOU DO NOT CHANGE THE PUBLICLY-KNOWN CREDENTIALS, YOU WILL
    NOT BE ABLE TO LOG INTO THE DEVICE AGAIN AFTER YOU HAVE LOGGED OFF.
    For more information about Cisco CP please follow the instructions in the
    QUICK START GUIDE for your router or go to http://www.cisco.com/go/ciscocp
    ^C
    line con 0
    no modem enable
    stopbits 1
    line aux 0
    line vty 0 4
    access-class 10 in
    privilege level 15
    password 434Zaty
    transport input telnet ssh
    scheduler max-task-time 5000
    end
    Any help will be most gratefully recieved.

    Rick,
    Thanks for replying.  Here is the output from the 800 Show Crypto command:
    interface: Dialer0
        Crypto map tag: SDM_CMAP_1, local addr 81.136.160.237
       protected vrf: (none)
       local  ident (addr/mask/prot/port): (172.20.224.0/255.255.240.0/0/0)
       remote ident (addr/mask/prot/port): (192.168.168.0/255.255.255.0/0/0)
       current_peer 217.36.32.222 port 500
         PERMIT, flags={origin_is_acl,}
        #pkts encaps: 10928, #pkts encrypt: 10928, #pkts digest: 10928
        #pkts decaps: 0, #pkts decrypt: 0, #pkts verify: 0
        #pkts compressed: 0, #pkts decompressed: 0
        #pkts not compressed: 0, #pkts compr. failed: 0
        #pkts not decompressed: 0, #pkts decompress failed: 0
        #send errors 2, #recv errors 0
         local crypto endpt.: 81.136.160.237, remote crypto endpt.: 217.36.32.222
         path mtu 1500, ip mtu 1500, ip mtu idb Virtual-Access2
         current outbound spi: 0x0(0)
         inbound esp sas:
         inbound ah sas:
         inbound pcp sas:
         outbound esp sas:
         outbound ah sas:
         outbound pcp sas:
    interface: Virtual-Access2
        Crypto map tag: SDM_CMAP_1, local addr 81.136.160.237
       protected vrf: (none)
       local  ident (addr/mask/prot/port): (172.20.224.0/255.255.240.0/0/0)
       remote ident (addr/mask/prot/port): (192.168.168.0/255.255.255.0/0/0)
       current_peer 217.36.32.222 port 500
         PERMIT, flags={origin_is_acl,}
        #pkts encaps: 10928, #pkts encrypt: 10928, #pkts digest: 10928
        #pkts decaps: 0, #pkts decrypt: 0, #pkts verify: 0
        #pkts compressed: 0, #pkts decompressed: 0
        #pkts not compressed: 0, #pkts compr. failed: 0
        #pkts not decompressed: 0, #pkts decompress failed: 0
        #send errors 2, #recv errors 0
         local crypto endpt.: 81.136.160.237, remote crypto endpt.: 217.36.32.222
         path mtu 1500, ip mtu 1500, ip mtu idb Virtual-Access2
         current outbound spi: 0x0(0)
         inbound esp sas:
         inbound ah sas:
         inbound pcp sas:
         outbound esp sas:
         outbound ah sas:
         outbound pcp sas:
    and this is the running config frm our ASA at HQ:
    Result of the command: "sh run"
    : Saved
    ASA Version 8.2(1)
    hostname secure-access
    domain-name hhp.com
    enable password UWWykvGjAPmxufUo encrypted
    passwd 2KFQnbNIdI.2KYOU encrypted
    names
    interface Vlan1
    nameif inside
    security-level 100
    ip address 192.168.168.1 255.255.255.0
    interface Vlan2
    nameif outside
    security-level 0
    pppoe client vpdn group BT
    ip address 217.36.32.222 255.255.255.255 pppoe
    interface Vlan12
    nameif DMZ
    security-level 50
    ip address 192.168.169.1 255.255.255.0
    interface Vlan22
    nameif Wireless_HHP
    security-level 100
    ip address 172.16.36.1 255.255.254.0
    interface Vlan32
    nameif CNES
    security-level 100
    ip address 187.187.168.1 255.255.0.0
    interface Ethernet0/0
    switchport access vlan 2
    interface Ethernet0/1
    interface Ethernet0/2
    switchport access vlan 12
    interface Ethernet0/3
    switchport access vlan 22
    interface Ethernet0/4
    switchport access vlan 32
    interface Ethernet0/5
    switchport access vlan 12
    interface Ethernet0/6
    switchport access vlan 12
    interface Ethernet0/7
    ftp mode passive
    dns domain-lookup inside
    dns domain-lookup outside
    dns domain-lookup DMZ
    dns domain-lookup Wireless_HHP
    dns domain-lookup CNES
    dns server-group DefaultDNS
    name-server 192.168.168.2
    domain-name hhp.com
    same-security-traffic permit inter-interface
    same-security-traffic permit intra-interface
    object-group network NET-cnes_HHP-Sty
    network-object 172.20.224.0 255.255.240.0
    object-group network NET-cnes_HHP-Balivanich
    network-object 172.20.192.0 255.255.240.0
    object-group network Oak-DC1
    network-object 192.168.168.2 255.255.255.255
    object-group network Maple-DC2
    network-object 192.168.168.3 255.255.255.255
    object-group network HHP_Domain_Controllers
    group-object Oak-DC1
    group-object Maple-DC2
    object-group network PC-Support
    network-object 187.187.60.1 255.255.255.255
    network-object 187.187.60.2 255.255.255.254
    network-object 187.187.60.4 255.255.255.254
    network-object 187.187.60.6 255.255.255.255
    object-group network ELM-ActiveH
    network-object 192.168.168.6 255.255.255.255
    object-group network Pine-GP
    network-object 192.168.168.12 255.255.255.255
    object-group network HHP_Application_Servers
    group-object ELM-ActiveH
    group-object Pine-GP
    object-group network Fern-TS1
    network-object 192.168.168.4 255.255.255.255
    object-group network Fir-TS2
    network-object 192.168.168.5 255.255.255.255
    object-group network HHP_Terminal_Servers
    group-object Fern-TS1
    group-object Fir-TS2
    object-group service Global_Catalog_LDAP
    description (Generated by Cisco SM from Object "Global Catalog LDAP")
    service-object tcp eq 3268
    object-group service Global_Catalog_LDAP_SSL
    description (Generated by Cisco SM from Object "Global Catalog LDAP SSL")
    service-object tcp eq 3269
    object-group service UDP-389
    description UDP port for LDAP
    service-object udp eq 389
    object-group service TCP-88
    description TCP Port 88
    service-object tcp eq 88
    object-group service TCP-445
    description SMB
    service-object tcp eq 445
    object-group network John_-_Laptop
    description John's Laptop
    network-object 187.187.10.65 255.255.255.255
    object-group network Graham_-_PC
    description Graham Morrison's PC
    network-object 187.187.10.90 255.255.255.255
    object-group network john_test
    network-object 187.187.40.7 255.255.255.255
    object-group network Iain_PC
    description Iain Macaulay IT
    network-object 187.187.10.19 255.255.255.255
    object-group network John_-_PC
    description John MacPhail's PC
    network-object 187.187.10.7 255.255.255.255
    object-group network it-alahen-lap
    network-object 187.187.10.230 255.255.255.255
    object-group network Catriona_-_Laptop
    description Catriona's Laptop
    network-object 187.187.10.60 255.255.255.255
    object-group network Graham_-_Laptop
    network-object 187.186.10.120 255.255.255.255
    object-group network it-innive-xp
    description Innes MacIver's PC
    network-object 187.187.10.14 255.255.255.255
    object-group network it-alahen-xp
    description Desktop
    network-object 187.187.10.229 255.255.255.255
    object-group network Cat_-_PC
    description Catriona Macmillan's PC
    network-object 187.187.10.4 255.255.255.255
    object-group network it-davdon-xp
    description Desktop
    network-object 187.187.160.7 255.255.255.255
    object-group network cat-laptop
    description Catriona's Laptop addresses
    network-object 187.187.77.81 255.255.255.255
    network-object 187.187.77.82 255.255.255.255
    object-group network Catriona_old_pc
    network-object 187.187.10.44 255.255.255.255
    object-group network cat-tablet
    description Catriona's Tablet ip address's
    network-object 187.187.77.78 255.255.255.254
    object-group network DSO-SQLServer
    description Task Database Server
    network-object 187.187.1.33 255.255.255.255
    object-group network it-finfernew-xp
    description Findlay Ferguson PC
    network-object 187.187.10.153 255.255.255.255
    object-group network PC_Support
    group-object John_-_Laptop
    group-object Graham_-_PC
    group-object john_test
    group-object Iain_PC
    group-object John_-_PC
    group-object it-alahen-lap
    group-object Catriona_-_Laptop
    group-object Graham_-_Laptop
    group-object it-alahen-xp
    group-object Cat_-_PC
    group-object it-davdon-xp
    group-object cat-laptop
    group-object Catriona_old_pc
    group-object cat-tablet
    group-object it-innive-xp
    network-object 187.187.1.128 255.255.255.255
    network-object 187.187.10.76 255.255.255.255
    group-object DSO-SQLServer
    network-object 187.187.15.234 255.255.255.255
    network-object 187.187.4.60 255.255.255.255
    network-object 187.187.10.134 255.255.255.255
    network-object 172.18.194.22 255.255.255.255
    group-object it-finfernew-xp
    object-group network Entire_CNE
    description Entire CNE range
    network-object 187.0.0.0 255.0.0.0
    object-group network NET-cnes_HHP-Sty-Staff
    network-object 172.20.225.0 255.255.255.0
    object-group network NET-cnes_HHP-Balivanich-staff
    network-object 172.20.193.0 255.255.255.0
    object-group network Alder-Intranet
    network-object 192.168.168.13 255.255.255.255
    object-group network Aspen-ISA
    network-object 192.168.168.10 255.255.255.255
    object-group service tcp-8080
    description TCP Port 8080
    service-object tcp eq 8080
    object-group network Beech-External
    network-object 217.36.32.210 255.255.255.255
    object-group network it-csm
    description cisco security manager
    network-object 187.187.1.72 255.255.255.255
    object-group network Juniper-External
    description Internet Server
    network-object 217.36.32.211 255.255.255.255
    object-group network HHP_Server_Network
    network-object 192.168.168.0 255.255.255.0
    object-group network Messagelabs_Incoming_HHP
    network-object 67.219.240.0 255.255.240.0
    network-object 95.131.104.0 255.255.248.0
    network-object 193.109.254.0 255.255.254.0
    network-object 195.245.230.0 255.255.254.0
    network-object 216.82.240.0 255.255.240.0
    network-object 85.158.136.0 255.255.248.0
    network-object 117.120.16.0 255.255.248.0
    network-object 194.106.220.0 255.255.254.0
    object-group network Angus-Maclean-PC
    network-object 187.187.10.250 255.255.255.255
    object-group service RDP
    service-object tcp eq 3389
    object-group network it-dbserver
    description Database Server (Live)
    network-object 187.187.1.65 255.255.255.255
    object-group network it-sql-test
    description Test SQL / database server
    network-object 187.187.1.81 255.255.255.255
    object-group service DNS-Resolving
    description Domain Name Server
    service-object tcp eq domain
    service-object udp eq domain
    object-group network Beech-Exchange
    network-object 192.168.168.91 255.255.255.255
    object-group network Messagelabs_-_Incoming
    description List of MessageLab addresses that SMTP connections are accepted from
    network-object 212.125.75.0 255.255.255.224
    network-object 216.82.240.0 255.255.240.0
    network-object 195.216.16.211 255.255.255.255
    network-object 194.205.110.128 255.255.255.224
    network-object 194.106.220.0 255.255.254.0
    network-object 193.109.254.0 255.255.254.0
    network-object 62.231.131.0 255.255.255.0
    network-object 62.173.108.208 255.255.255.240
    network-object 62.173.108.16 255.255.255.240
    network-object 212.125.74.44 255.255.255.255
    network-object 195.245.230.0 255.255.254.0
    network-object 85.158.136.0 255.255.248.0
    object-group network MIS_Support
    network-object 192.168.168.250 255.255.255.254
    object-group network it-donadon-xp
    description Donald Macdonald's PC
    network-object 187.187.10.13 255.255.255.255
    object-group network Angela_PC
    network-object 187.187.10.155 255.255.255.255
    object-group network Katie_PC
    network-object 187.187.10.151 255.255.255.255
    object-group network Pauline_PC
    network-object 187.187.10.12 255.255.255.255
    object-group network it-paye-net
    network-object 187.187.1.92 255.255.255.255
    object-group network MessageLabs-Towers
    description Message Labs IP Address ranges
    network-object 216.82.240.0 255.255.240.0
    network-object 67.219.240.0 255.255.240.0
    network-object 85.158.136.0 255.255.248.0
    network-object 95.131.104.0 255.255.248.0
    network-object 117.120.16.0 255.255.248.0
    network-object 193.109.254.0 255.255.254.0
    network-object 194.106.220.0 255.255.254.0
    network-object 195.245.230.0 255.255.254.0
    network-object 62.231.131.0 255.255.255.0
    network-object 212.125.75.16 255.255.255.240
    object-group network NET_cnes-castlebay-staff
    network-object 172.19.17.0 255.255.255.0
    object-group network NET_cnes_tarbert_staff
    description NET_cnes_tarbert_staff
    network-object 172.19.33.0 255.255.255.0
    object-group network Juniper
    network-object 192.168.169.5 255.255.255.255
    object-group network HHP_DMZ_Network
    network-object 192.168.169.0 255.255.255.0
    object-group network Ash
    network-object 192.168.168.100 255.255.255.255
    object-group service UDP-445
    service-object udp eq 445
    object-group service tcp-udp-135-139
    service-object tcp-udp range 135 139
    object-group network HHP-ELM
    description HHP's ELM ActiveH server
    network-object 187.187.1.203 255.255.255.255
    object-group network CNES-Ext-GW
    description CNES External Address
    network-object 194.83.245.242 255.255.255.255
    object-group service IPSEC
    description IPSEC
    service-object 57
    service-object ah
    service-object esp
    service-object udp eq isakmp
    object-group network Alamur-PC
    network-object 187.187.10.15 255.255.255.255
    object-group network Iain-Nicolson-PC
    network-object 187.187.10.159 255.255.255.255
    object-group network HHP_Remote_Access_Pool
    network-object 192.168.168.200 255.255.255.248
    network-object 192.168.168.208 255.255.255.240
    network-object 192.168.168.224 255.255.255.252
    network-object 192.168.168.228 255.255.255.254
    object-group network Holly-AV
    network-object 192.168.168.9 255.255.255.255
    object-group service AVG_Ports
    description For AVG server to HHP PCs
    service-object tcp-udp eq 6150
    service-object tcp-udp eq 6051
    service-object tcp-udp eq 445
    service-object tcp-udp eq 138
    service-object tcp-udp eq 135
    service-object tcp-udp eq 6054
    service-object tcp-udp eq 4158
    service-object tcp-udp eq 139
    service-object tcp-udp eq 137
    object-group network CNES_Access
    network-object 192.168.168.230 255.255.255.254
    network-object 192.168.168.232 255.255.255.248
    network-object 192.168.168.240 255.255.255.248
    network-object 192.168.168.248 255.255.255.254
    object-group network HHP-068
    description BACS PC
    network-object 172.20.225.6 255.255.255.255
    object-group network Banyan
    network-object 192.168.168.105 255.255.255.255
    object-group service TCP81
    description TCP Port 81
    service-object tcp eq 81
    object-group network Gavin_-_new_PC
    network-object 187.187.10.150 255.255.255.255
    object-group network Secudoors
    network-object 172.20.224.4 255.255.255.255
    access-list outside_access_in remark Time sync to external ntp server
    access-list outside_access_in extended permit udp host 192.108.114.23 object-group HHP_Domain_Controllers eq ntp
    access-list outside_access_in extended permit tcp object-group MessageLabs-Towers object-group Beech-External eq smtp
    access-list outside_access_in extended permit ip host 81.136.160.237 object-group HHP_Server_Network
    access-list outside_access_in extended permit ip object-group CNES_Access object-group HHP_Server_Network
    access-list outside_access_in extended permit ip object-group MIS_Support object-group HHP_Server_Network
    access-list outside_access_in extended permit ip object-group HHP_Remote_Access_Pool object-group HHP_Server_Network
    access-list outside_access_in extended permit tcp any object-group Juniper-External eq www
    access-list outside_access_in extended permit tcp any object-group Juniper-External eq https
    access-list outside_access_in extended deny ip any any
    access-list outside_access_in_1 extended permit ip any any
    access-list CSM_FW_ACL_Wireless_HHP extended permit ip object-group NET-cnes_HHP-Balivanich object-group HHP_Server_Network
    access-list CSM_FW_ACL_Wireless_HHP extended permit ip object-group NET-cnes_HHP-Sty object-group HHP_Server_Network
    access-list CSM_FW_ACL_Wireless_HHP extended permit tcp object-group HHP-068 any eq www
    access-list CSM_FW_ACL_Wireless_HHP extended permit tcp object-group HHP-068 any eq domain
    access-list CSM_FW_ACL_Wireless_HHP extended permit udp object-group HHP-068 any eq domain
    access-list CSM_FW_ACL_Wireless_HHP extended permit tcp object-group HHP-068 any eq https
    access-list CSM_FW_ACL_Wireless_HHP extended permit object-group DNS-Resolving object-group HHP-068 any
    access-list CSM_FW_ACL_Wireless_HHP extended permit object-group tcp-8080 object-group HHP-068 any
    access-list CSM_FW_ACL_Wireless_HHP extended permit ip host 172.20.193.53 object-group CNES-Ext-GW
    access-list CSM_FW_ACL_Wireless_HHP extended permit ip object-group Secudoors any
    access-list CSM_FW_ACL_inside extended permit ip object-group HHP_Server_Network object-group NET-cnes_HHP-Balivanich
    access-list CSM_FW_ACL_inside extended permit ip object-group HHP_Server_Network object-group NET-cnes_HHP-Sty
    access-list CSM_FW_ACL_inside extended permit ip object-group HHP_Application_Servers object-group PC_Support
    access-list CSM_FW_ACL_inside extended permit ip object-group HHP_Domain_Controllers object-group PC_Support
    access-list CSM_FW_ACL_inside extended permit ip object-group HHP_Terminal_Servers object-group PC_Support
    access-list CSM_FW_ACL_inside extended permit tcp object-group Oak-DC1 any eq domain
    access-list CSM_FW_ACL_inside extended permit udp object-group Oak-DC1 any eq domain
    access-list CSM_FW_ACL_inside extended permit object-group DNS-Resolving object-group Oak-DC1 any
    access-list CSM_FW_ACL_inside extended permit tcp object-group Maple-DC2 any eq domain
    access-list CSM_FW_ACL_inside extended permit udp object-group Maple-DC2 any eq domain
    access-list CSM_FW_ACL_inside extended permit object-group DNS-Resolving object-group Maple-DC2 any
    access-list CSM_FW_ACL_inside extended permit tcp object-group Aspen-ISA any eq www
    access-list CSM_FW_ACL_inside extended permit tcp object-group Aspen-ISA any eq domain
    access-list CSM_FW_ACL_inside extended permit udp object-group Aspen-ISA any eq domain
    access-list CSM_FW_ACL_inside extended permit tcp object-group Aspen-ISA any eq https
    access-list CSM_FW_ACL_inside extended permit object-group DNS-Resolving object-group Aspen-ISA any
    access-list CSM_FW_ACL_inside extended permit object-group tcp-8080 object-group Aspen-ISA any
    access-list CSM_FW_ACL_inside remark For Symantec Liveupdates
    access-list CSM_FW_ACL_inside extended permit tcp object-group Banyan any eq ftp
    access-list CSM_FW_ACL_inside extended permit tcp object-group Banyan any eq www
    access-list CSM_FW_ACL_inside extended permit tcp object-group Banyan any eq https
    access-list CSM_FW_ACL_inside remark IPSec VPN access from ELm to CNES
    access-list CSM_FW_ACL_inside extended permit object-group IPSEC object-group ELM-ActiveH object-group CNES-Ext-GW
    access-list CSM_FW_ACL_inside extended permit udp object-group ELM-ActiveH object-group CNES-Ext-GW eq 4500
    access-list CSM_FW_ACL_inside extended permit tcp object-group ELM-ActiveH object-group CNES-Ext-GW eq 4500
    access-list CSM_FW_ACL_inside extended permit icmp object-group HHP_Server_Network object-group HHP_DMZ_Network
    access-list CSM_FW_ACL_inside remark Time sync to external ntp server
    access-list CSM_FW_ACL_inside extended permit udp object-group HHP_Domain_Controllers host 192.108.114.23 eq ntp
    access-list CSM_FW_ACL_inside extended permit tcp object-group Beech-Exchange object-group Messagelabs_-_Incoming eq smtp
    access-list CSM_FW_ACL_inside extended permit tcp object-group Aspen-ISA object-group Juniper eq www
    access-list CSM_FW_ACL_inside extended permit tcp object-group Aspen-ISA object-group Juniper eq https
    access-list CSM_FW_ACL_inside extended permit ip object-group Holly-AV object-group Juniper
    access-list CSM_FW_ACL_inside extended deny ip any any
    access-list CSM_FW_ACL_CNES extended permit ip object-group PC_Support object-group HHP_Server_Network
    access-list CSM_FW_ACL_CNES extended permit ip object-group PC_Support object-group HHP_DMZ_Network
    access-list CSM_FW_ACL_CNES extended permit ip object-group PC_Support object-group NET-cnes_HHP-Balivanich
    access-list CSM_FW_ACL_CNES extended permit ip object-group PC_Support object-group NET-cnes_HHP-Sty
    access-list CSM_FW_ACL_CNES extended permit tcp object-group it-csm any eq ssh
    access-list CSM_FW_ACL_CNES extended permit tcp object-group it-csm any eq www
    access-list CSM_FW_ACL_CNES extended permit tcp object-group it-csm any eq https
    access-list CSM_FW_ACL_CNES remark Aim's access to Active H server: DSO SQL
    access-list CSM_FW_ACL_CNES remark server's access (Task)
    access-list CSM_FW_ACL_CNES remark IT Ops - mapped drive for FTP transfer to and from E450/Elm of Entitlement Adjustments
    access-list CSM_FW_ACL_CNES remark and Tenancy Changes
    access-list CSM_FW_ACL_CNES extended permit ip object-group it-sql-test object-group ELM-ActiveH
    access-list CSM_FW_ACL_CNES extended permit ip object-group DSO-SQLServer object-group ELM-ActiveH
    access-list CSM_FW_ACL_CNES extended permit ip object-group it-paye-net object-group ELM-ActiveH
    access-list CSM_FW_ACL_CNES extended permit ip object-group Angela_PC object-group ELM-ActiveH
    access-list CSM_FW_ACL_CNES extended permit ip object-group Katie_PC object-group ELM-ActiveH
    access-list CSM_FW_ACL_CNES extended permit ip object-group Pauline_PC object-group ELM-ActiveH
    access-list CSM_FW_ACL_CNES remark donald and Findlay RDP access to Active H
    access-list CSM_FW_ACL_CNES extended permit object-group RDP object-group it-donadon-xp object-group ELM-ActiveH
    access-list CSM_FW_ACL_CNES extended permit object-group RDP object-group it-donadon-xp object-group HHP_Terminal_Servers
    access-list CSM_FW_ACL_CNES extended permit object-group RDP object-group it-finfernew-xp object-group ELM-ActiveH
    access-list CSM_FW_ACL_CNES extended permit object-group RDP object-group it-finfernew-xp object-group HHP_Terminal_Servers
    access-list CSM_FW_ACL_CNES extended permit ip object-group Angus-Maclean-PC object-group Alder-Intranet
    access-list CSM_FW_ACL_CNES extended permit ip object-group Angus-Maclean-PC host 192.168.168.17
    access-list CSM_FW_ACL_CNES extended permit ip object-group Angus-Maclean-PC object-group Juniper
    access-list CSM_FW_ACL_CNES extended permit ip object-group Iain-Nicolson-PC object-group Alder-Intranet
    access-list CSM_FW_ACL_CNES extended permit ip object-group Iain-Nicolson-PC host 192.168.168.17
    access-list CSM_FW_ACL_CNES extended permit ip object-group Iain-Nicolson-PC object-group Juniper
    access-list CSM_FW_ACL_CNES extended permit ip object-group it-davdon-xp object-group Alder-Intranet
    access-list CSM_FW_ACL_CNES extended permit ip object-group it-davdon-xp host 192.168.168.17
    access-list CSM_FW_ACL_CNES extended permit ip object-group it-davdon-xp object-group Juniper
    access-list CSM_FW_ACL_CNES extended permit ip object-group Alamur-PC object-group Alder-Intranet
    access-list CSM_FW_ACL_CNES extended permit ip object-group Alamur-PC host 192.168.168.17
    access-list CSM_FW_ACL_CNES extended permit ip object-group Alamur-PC object-group Juniper
    access-list CSM_FW_ACL_CNES extended permit ip object-group Gavin_-_new_PC object-group Alder-Intranet
    access-list CSM_FW_ACL_CNES extended permit ip object-group Gavin_-_new_PC host 192.168.168.17
    access-list CSM_FW_ACL_CNES extended permit ip object-group Gavin_-_new_PC object-group Juniper
    access-list CSM_FW_ACL_CNES extended permit object-group RDP object-group NET_cnes-castlebay-staff object-group HHP_Server_Network
    access-list CSM_FW_ACL_CNES extended permit object-group RDP object-group NET_cnes_tarbert_staff object-group HHP_Server_Network
    access-list MIS_splitTunnelAcl standard permit 192.168.168.0 255.255.255.0
    access-list inside_nat0_outbound extended permit ip object-group HHP_Server_Network 192.168.168.250 255.255.255.254
    access-list inside_nat0_outbound extended permit ip object-group HHP_Server_Network 192.168.168.224 255.255.255.224
    access-list CSM_FW_ACL_DMZ extended permit ip object-group HHP_DMZ_Network object-group PC_Support
    access-list CSM_FW_ACL_DMZ extended permit icmp object-group HHP_DMZ_Network object-group HHP_Server_Network
    access-list CSM_FW_ACL_DMZ extended permit ip object-group Juniper object-group Angus-Maclean-PC
    access-list CSM_FW_ACL_DMZ extended permit ip object-group Juniper object-group Holly-AV
    access-list CSM_FW_ACL_DMZ extended permit tcp object-group Juniper object-group Beech-Exchange eq smtp
    access-list CSM_FW_ACL_DMZ extended permit tcp object-group Juniper object-group HHP_Domain_Controllers eq domain
    access-list CSM_FW_ACL_DMZ extended permit udp object-group Juniper object-group HHP_Domain_Controllers eq domain
    access-list CSM_FW_ACL_DMZ remark for backups to USB drive on ASH
    access-list CSM_FW_ACL_DMZ extended permit object-group TCP-445 object-group Juniper object-group Ash
    access-list CSM_FW_ACL_DMZ extended permit object-group UDP-445 object-group Juniper object-group Ash
    access-list CSM_FW_ACL_DMZ extended permit object-group tcp-udp-135-139 object-group Juniper object-group Ash
    access-list CSM_FW_ACL_DMZ extended deny ip any any
    access-list CNES_Support_splitTunnelAcl standard permit 192.168.168.0 255.255.255.0
    access-list RemoteAccess_splitTunnelAcl standard permit 192.168.168.0 255.255.255.0
    access-list outside_cryptomap extended permit ip object-group HHP_Server_Network object-group NET-cnes_HHP-Sty
    pager lines 24
    logging enable
    logging asdm informational
    mtu inside 1500
    mtu outside 1492
    mtu DMZ 1500
    mtu Wireless_HHP 1500
    mtu CNES 1500
    ip local pool CNES_Access 192.168.168.230-192.168.168.249
    ip local pool MIS_Support 192.168.168.250-192.168.168.251
    ip local pool OLM-VPN-Pool 192.168.168.252
    ip local pool HHP_Remote_Access_Pool 192.168.168.200-192.168.168.229
    no failover
    icmp unreachable rate-limit 1 burst-size 1
    no asdm history enable
    arp timeout 14400
    nat-control
    global (outside) 1 interface
    nat (inside) 0 access-list inside_nat0_outbound
    nat (inside) 1 0.0.0.0 0.0.0.0
    nat (Wireless_HHP) 1 172.20.193.53 255.255.255.255
    nat (Wireless_HHP) 1 172.20.225.0 255.255.255.0
    static (inside,CNES) 192.168.168.0 192.168.168.0 netmask 255.255.255.0
    static (CNES,inside) 187.187.0.0 255.255.0.0 netmask 255.255.0.0
    static (Wireless_HHP,inside) 172.20.224.0 172.20.224.0 netmask 255.255.240.0
    static (inside,Wireless_HHP) 192.168.168.0 192.168.168.0 netmask 255.255.255.0
    static (CNES,Wireless_HHP) 187.187.0.0 187.187.0.0 netmask 255.255.0.0
    static (inside,outside) 217.36.32.210 192.168.168.91 netmask 255.255.255.255
    static (DMZ,outside) 217.36.32.211 192.168.169.5 netmask 255.255.255.255
    static (inside,DMZ) 192.168.168.0 192.168.168.0 netmask 255.255.255.0
    static (CNES,DMZ) 187.0.0.0 187.0.0.0 netmask 255.0.0.0
    access-group CSM_FW_ACL_inside in interface inside
    access-group outside_access_in_1 in interface outside control-plane
    access-group outside_access_in in interface outside
    access-group CSM_FW_ACL_DMZ in interface DMZ
    access-group CSM_FW_ACL_Wireless_HHP in interface Wireless_HHP
    access-group CSM_FW_ACL_CNES in interface CNES
    route outside 0.0.0.0 0.0.0.0 81.148.0.157 1
    route Wireless_HHP 172.20.192.0 255.255.240.0 172.16.36.3 1
    route Wireless_HHP 172.20.224.0 255.255.240.0 172.16.36.2 1
    timeout xlate 3:00:00
    timeout conn 1:00:00 half-closed 0:10:00 udp 0:02:00 icmp 0:00:02
    timeout sunrpc 0:10:00 h323 0:05:00 h225 1:00:00 mgcp 0:05:00 mgcp-pat 0:05:00
    timeout sip 0:30:00 sip_media 0:02:00 sip-invite 0:03:00 sip-disconnect 0:02:00
    timeout sip-provisional-media 0:02:00 uauth 0:05:00 absolute
    timeout tcp-proxy-reassembly 0:01:00
    dynamic-access-policy-record DfltAccessPolicy
    aaa-server HHP protocol ldap
    aaa-server HHP (inside) host 192.168.168.2
    timeout 5
    ldap-base-dn dc=hhp,dc=com
    ldap-scope subtree
    ldap-naming-attribute sAMAccountName
    ldap-login-password *
    ldap-login-dn cn=gramor,cn=users,dc=hhp,dc=com
    server-type microsoft
    aaa-server HHP_1 protocol ldap
    aaa-server HHP_1 (inside) host 192.168.168.2
    timeout 5
    ldap-base-dn dc=hhp,dc=com
    ldap-scope subtree
    ldap-naming-attribute sAMAccountName
    ldap-login-password *
    ldap-login-dn cn=administrator,cn=users,dc=hhp,dc=com
    server-type microsoft
    aaa-server HHP_3 protocol ldap
    aaa-server HHP_3 (inside) host 192.168.168.2
    timeout 5
    ldap-base-dn dc=hhp,dc=com
    ldap-scope subtree
    ldap-naming-attribute sAMAccountName
    ldap-login-password *
    ldap-login-dn cn=administrator,cn=users,dc=hhp,dc=com
    server-type microsoft
    aaa authentication ssh console LOCAL
    aaa authentication http console LOCAL
    http server enable
    http 192.168.1.0 255.255.255.0 inside
    http 192.168.168.0 255.255.255.0 inside
    http 0.0.0.0 0.0.0.0 outside
    http 194.83.245.242 255.255.255.255 outside
    http 187.187.1.72 255.255.255.255 CNES
    http 187.187.10.90 255.255.255.255 CNES
    no snmp-server location
    no snmp-server contact
    snmp-server enable traps snmp authentication linkup linkdown coldstart
    crypto ipsec transform-set ESP-AES-256-MD5 esp-aes-256 esp-md5-hmac
    crypto ipsec transform-set ESP-DES-SHA esp-des esp-sha-hmac
    crypto ipsec transform-set ESP-DES-MD5 esp-des esp-md5-hmac
    crypto ipsec transform-set ESP-AES-192-MD5 esp-aes-192 esp-md5-hmac
    crypto ipsec transform-set ESP-3DES-MD5 esp-3des esp-md5-hmac
    crypto ipsec transform-set ESP-AES-256-SHA esp-aes-256 esp-sha-hmac
    crypto ipsec transform-set ESP-AES-128-SHA esp-aes esp-sha-hmac
    crypto ipsec transform-set ESP-AES-192-SHA esp-aes-192 esp-sha-hmac
    crypto ipsec transform-set ESP-AES-128-MD5 esp-aes esp-md5-hmac
    crypto ipsec transform-set ESP-3DES-SHA esp-3des esp-sha-hmac
    crypto ipsec security-association lifetime seconds 28800
    crypto ipsec security-association lifetime kilobytes 4608000
    crypto dynamic-map SYSTEM_DEFAULT_CRYPTO_MAP 65535 set pfs group1
    crypto dynamic-map SYSTEM_DEFAULT_CRYPTO_MAP 65535 set transform-set ESP-AES-128-SHA ESP-AES-128-MD5 ESP-AES-192-SHA ESP-AES-192-MD5 ESP-AES-256-SHA ESP-AES-256-MD5 ESP-3DES-SHA ESP-3DES-MD5 ESP-DES-SHA ESP-DES-MD5
    crypto dynamic-map outside_map_dynamic 2 set transform-set ESP-AES-128-SHA ESP-AES-128-MD5 ESP-AES-192-SHA ESP-AES-192-MD5 ESP-AES-256-SHA ESP-AES-256-MD5 ESP-3DES-SHA ESP-3DES-MD5 ESP-DES-SHA ESP-DES-MD5
    crypto map outside_map 1 match address outside_cryptomap
    crypto map outside_map 1 set peer 81.136.160.237
    crypto map outside_map 1 set transform-set ESP-AES-128-SHA ESP-AES-128-MD5 ESP-AES-192-SHA ESP-AES-192-MD5 ESP-AES-256-SHA ESP-AES-256-MD5 ESP-3DES-SHA ESP-3DES-MD5 ESP-DES-SHA ESP-DES-MD5
    crypto map outside_map 30001 ipsec-isakmp dynamic outside_map_dynamic
    crypto map outside_map 65535 ipsec-isakmp dynamic SYSTEM_DEFAULT_CRYPTO_MAP
    crypto map outside_map interface outside
    crypto ca trustpoint ASDM_TrustPoint0
    enrollment terminal
    fqdn none
    subject-name O=Hebridean Housing Partnership Limited,CN=secure-access.hebrideanhousing.co.uk,L=Isle of Lewis,ST=Scotland,C=GB
    keypair SSL_Certificate
    crl configure
    crypto ca trustpoint ASDM_TrustPoint1
    enrollment terminal
    fqdn none
    crl configure
    crypto ca certificate chain ASDM_TrustPoint0
    certificate 0100000000012790a5c005
        30820530 30820418 a0030201 02020b01 00000000 012790a5 c005300d 06092a86
        4886f70d 01010505 00306a31 23302106 0355040b 131a4f72 67616e69 7a617469
        6f6e2056 616c6964 6174696f 6e204341 31133011 06035504 0a130a47 6c6f6261
        6c536967 6e312e30 2c060355 04031325 476c6f62 616c5369 676e204f 7267616e
        697a6174 696f6e20 56616c69 64617469 6f6e2043 41301e17 0d313030 33323431
        34313835 385a170d 31333033 32343134 31383534 5a308197 310b3009 06035504
        06130247 42311130 0f060355 04081308 53636f74 6c616e64 31163014 06035504
        07130d49 736c6520 6f66204c 65776973 312e302c 06035504 0a132548 65627269
        6465616e 20486f75 73696e67 20506172 746e6572 73686970 204c696d 69746564
        312d302b 06035504 03132473 65637572 652d6163 63657373 2e686562 72696465
        616e686f 7573696e 672e636f 2e756b30 82012230 0d06092a 864886f7 0d010101
        05000382 010f0030 82010a02 82010100 def181d9 c34c58a8 9abcc849 7d8ad0a9
        3c64c77f f3126c81 30911f41 5903a92c 81fb374b 2fe2680e 10b26dce 81ca0c23
        af2c9f9a 52295e8c d2223fa6 7c4c386d 51c6fb16 a47688e6 e47e2410 b0283503
        fd72abd3 e59d3b02 cd47706e babf948c 4e0282a3 5f789ff7 8041b2db ceac64eb
        3e163b38 3a8ecc25 0c4802a8 d17fecd9 f1a36288 29202df4 b20ae891 f95ce055
        6e670559 3d075024 7f3ac7ef 26218154 a7f6a399 34c43c4a 97c2c88c c4588ee4
        77cc2ad8 b1bd868d d55c2b9b 727e9904 66d0fb52 c212abd7 a06f28f1 ad2aa04b
        3d7b3094 c59c00d4 cf51fefb d8bfa101 8ba9c4ba 5cf629ff c50716d3 71019a98
        8fa55b83 6b158b6d 1043f092 646ef07d 02030100 01a38201 a7308201 a3301f06
        03551d23 04183016 80147d6d 2aec66ab a75136ab 0269f170 8fc4590b 9a1f3049
        06082b06 01050507 0101043d 303b3039 06082b06 01050507 3002862d 68747470
        3a2f2f73 65637572 652e676c 6f62616c 7369676e 2e6e6574 2f636163 6572742f
        6f726776 312e6372 74303f06 03551d1f 04383036 3034a032 a030862e 68747470
        3a2f2f63 726c2e67 6c6f6261 6c736967 6e2e6e65 742f4f72 67616e69 7a617469
        6f6e5661 6c312e63 726c301d 0603551d 0e041604 14d398d5 ddf29355 15b04750
        baccc6b3 0f97a6c9 94302f06 03551d11 04283026 82247365 63757265 2d616363
        6573732e 68656272 69646561 6e686f75 73696e67 2e636f2e 756b3009 0603551d
        13040230 00300e06 03551d0f 0101ff04 04030205 a0302906 03551d25 04223020
        06082b06 01050507 03010608 2b060105 05070302 060a2b06 01040182 370a0303
        304b0603 551d2004 44304230 4006092b 06010401 a0320114 30333031 06082b06
        01050507 02011625 68747470 3a2f2f77 77772e67 6c6f6261 6c736967 6e2e6e65
        742f7265 706f7369 746f7279 2f301106 09608648 0186f842 01010404 030206c0
        300d0609 2a864886 f70d0101 05050003 82010100 8af3be01 c4830d83 9b347355
        de7496ef bd76b86c ee92f32f 1157ef11 6ad949b6 611537ad 81f06408 73ec6fe2
        6466675c cf31a80f bead422d ec574f95 55fe0b7a 97e271e7 0220c7b1 53376843
        ff7f7280 f9bfdee6 3584e123 00c37d9f 5004b766 9469ead5 f002744c fd50271c
        6bcdb54c e5db85aa 9760a330 d72464a2 bc8ecdff d80bbc27 7551e97c ee9b7078
        9207f9d6 b969a47a 6df722b6 14ce803d 8d4bb9e9 4695e8e6 d453950e 06506594
        ec7652ea 365cdf94 90e2f7ee 855dadb5 c0459d73 bb6d01a8 3c076718 7f80de40
        c5eb9e0e 17c93087 fd5c5fc1 fd6401fe 7e5038b1 3da1d250 01ccd8be 964d5557
        b320c4c1 0015d1b7 daad7527 930b0c90 7711704f
      quit
    crypto ca certificate chain ASDM_TrustPoint1
    certificate ca 0400000000011e44a5f52a
        30820467 3082034f a0030201 02020b04 00000000 011e44a5 f52a300d 06092a86
        4886f70d 01010505 00305731 0b300906 03550406 13024245 31193017 06035504
        0a131047 6c6f6261 6c536967 6e206e76 2d736131 10300e06 0355040b 1307526f
        6f742043 41311b30 19060355 04031312 476c6f62 616c5369 676e2052 6f6f7420
        4341301e 170d3037 30343131 31323030 30305a17 0d313730 34313131 32303030
        305a306a 31233021 06035504 0b131a4f 7267616e 697a6174 696f6e20 56616c69
        64617469 6f6e2043 41311330 11060355 040a130a 476c6f62 616c5369 676e312e
        302c0603 55040313 25476c6f 62616c53 69676e20 4f726761 6e697a61 74696f6e
        2056616c 69646174 696f6e20 43413082 0122300d 06092a86 4886f70d 01010105
        00038201 0f003082 010a0282 010100a1 2fc4bcce 8703e967 c189c8e5 93fc7db4
        ad9ef663 4e6ae89c 2c7389a2 01f48f21 f8fd259d 58166d86 f6ee4957 757e75ea
        22117e3d fbc74241 dcfcc50c 9155807b eb64331d 9bf9ca38 e9abc625 43512540
        f4e47e18 556aa98f 103a401e d65783ef 7f2f342f 2dd2f653 c2190db7 edc981f5
        462cb423 425e9d13 0375ecea 6afc577c c936973b 98dc1313 ecec41fa 5d34eab9
        93e71016 65cc9c92 fdf5c59d 3e4ab909 fce45f1e 695f4df4 567244b1 1d2303c8
        36f66588 c8bf3916 458e1e26 6c5116c5 2a0038c5 a4136995 7dab013b a8c414b4
        80daac1a 4420d5fe a9067b14 27afe030 21dd90f4 a9d52319 2e1e03e6 c1df9529
        e4c19443 dd3e90aa cb4bc9be 8ad33902 03010001 a382011f 3082011b 300e0603
        551d0f01 01ff0404 03020106 30120603 551d1301 01ff0408 30060101 ff020100
        301d0603 551d0e04 1604147d 6d2aec66 aba75136 ab0269f1 708fc459 0b9a1f30
        4b060355 1d200444 30423040 06092b06 010401a0 32011430 33303106 082b0601
        05050702 01162568 7474703a 2f2f7777 772e676c 6f62616c 7369676e 2e6e6574
        2f726570 6f736974 6f72792f 30330603 551d1f04 2c302a30 28a026a0 24862268
        7474703a 2f2f6372 6c2e676c 6f62616c 7369676e 2e6e6574 2f726f6f 742e6372
        6c301106 09608648 0186f842 01010404 03020204 30200603 551d2504 19301706
        0a2b0601 04018237 0a030306 09608648 0186f842 0401301f 0603551d 23041830
        16801460 7b661a45 0d97ca89 502f7d04 cd34a8ff fcfd4b30 0d06092a 864886f7
        0d010105 05000382 01010079 47fc15d7 4c79df0f 7a9eced4 7c4b63c9 89b57b3f
        9912e89c 8c9a492f e04e954a edc7bcbe f1a2db8e 931dba71 54aa4bd9 89222487
        c504a8ac 8252a052 f8b8e14f a1276663 214a39e7 c7c54e5f b2d61d13 6d30e9ce
        d7a21cbc 290a733c 5b2349fe d6ffcab0 4ff5f267 98c04711 f8b748a6 9009d642
        beeab1b9 5342c39c 20c9fba1 5bb5566d 8781c860 acc4b972 270a8e1e a8b12ecd
        32a27857 b09cf895 bb438e8c 31866e53 0dc61205 ba416ea8 35300918 1d0261ff
        fdee35de 6ac33bd0 4d4b4e50 b256360c 445dda1a 652ae698 56a96333 2e04e7ae
        e8f48eb7 b2da7dc0 c8e2aea6 282fe3c9 73bdfc07 4134b7aa 6eeea7db d1933ced
        90ec3292 88d9c823 6c7421
      quit
    crypto isakmp identity hostname
    crypto isakmp enable outside
    crypto isakmp policy 10
    authentication pre-share
    encryption 3des
    hash sha
    group 2
    lifetime 86400
    crypto isakmp policy 30
    authentication pre-share
    encryption 3des
    hash sha
    group 1
    lifetime 86400
    telnet 0.0.0.0 0.0.0.0 inside
    telnet timeout 5
    ssh 187.187.1.41 255.255.255.255 inside
    ssh 187.187.1.72 255.255.255.255 inside
    ssh 187.187.77.81 255.255.255.255 inside
    ssh 187.187.10.19 255.255.255.255 inside
    ssh 187.187.10.229 255.255.255.255 inside
    ssh 187.187.160.7 255.255.255.255 inside
    ssh 187.187.1.41 255.255.255.255 outside
    ssh 187.187.1.72 255.255.255.255 outside
    ssh 187.187.77.81 255.255.255.255 outside
    ssh 187.187.10.19 255.255.255.255 outside
    ssh 187.187.10.229 255.255.255.255 outside
    ssh 187.187.160.7 255.255.255.255 outside
    ssh timeout 15
    console timeout 0
    vpdn group BT request dialout pppoe
    vpdn group BT localname B*******.btclick.com
    vpdn group BT ppp authentication chap
    vpdn username B*******@hg39.btclick.com password *********
    dhcpd auto_config outside
    threat-detection basic-threat
    threat-detection statistics access-list
    no threat-detection statistics tcp-intercept
    ssl trust-point ASDM_TrustPoint0 outside
    ssl trust-point ASDM_TrustPoint0 outside vpnlb-ip
    webvpn
    enable inside
    enable outside
    group-policy HHP_Remote_Access_1 internal
    group-policy HHP_Remote_Access_1 attributes
    wins-server value 192.168.168.2 192.168.168.2
    dns-server value 192.168.168.2 192.168.168.3
    vpn-tunnel-protocol IPSec
    split-tunnel-policy tunnelspecified
    split-tunnel-network-list value CNES_Support_splitTunnelAcl
    group-policy HHP_Remote_Access internal
    group-policy HHP_Remote_Access attributes
    wins-server value 192.168.168.2 192.168.168.2
    dns-server value 192.168.168.2 192.168.168.3
    vpn-tunnel-protocol IPSec
    split-tunnel-policy tunnelspecified
    split-tunnel-network-list value CNES_Support_splitTunnelAcl
    group-policy Omfax internal
    group-policy Omfax attributes
    wins-server value 192.168.168.2 192.168.168.3
    dns-server value 192.168.168.2 192.168.168.3
    vpn-tunnel-protocol IPSec webvpn
    webvpn
      svc ask none default webvpn
    group-policy MIS_1 internal
    group-policy MIS_1 attributes
    wins-server value 192.168.168.2 192.168.168.3
    dns-server value 192.168.168.2 192.168.168.3
    vpn-tunnel-protocol IPSec
    split-tunnel-policy tunnelspecified
    split-tunnel-network-list value MIS_splitTunnelAcl
    default-domain value hhp.com
    group-policy RemoteAccess internal
    group-policy RemoteAccess attributes
    wins-server value 192.168.168.2 192.168.168.3
    dns-server value 192.168.168.2 192.168.168.3
    vpn-tunnel-protocol IPSec
    split-tunnel-policy tunnelspecified
    split-tunnel-network-list value RemoteAccess_splitTunnelAcl
    group-policy CNES_Access internal
    group-policy CNES_Access attributes
    wins-server value 192.168.168.2 192.168.168.3
    dns-server value 192.168.168.2 192.168.168.3
    vpn-tunnel-protocol IPSec
    split-tunnel-policy tunnelspecified
    split-tunnel-network-list value CNES_Support_splitTunnelAcl
    group-policy HHP internal
    group-policy HHP attributes
    dhcp-network-scope none
    vpn-access-hours none
    vpn-idle-timeout none
    vpn-session-timeout none
    vpn-filter none
    vpn-tunnel-protocol IPSec webvpn
    password-storage disable
    ip-comp disable
    re-xauth disable
    group-lock none
    pfs disable
    ipsec-udp disable
    split-tunnel-policy tunnelall
    split-tunnel-network-list none
    split-dns none
    secure-unit-authentication disable
    user-authentication disable
    user-authentication-idle-timeout none
    ip-phone-bypass disable
    leap-bypass disable
    nem disable
    backup-servers keep-client-config
    client-firewall none
    webvpn
      url-list value Severs
      filter none
      homepage none
      port-forward disable
      http-proxy disable
      sso-server none
      svc dtls none
      svc keep-installer none
      svc rekey time none
      svc rekey method none
      svc dpd-interval client none
      svc dpd-interval gateway none
      svc compression none
      svc modules none
      svc profiles none
      svc ask none default webvpn
      customization none
      http-comp none
      user-storage none
      storage-key none
      hidden-shares none
      smart-tunnel disable
      activex-relay disable
      file-entry disable
      file-browsing disable
      url-entry disable
      deny-message none
    group-policy MIS internal
    group-policy MIS attributes
    wins-server value 192.168.168.2 192.168.168.3
    dns-server value 192.168.168.2 192.168.168.3
    vpn-tunnel-protocol IPSec
    split-tunnel-policy tunnelspecified
    split-tunnel-network-list value MIS_splitTunnelAcl
    username test password Kg/Rgy23do7gPGTv encrypted privilege 0
    username test attributes
    vpn-group-policy HHP_Remote_Access
    username catneil password yOgiHCGobUNIkjcN encrypted privilege 0
    username omfax password pvUaCLwilGmQVifd encrypted privilege 0
    username backup password IHQbl.JAoESlM9Jv encrypted privilege 0
    username misadmin password 8IZXmHa67HIJYHK1 encrypted
    username misadmin attributes
    service-type remote-access
    username gramor password ne829U0rGFVEedhY encrypted privilege 15
    username gramor attributes
    vpn-group-policy HHP_Remote_Access
    webvpn
      url-list value Severs
    username aim_user password 5OQaWCdB18qiHlOn encrypted privilege 0
    username aim_user attributes
    vpn-group-policy CNES_Support
    username katask password 2WsX.HoqKXuiqkDk encrypted privilege 0
    username katask attributes
    vpn-group-policy CNES_Support
    username janboyd password ZEUyykwzME6hII2i encrypted privilege 0
    username marmor password C5n48AiRLXwxAeBQ encrypted privilege 0
    username marste password amwTL584WdiT87Tb encrypted privilege 0
    username helmah password RvU8c.3w0H3/MJz4 encrypted privilege 0
    username anglea password wGlUJDBrmJI.uz./ encrypted privilege 0
    username anglea attributes
    vpn-group-policy CNES_Support
    username fiobuc password 5Uispw90wqvDYerQ encrypted privilege 0
    tunnel-group DefaultRAGroup general-attributes
    authentication-server-group HHP_1
    tunnel-group DefaultWEBVPNGroup general-attributes
    authentication-server-group HHP_1
    default-group-policy HHP
    tunnel-group DefaultWEBVPNGroup webvpn-attributes
    nbns-server 192.168.168.2 timeout 2 retry 2
    nbns-server 192.168.168.3 timeout 2 retry 2
    tunnel-group WebVPN type remote-access
    tunnel-group WebVPN general-attributes
    authentication-server-group HHP_3
    default-group-policy HHP
    username-from-certificate UID
    tunnel-group CNES_Access

  • ASA 5505 Best Practice Guidance Requested

    I am hoping to tap into the vast wealth of knowledge on this board in order to gain some "best practice" guidance to assist me with the overall setup using the ASA 5505 for a small business client.  I'm fairly new to the ASA 5505 so any help would be most appreciated!
    My current client configuration is as follows:
    a) business internet service (cable) with a fixed IP address
    b) a Netgear N600 Wireless Dual Band router (currently setup as gateway and used for internet/WiFi access)
    c) a Cisco SG-500-28 switch
    d) one server running Windows Small Business Server 2011 Standard (primary Domain Controller)
         (This server is currently the DNS and DHCP server)
    e) one server running Windows Server 2008 R2 (secondary Domain Controller)
    f) approximately eight Windows 7 clients (connected via SG-500-28 switch)
    g) approximately six printers connected via internal network (connected via SG-500-28 switch)
    All the servers, clients, and printers are connected to the SG-500-28 switch.
    The ISP provides the cable modem for the internet service.
    The physical cable for internet is connected to the cable modem.
    From the cable modem, a CAT 6 ethernet cable is connected to the internet (WAN) port of the Netgear N600 router.
    A Cat 6 ethernet cable is connected from Port 1 of the local ethernet (LAN) port on the N600 router to the SG-500-28 switch.
    cable modem -> WAN router port
    LAN router port -> SG-500-28
    The ASA 5505 will be setup with an "LAN" (inside) interface and a "WAN" (outside) interface.  Port e0/0 on the ASA 5505 will be used for the outside interface and the remaining ports will be used for the inside interface.
    So my basic question is, given the information above of our setup, where should the ASA 5505 be "inserted" to maximize its performance?  Also, based on the answer to the previous question, can you provide some insight as to how the ethernet cables should be connected to achieve this?
    Another concern I have is what device will be used as the default gateway.  Currently, the Netgear N600 is set as the default gateway on both Windows servers.  In your recommended best practice solution, does the ASA 5505 become the default gateway or does the router remain the default gateway?
    And my final area of concern is with DHCP.  As I stated earlier, I am running DHCP on Windows Small Business Server 2011 Standard.  Most of the examples I have studied for the ASA 5505 utilize its DHCP functionality.  I also have done some research on the "dhcprelay server" command.  So I'm not quite sure which is the best way to go. First off, does the "dhcprelay server" even work with SBS 2011?  And secondly, if it does work, is the best practice to use the "dhcprelay" command or to let the ASA 5505 perform the DHCP server role?
    All input/guidance/suggestions with these issues would be greatly appreciated!  I want to implement the ASA 5505 firewall solution following "best practices" recommendations in order to maximize its functionality and minimize the time to implement.
    FYI, the information (from the "show version" command) for the ASA 5505 is shown below:
    Cisco Adaptive Security Appliance Software Version 8.4(7)
    Device Manager Version 7.1(5)100
    Compiled on Fri 30-Aug-13 19:48 by builders
    System image file is "disk0:/asa847-k8.bin"
    Config file at boot was "startup-config"
    ciscoasa up 2 days 9 hours
    Hardware:   ASA5505, 512 MB RAM, CPU Geode 500 MHz
    Internal ATA Compact Flash, 128MB
    BIOS Flash M50FW016 @ 0xfff00000, 2048KB
    Encryption hardware device : Cisco ASA-5505 on-board accelerator (revision 0x0)
                                 Boot microcode   : CN1000-MC-BOOT-2.00
                                 SSL/IKE microcode: CNLite-MC-SSLm-PLUS-2.03
                                 IPSec microcode  : CNlite-MC-IPSECm-MAIN-2.06
                                 Number of accelerators: 1
    0: Int: Internal-Data0/0    : address is a493.4c99.8c0b, irq 11
    1: Ext: Ethernet0/0         : address is a493.4c99.8c03, irq 255
    2: Ext: Ethernet0/1         : address is a493.4c99.8c04, irq 255
    3: Ext: Ethernet0/2         : address is a493.4c99.8c05, irq 255
    4: Ext: Ethernet0/3         : address is a493.4c99.8c06, irq 255
    5: Ext: Ethernet0/4         : address is a493.4c99.8c07, irq 255
    6: Ext: Ethernet0/5         : address is a493.4c99.8c08, irq 255
    7: Ext: Ethernet0/6         : address is a493.4c99.8c09, irq 255
    8: Ext: Ethernet0/7         : address is a493.4c99.8c0a, irq 255
    9: Int: Internal-Data0/1    : address is 0000.0003.0002, irq 255
    10: Int: Not used            : irq 255
    11: Int: Not used            : irq 255
    Licensed features for this platform:
    Maximum Physical Interfaces       : 8              perpetual
    VLANs                             : 3              DMZ Restricted
    Dual ISPs                         : Disabled       perpetual
    VLAN Trunk Ports                  : 0              perpetual
    Inside Hosts                      : 10             perpetual
    Failover                          : Disabled       perpetual
    VPN-DES                           : Enabled        perpetual
    VPN-3DES-AES                      : Enabled        perpetual
    AnyConnect Premium Peers          : 2              perpetual
    AnyConnect Essentials             : Disabled       perpetual
    Other VPN Peers                   : 10             perpetual
    Total VPN Peers                   : 12             perpetual
    Shared License                    : Disabled       perpetual
    AnyConnect for Mobile             : Disabled       perpetual
    AnyConnect for Cisco VPN Phone    : Disabled       perpetual
    Advanced Endpoint Assessment      : Disabled       perpetual
    UC Phone Proxy Sessions           : 2              perpetual
    Total UC Proxy Sessions           : 2              perpetual
    Botnet Traffic Filter             : Disabled       perpetual
    Intercompany Media Engine         : Disabled       perpetual
    This platform has a Base license.

    Hey Jon,
    Again, many thanks for the info!
    I guess I left that minor detail out concerning the Guest network.  I have a second Netgear router that I am using for Guest netowrk access.  It is plugged in to one of the LAN network ports on the first Netgear router.
    The second Netgear (Guest) router is setup on a different subnet and I am letting the router hand out IP addresses using DHCP.
    Basic setup is the 192.168.1.x is the internal network and 192.168.11.x is the Guest network.  As far as the SBS 2011 server, it knows nothing about the Guest network in terms of the DHCP addresses it hands out.
    Your assumption about the Guest network is correct, I only want to allow guest access to the internet and no access to anything internal.  I like your idea of using the restricted DMZ feature of the ASA for the Guest network.  (I don't know how to do it, but I like it!)  Perhaps you could share more of your knowledge on this?
    One final thing, the (internal) Netgear router setup does provide the option for a separate Guest network, however it all hinges on the router being the DHCP server.  This is what led me to the second (Guest) Netgear router because I wanted the (internal) Netgear router NOT to use DHCP.  Instead I wanted SBS 2011 to be the DHCP server.  That's what led to the idea of a second (Guest) router with DHCP enabled.
    The other factor in all this is SBS 2011.  Not sure what experience you've had with the Small Business Server OS's but they tend to get a little wonky if some of the server roles are disabled.  For instance, this is a small busines with a total of about 20 devices including servers, workstations and printers.  Early on I thought, "nah, I don't need this IPv6 stuff," so I found an article on how to disable it and did so.  The server performance almost immediately took a nose dive.  Rebooting the server went from a 5 minute process to a 20 minute process.  And this was after I followed the steps of an MSDN article on disabling IPv6 on SBS 2011!  Well, long story short, I enabled IPv6 again and the two preceeding issues cleared right up.  So, since SBS 2011 by "default" wants DHCP setup I want to try my best to accomodate it.  So, again, your opinion/experiece related to this is a tremendous help!
    Thanks!

  • Problem with ASA 5505

    Good morning,
    I'm having the following problem. I configured a ASA 5505 with VPN and a VPN Remote Access Site-to-site. Everything is working, but when I reload the ASA does not work anymore VPNs, Remote Access error 412 and the Site-to-site does not connect more to solve, I have to reset and reconfigure the ASA. This is happening dopo updating the ASA, I have version 842-k8 and asdm645-106.
    Does anyone have any idea what can be?
    Thank you.
    Running-config:
    : Saved
    : Written by master at 10:34:14.839 BRDT Mon Oct 10 2011
    ASA Version 8.4(2)
    hostname ciscoasa
    domain-name default.domain.invalid
    enable password 2KFQnbNIdI.2KYOU encrypted
    passwd 2KFQnbNIdI.2KYOU encrypted
    names
    interface Ethernet0/0
    switchport access vlan 2
    interface Ethernet0/1
    interface Ethernet0/2
    interface Ethernet0/3
    interface Ethernet0/4
    interface Ethernet0/5
    interface Ethernet0/6
    interface Ethernet0/7
    interface Vlan1
    nameif inside
    security-level 0
    ip address 172.16.0.140 255.255.252.0
    interface Vlan2
    nameif outside
    security-level 0
    pppoe client vpdn group gvt
    ip address pppoe setroute
    boot system disk0:/asa842-k8.bin
    ftp mode passive
    clock timezone BRST -3
    clock summer-time BRDT recurring 2 Sun Oct 0:00 3 Sun Feb 0:00
    dns server-group DefaultDNS
    domain-name default.domain.invalid
    same-security-traffic permit inter-interface
    same-security-traffic permit intra-interface
    object network NETWORK_OBJ_172.16.0.0_22
    subnet 172.16.0.0 255.255.252.0
    object network NETWORK_OBJ_172.16.0.128_26
    subnet 172.16.0.128 255.255.255.192
    object network NETWORK_OBJ_20.0.0.0_24
    subnet 20.0.0.0 255.255.255.0
    object network NETWORK_OBJ_172.16.11.0_24
    subnet 172.16.11.0 255.255.255.0
    object-group network obj_any
    access-list 1 standard permit 172.16.0.0 255.255.252.0
    access-list 1 standard permit 20.0.0.0 255.255.255.0
    access-list outside_cryptomap extended permit ip 172.16.0.0 255.255.252.0 20.0.0.0 255.255.255.0
    access-list outside_cryptomap_1 extended permit ip 172.16.0.0 255.255.252.0 172.16.11.0 255.255.255.0
    pager lines 24
    logging asdm informational
    mtu inside 1500
    mtu outside 1500
    ip local pool pool 172.16.0.150-172.16.0.160 mask 255.255.252.0
    no failover
    icmp unreachable rate-limit 1 burst-size 1
    asdm image disk0:/asdm-645-106.bin
    no asdm history enable
    arp timeout 14400
    nat (inside,outside) source static NETWORK_OBJ_172.16.0.0_22 NETWORK_OBJ_172.16.0.0_22 destination static NETWORK_OBJ_172.16.0.128_26 NETWORK_OBJ_172.16.0.128_26 no-proxy-arp route-lookup
    nat (inside,outside) source static NETWORK_OBJ_172.16.0.0_22 NETWORK_OBJ_172.16.0.0_22 destination static NETWORK_OBJ_20.0.0.0_24 NETWORK_OBJ_20.0.0.0_24 no-proxy-arp route-lookup
    nat (inside,outside) source static NETWORK_OBJ_172.16.0.0_22 NETWORK_OBJ_172.16.0.0_22 destination static NETWORK_OBJ_172.16.11.0_24 NETWORK_OBJ_172.16.11.0_24 no-proxy-arp route-lookup
    nat (inside,outside) after-auto source dynamic any interface
    route outside 172.16.11.0 255.255.255.0 187.16.33.131 10
    timeout xlate 3:00:00
    timeout conn 1:00:00 half-closed 0:10:00 udp 0:02:00 icmp 0:00:02
    timeout sunrpc 0:10:00 h323 0:05:00 h225 1:00:00 mgcp 0:05:00 mgcp-pat 0:05:00
    timeout sip 0:30:00 sip_media 0:02:00 sip-invite 0:03:00 sip-disconnect 0:02:00
    timeout sip-provisional-media 0:02:00 uauth 0:05:00 absolute
    timeout tcp-proxy-reassembly 0:01:00
    timeout floating-conn 0:00:00
    dynamic-access-policy-record DfltAccessPolicy
    user-identity default-domain LOCAL
    http server enable
    http 172.16.0.0 255.255.252.0 inside
    no snmp-server location
    no snmp-server contact
    crypto ipsec ikev1 transform-set ESP-AES-256-MD5 esp-aes-256 esp-md5-hmac
    crypto ipsec ikev1 transform-set ESP-DES-SHA esp-des esp-sha-hmac
    crypto ipsec ikev1 transform-set ESP-3DES-SHA esp-3des esp-sha-hmac
    crypto ipsec ikev1 transform-set ESP-DES-MD5 esp-des esp-md5-hmac
    crypto ipsec ikev1 transform-set ESP-AES-192-MD5 esp-aes-192 esp-md5-hmac
    crypto ipsec ikev1 transform-set ESP-3DES-MD5 esp-3des esp-md5-hmac
    crypto ipsec ikev1 transform-set ESP-AES-256-SHA esp-aes-256 esp-sha-hmac
    crypto ipsec ikev1 transform-set ESP-AES-128-SHA esp-aes esp-sha-hmac
    crypto ipsec ikev1 transform-set ESP-AES-192-SHA esp-aes-192 esp-sha-hmac
    crypto ipsec ikev1 transform-set ESP-AES-128-MD5 esp-aes esp-md5-hmac
    crypto ipsec ikev2 ipsec-proposal DES
    protocol esp encryption des
    protocol esp integrity sha-1 md5
    crypto ipsec ikev2 ipsec-proposal 3DES
    protocol esp encryption 3des
    protocol esp integrity sha-1 md5
    crypto ipsec ikev2 ipsec-proposal AES
    protocol esp encryption aes
    protocol esp integrity sha-1 md5
    crypto ipsec ikev2 ipsec-proposal AES192
    protocol esp encryption aes-192
    protocol esp integrity sha-1 md5
    crypto ipsec ikev2 ipsec-proposal AES256
    protocol esp encryption aes-256
    protocol esp integrity sha-1 md5
    crypto dynamic-map SYSTEM_DEFAULT_CRYPTO_MAP 65535 set ikev1 transform-set ESP-AES-128-SHA ESP-AES-128-MD5 ESP-AES-192-SHA ESP-AES-192-MD5 ESP-AES-256-SHA ESP-AES-256-MD5 ESP-3DES-SHA ESP-3DES-MD5 ESP-DES-SHA ESP-DES-MD5
    crypto map outside_map 1 match address outside_cryptomap
    crypto map outside_map 1 set peer 189.11.56.237
    crypto map outside_map 1 set ikev1 transform-set ESP-AES-128-SHA ESP-AES-128-MD5 ESP-AES-192-SHA ESP-AES-192-MD5 ESP-AES-256-SHA ESP-AES-256-MD5 ESP-3DES-SHA ESP-3DES-MD5 ESP-DES-SHA ESP-DES-MD5
    crypto map outside_map 1 set ikev2 ipsec-proposal AES256 AES192 AES 3DES DES
    crypto map outside_map 2 match address outside_cryptomap_1
    crypto map outside_map 2 set peer 187.16.33.131
    crypto map outside_map 2 set ikev1 transform-set ESP-AES-128-SHA ESP-AES-128-MD5 ESP-AES-192-SHA ESP-AES-192-MD5 ESP-AES-256-SHA ESP-AES-256-MD5 ESP-3DES-SHA ESP-3DES-MD5 ESP-DES-SHA ESP-DES-MD5
    crypto map outside_map 2 set ikev2 ipsec-proposal DES 3DES AES AES192 AES256
    crypto map outside_map 65535 ipsec-isakmp dynamic SYSTEM_DEFAULT_CRYPTO_MAP
    crypto map outside_map interface outside
    crypto ikev2 policy 1
    encryption aes-256
    integrity sha
    group 5 2
    prf sha
    lifetime seconds 86400
    crypto ikev2 policy 10
    encryption aes-192
    integrity sha
    group 5 2
    prf sha
    lifetime seconds 86400
    crypto ikev2 policy 20
    encryption aes
    integrity sha
    group 5 2
    prf sha
    lifetime seconds 86400
    crypto ikev2 policy 30
    encryption 3des
    integrity sha
    group 5 2
    prf sha
    lifetime seconds 86400
    crypto ikev2 policy 40
    encryption des
    integrity sha
    group 5 2
    prf sha
    lifetime seconds 86400
    crypto ikev2 enable outside
    crypto ikev1 enable outside
    crypto ikev1 ipsec-over-tcp port 10000
    crypto ikev1 policy 10
    authentication crack
    encryption aes-256
    hash sha
    group 2
    lifetime 86400
    crypto ikev1 policy 20
    authentication rsa-sig
    encryption aes-256
    hash sha
    group 2
    lifetime 86400
    crypto ikev1 policy 30
    authentication pre-share
    encryption aes-256
    hash sha
    group 2
    lifetime 86400
    crypto ikev1 policy 40
    authentication crack
    encryption aes-192
    hash sha
    group 2
    lifetime 86400
    crypto ikev1 policy 50
    authentication rsa-sig
    encryption aes-192
    hash sha
    group 2
    lifetime 86400
    crypto ikev1 policy 60
    authentication pre-share
    encryption aes-192
    hash sha
    group 2
    lifetime 86400
    crypto ikev1 policy 70
    authentication crack
    encryption aes
    hash sha
    group 2
    lifetime 86400
    crypto ikev1 policy 80
    authentication rsa-sig
    encryption aes
    hash sha
    group 2
    lifetime 86400
    crypto ikev1 policy 90
    authentication pre-share
    encryption aes
    hash sha
    group 2
    lifetime 86400
    crypto ikev1 policy 100
    authentication crack
    encryption 3des
    hash sha
    group 2
    lifetime 86400
    crypto ikev1 policy 110
    authentication rsa-sig
    encryption 3des
    hash sha
    group 2
    lifetime 86400
    crypto ikev1 policy 120
    authentication pre-share
    encryption 3des
    hash sha
    group 2
    lifetime 86400
    crypto ikev1 policy 130
    authentication crack
    encryption des
    hash sha
    group 2
    lifetime 86400
    crypto ikev1 policy 140
    authentication rsa-sig
    encryption des
    hash sha
    group 2
    lifetime 86400
    crypto ikev1 policy 150
    authentication pre-share
    encryption des
    hash sha
    group 2
    lifetime 86400
    telnet timeout 5
    ssh timeout 5
    console timeout 0
    vpdn group gvt request dialout pppoe
    vpdn group gvt localname *******@turbonetpro
    vpdn group gvt ppp authentication pap
    vpdn username *******@turbonetpro password *****
    dhcpd auto_config outside
    dhcpd address 172.16.0.144-172.16.1.143 inside
    threat-detection basic-threat
    threat-detection statistics access-list
    no threat-detection statistics tcp-intercept
    webvpn
    group-policy crv internal
    group-policy crv attributes
    dns-server value 172.16.0.253 8.8.8.8
    vpn-tunnel-protocol ikev1
    split-tunnel-policy tunnelspecified
    split-tunnel-network-list value 1
    default-domain value crvnatural.com.br
    group-policy GroupPolicy_189.11.56.237 internal
    group-policy GroupPolicy_189.11.56.237 attributes
    vpn-filter value 1
    vpn-tunnel-protocol ikev1 ikev2
    group-policy GroupPolicy_187.16.33.131 internal
    group-policy GroupPolicy_187.16.33.131 attributes
    vpn-filter value 1
    vpn-tunnel-protocol ikev1 ikev2
    username master password kWH7f2vqtjMEg2Yp encrypted
    tunnel-group crv type remote-access
    tunnel-group crv general-attributes
    default-group-policy crv
    dhcp-server 172.16.0.253
    tunnel-group crv ipsec-attributes
    ikev1 pre-shared-key *****
    tunnel-group 189.11.**.*** type ipsec-l2l
    tunnel-group 189.11.**.*** general-attributes
    default-group-policy GroupPolicy_189.11.**.***
    tunnel-group 189.11.**.*** ipsec-attributes
    ikev1 pre-shared-key *****
    ikev2 remote-authentication pre-shared-key ****
    ikev2 local-authentication pre-shared-key *****
    tunnel-group 187.16.33.*** type ipsec-l2l
    tunnel-group 187.16.33.*** general-attributes
    default-group-policy GroupPolicy_187.16.33.***
    tunnel-group 187.16.33.*** ipsec-attributes
    ikev1 pre-shared-key ******
    ikev2 remote-authentication pre-shared-key *****
    ikev2 local-authentication pre-shared-key *****
    class-map inspection_default
    match default-inspection-traffic
    policy-map type inspect dns preset_dns_map
    parameters
      message-length maximum client auto
      message-length maximum 512
    policy-map global_policy
    class inspection_default
      inspect dns preset_dns_map
      inspect ftp
      inspect h323 h225
      inspect h323 ras
      inspect rsh
      inspect rtsp
      inspect esmtp
      inspect sqlnet
      inspect skinny 
      inspect sunrpc
      inspect xdmcp
      inspect sip 
      inspect netbios
      inspect tftp
      inspect ip-options
    service-policy global_policy global
    prompt hostname context
    call-home
    profile CiscoTAC-1
      no active
      destination address http https://tools.cisco.com/its/service/oddce/services/DDCEService
      destination address email [email protected]
      destination transport-method http
      subscribe-to-alert-group diagnostic
      subscribe-to-alert-group environment
      subscribe-to-alert-group inventory periodic monthly
      subscribe-to-alert-group configuration periodic monthly
      subscribe-to-alert-group telemetry periodic daily
    Cryptochecksum:50ed6f55182534a2429d065a26e9b45c
    : end

    David,
    In order to understand why LDAP is not working run a "debug ldap 255" and then try to login or run a AAA test.
    Attach the output to find out the issue.
    Please check this out as well, to make sure that you have the correct settings:
    ASA 8.0: Configure LDAP Authentication for WebVPN Users
    HTH.
    Portu.

  • Asa 5505, the outside cant access to a server in the inside

    hi, i have an Asa 5505, a pc in the outside with the ip 10.1.1.6 cant access to a server in the inside 192.168.1.4, pls help...
    this is my conf:
    ASA Version 8.0(4)
    hostname ciscoasa
    domain-name default.domain.invalid
    enable password 8Ry2YjIyt7RRXU24 encrypted
    passwd 2KFQnbNIdI.2KYOU encrypted
    names
    interface Vlan1
    nameif inside
    security-level 0
    ip address 192.168.1.1 255.255.255.0
    interface Vlan2
    nameif outside
    security-level 0
    ip address 10.1.1.2 255.255.255.0
    interface Ethernet0/0
    switchport access vlan 2
    interface Ethernet0/1
    interface Ethernet0/2
    interface Ethernet0/3
    interface Ethernet0/4
    interface Ethernet0/5
    interface Ethernet0/6
    interface Ethernet0/7
    boot system disk0:/asa804-k8.bin
    ftp mode passive
    dns server-group DefaultDNS
    domain-name default.domain.invalid
    object-group protocol TCPUDP
    protocol-object udp
    protocol-object tcp
    access-list 100 extended permit tcp any host 10.1.1.3 eq www
    pager lines 24
    logging enable
    logging asdm debugging
    mtu inside 1500
    <--- More --->
    mtu outside 1500
    no failover
    icmp unreachable rate-limit 1 burst-size 1
    asdm image disk0:/asdm-613.bin
    no asdm history enable
    arp timeout 14400
    nat-control
    global (outside) 1 interface
    nat (inside) 1 0.0.0.0 0.0.0.0
    static (inside,outside) 10.1.1.3 192.168.1.4 netmask 255.255.255.255
    access-group 100 in interface outside
    timeout xlate 3:00:00
    timeout conn 1:00:00 half-closed 0:10:00 udp 0:02:00 icmp 0:00:02
    timeout sunrpc 0:10:00 h323 0:05:00 h225 1:00:00 mgcp 0:05:00 mgcp-pat 0:05:00
    timeout sip 0:30:00 sip_media 0:02:00 sip-invite 0:03:00 sip-disconnect 0:02:00
    timeout sip-provisional-media 0:02:00 uauth 0:05:00 absolute
    dynamic-access-policy-record DfltAccessPolicy
    http server enable
    http 192.168.1.0 255.255.255.0 inside
    no snmp-server location
    no snmp-server contact
    snmp-server enable traps snmp authentication linkup linkdown coldstart
    crypto ipsec security-association lifetime seconds 28800
    crypto ipsec security-association lifetime kilobytes 4608000
    <--- More --->
    telnet timeout 5
    ssh timeout 5
    console timeout 0
    dhcpd auto_config outside
    dhcpd address 192.168.1.2-192.168.1.254 inside
    dhcpd enable inside
    threat-detection basic-threat
    threat-detection statistics access-list
    no threat-detection statistics tcp-intercept
    class-map inspection_default
    match default-inspection-traffic
    policy-map type inspect dns preset_dns_map
    parameters
    message-length maximum 512
    policy-map global_policy
    class inspection_default
    inspect dns preset_dns_map
    inspect ftp
    <--- More --->
    inspect h323 h225
    inspect h323 ras
    inspect rsh
    inspect rtsp
    inspect esmtp
    inspect sqlnet
    inspect skinny
    inspect sunrpc
    inspect xdmcp
    inspect sip
    inspect netbios
    inspect tftp
    service-policy global_policy global
    prompt hostname context
    Cryptochecksum:14e7b74fabc386613ae646b915f60e9e
    : end
    ciscoasa#

    Andres
    The security level for your inside interface should be 100 ie.
    interface Vlan1
    nameif inside
    security-level 100
    ip address 192.168.1.1 255.255.255.0
    After changing that can you
    1) ping the outside interface of the ASA from the pc or ping the PC from the ASA
    2) I'm assuming you are trying to connect to 10.1.1.3 when you attempt the connection ?
    Jon

  • Connect Inside to Outside in ASA 5505

    Hi there,
    I have a test ASA 5505 with the setting below:
    How can I connect to the internet (Vlan 1 to VLan 11)
    TestASA5505#  show run
    : Saved
    ASA Version 8.2(4)
    hostname TestASA5505
    domain-name default.domain.invalid
    enable password 8Ry2YjIyt7RRXU24 encrypted
    passwd 2KFQnbNIdI.2KYOU encrypted
    names
    interface Ethernet0/0
    switchport access vlan 11
    interface Ethernet0/1
    switchport access vlan 3
    interface Ethernet0/2
    interface Ethernet0/3
    interface Ethernet0/4
    interface Ethernet0/5
    interface Ethernet0/6
    interface Ethernet0/7
    interface Vlan1
    nameif inside
    security-level 100
    ip address 192.168.99.1 255.255.255.0
    interface Vlan11
    nameif outside
    security-level 0
    ip address 192.168.1.4 255.255.255.0
    boot system disk0:/asa824-k8.bin
    ftp mode passive
    dns server-group DefaultDNS
    domain-name default.domain.invalid
    pager lines 24
    logging enable
    logging asdm informational
    mtu inside 1500
    mtu outside 1500
    icmp unreachable rate-limit 1 burst-size 1
    asdm image disk0:/asdm-647.bin
    no asdm history enable
    arp timeout 14400
    route outside 0.0.0.0 255.255.255.255 192.168.1.4 1
    timeout xlate 3:00:00
    timeout conn 1:00:00 half-closed 0:10:00 udp 0:02:00 icmp 0:00:02
    timeout sunrpc 0:10:00 h323 0:05:00 h225 1:00:00 mgcp 0:05:00 mgcp-pat 0:05:00
    timeout sip 0:30:00 sip_media 0:02:00 sip-invite 0:03:00 sip-disconnect 0:02:00
    timeout sip-provisional-media 0:02:00 uauth 0:05:00 absolute
    timeout tcp-proxy-reassembly 0:01:00
    dynamic-access-policy-record DfltAccessPolicy
    http server enable
    http 192.168.99.0 255.255.255.0 inside
    no snmp-server location
    no snmp-server contact
    snmp-server enable traps snmp authentication linkup linkdown coldstart
    crypto ipsec security-association lifetime seconds 28800
    crypto ipsec security-association lifetime kilobytes 4608000
    telnet timeout 5
    ssh timeout 5
    console timeout 0
    management-access inside
    dhcpd address 192.168.99.3-192.168.99.30 inside
    dhcpd enable inside
    threat-detection basic-threat
    threat-detection statistics access-list
    no threat-detection statistics tcp-intercept
    ssl encryption 3des-sha1 aes128-sha1 aes256-sha1 rc4-md5
    webvpn
    username admin password S1xyD1w.ZbjUT1yX encrypted privilege 15
    policy-map type inspect dns preset_dns_map
    parameters
      message-length maximum 512
    prompt hostname context
    call-home
    profile CiscoTAC-1
      no active
      destination address http https://tools.cisco.com/its/service/oddce/services/DDCEService
      destination address email [email protected]
      destination transport-method http
      subscribe-to-alert-group diagnostic
      subscribe-to-alert-group environment
      subscribe-to-alert-group inventory periodic monthly
      subscribe-to-alert-group configuration periodic monthly
      subscribe-to-alert-group telemetry periodic daily
    Cryptochecksum:096682b0996d6a1cad76597c01ffe5e2
    : end
    TestASA5505#
    Thank you in Advance for your time

    Hi,
    What device is in front of the ASA?
    Is there some ADSL modem doing NAT and providing Internet connection or something?
    One obvious problem in the above configuration is the route
    route outside 0.0.0.0 255.255.255.255 192.168.1.4 1
    Its not actually even a default route and furthermore its pointing to the ASA itself
    It should be something like this
    route outside 0.0.0.0 0.0.0.0 192.168.1.x
    Where the 192.168.1.x is the IP of the device providing the Internet connectivity to the ASA (Since ASA "outside" interface is using private IP address range)
    If there ASA doesnt need to do any NAT then you could also add this
    access-list INSIDE-NAT0 permit ip 192.168.99.0 255.255.255.0 any
    nat (inside) 0 access-list INSIDE-NAT0
    Also your DHCP configurations dont have any DNS servers defined.
    dhcpd dns
    - Jouni

Maybe you are looking for